Anda di halaman 1dari 194

Buku 1 KTO Matematika

Soal dan Solusi Kontes Juni–Desember 2015

Tim KTO Matematika

Penerbit Graha Ilmu Mulia


Buku 1 KTO Matematika
Soal dan Solusi Kontes Juni – Desember 2015

Penyusun:

Adi Suryanata Herwana | Erlang Wiratama Surya


Fransisca Susan Gozali | Jonathan Mulyawan Woenardi
Kevin Christian Wibisono | Made Tantrawan

Penyunting:

Edbert Theda | Eunice Sukarto


Herbert Ilhan Tanujaya | Otto Alexander Sutianto
Raja Oktovin | Reza Wahyu Kumara
Rio Fandi Dianco | Ronald Widjojo

ISBN: 978-602-6230-14-0
Cetakan 2: Desember 2016

Penerbit:

Penerbit Graha Ilmu Mulia


Jl. Ploso Timur IX/23
Telp. (031) 3897678
HP. 082141944261
Surabaya
Kata Pengantar

Pertama, saya ingin mengucapkan puji syukur kepada Tuhan Yang Maha Esa,
karena atas izin-Nya tim Kontes Terbuka Olimpiade Matematika mampu me-
nyelesaikan buku kumpulan soal dan solusi ini.
Saat ini, Kontes Terbuka Olimpiade Matematika sudah berjalan satu se-
tengah tahun lebih. Inisiatif yang terbentuk dari hasil diskusi sekitar enam
orang ini sudah berkembang menjadi tim yang meliputi sekitar 40 orang; kami
juga sudah berhasil menjalankan hampir 20 kali kontes, dimulai dari Simu-
lasi OSN 2015. Selain itu, kami sudah melakukan berbagai perkembangan
yang meliputi laman tersendiri di ktom.tomi.or.id, buku KTO Matematika ke-
dua yang berisi kumpulan soal dan solusi KTOM Januari–Juni 2016, official
account LINE yang berisi berbagai kerja sama yang sedang kami lakukan de-
ngan lomba-lomba matematika di Indonesia.
Semua ini kami lakukan untuk mewujudkan visi kami: olimpiade mate-
matika untuk semua. Kami ingin memajukan pendidikan olimpiade matema-
tika di Indonesia, dari Sabang sampai Merauke. Oleh karena itu, kami ber-
komitmen untuk terus mengadakan kontes ini secara rutin, online dan gratis.
Kualitas lomba ini juga bisa terus terjaga berkat kerja keras tim kami, yang
semuanya merupakan pemenang lomba nasional maupun internasional seper-
ti Olimpiade Sains Nasional dan International Mathematical Olympiad, ajang
perlombaan matematika tingkat SMA yang paling prestisius; pada saat ini,
terdapat setidaknya 15 orang dari tim kami yang meraih medali emas OSN
dan 10 orang dari tim kami yang meraih medali IMO.
Buku yang Anda pegang ini juga merupakan hasil perwujudan misi kami.
Selain mengandung kumpulan soal-soal KTO Matematika yang diadakan pa-
da bulan-bulan sebelumnya, buku ini juga berisi pembahasan yang mendetail
sehingga pembaca bisa memahami dengan dalam konsep-konsep yang diguna-

iii
Buku KTO Matematika 1

kan untuk memecahkan soal-soal tersebut. Kami juga memberikan komentar-


komentar yang bermanfaat di setiap soal. Komentar ini bisa mengandung hasil
olahan statistik partisipan kontes, motivasi di balik setiap solusi, buku-buku
lanjutan yang kami rekomendasikan untuk pendalaman materi, maupun ke-
salahan umum yang dilakukan para peserta (terutama untuk bagian esai).
Pada cetakan kedua ini, kami sudah memperbaiki berbagai kesalahan yang
ada, baik di bahasa maupun di solusi yang kami sajikan. Kami juga sudah ber-
usaha memberikan komentar maupun langkah-langkah tambahan ke bebera-
pa solusi agar menjadi lebih jelas.
Puji Tuhan, buku KTOM 1 cetakan pertama telah terjual habis, yakni se-
kitar 500 buku ke berbagai daerah di Indonesia. Buku yang terjual ini juga
termasuk berbagai kerja sama dengan lomba-lomba di Indonesia yang meli-
puti pendistribusian sekitar 200 buku, beberapa di antaranya gratis. Tampak
bahwa buku tersebut sudah sangat membantu berbagai orang di Indonesia de-
ngan adanya solusi komprehensif di setiap soal dari para ahlinya. Buku KTOM
2 yang baru saja mulai dijual beberapa bulan yang lalu juga cukup diterima
oleh masyarakat.
Kami menyarankan Anda untuk mengerjakan setiap soal tanpa melihat ja-
wabannya terlebih dahulu. Berdasarkan pengalaman, kami semua menyetu-
jui bahwa teknik belajar dengan hanya membaca soal dan solusinya sangatlah
tidak efektif. Rajin berlatih adalah kunci kesuksesan dalam olimpiade mate-
matika. Jika Anda menemui jalan buntu, kami menyediakan beberapa petun-
juk dalam berbagai tingkat untuk mengarahkan Anda ke solusi. Saran saya,
habiskan sekitar 30 menit untuk mengerjakan setiap soal; jika Anda tidak me-
nemukan ide, Anda baru melihat petunjuk-petunjuk yang kami sediakan. Jika
masih belum menemui solusinya, Anda dapat merujuk kepada pembahasan
yang kami berikan. Ini semua akan membuat pembelajaran matematika Anda
menjadi jauh lebih bermanfaat.
Pada kesempatan ini, saya ingin mengucapkan terima kasih kepada para
penyusun maupun penyunting buku ini, di antaranya:

• Kevin Christian Wibisono (medalis emas ke-2 OSN 2012, perunggu IMO
2013), sebagai ketua tim penyusun buku ini,

• Rio Fandi Dianco (pelatihan nasional tahap 2 untuk IMO 2016), sebagai
ketua tim penyunting buku ini untuk cetakan ke-2,

• Jonathan Mulyawan Woenardi (medalis emas ke-1 OSN 2013, perak IMO
2014),

iv
Kata Pengantar

• Fransisca Susan Gozali (medalis emas ke-1 OSN 2011, perak IMO 2013),

• Erlang Wiratama Surya (medalis emas ke-3 OSN 2014, perunggu IMO
2015),

• Adi Suryanata Herwana (medalis emas ke-2 OSN 2013, perak IMO 2015),

• Made Tantrawan (medalis emas ke-4 OSN 2008),

• Raja Oktovin Parhasian Damanik (perunggu IMO 2010),

• Ronald Widjojo (perunggu IMO 2009),

• Reza Wahyu Kumara (perunggu IMO 2013 dan 2014),

• Eunice Sukarto (pelatihan nasional tahap 2 untuk IMO 2016 dan IMO
2017),

• Edbert Theda (pelatihan nasional tahap 1 untuk IMO 2016), dan

• Otto Alexander Sutianto (pelatihan nasional tahap 2 untuk IMO 2016


dan IMO 2017).

Saya juga ingin mengucapkan terima kasih yang sebesar-besarnya kepada


Satria Stanza Pramayoga (peserta IMO 2008) dan Moh. Yasya Bahrul Ulum
(medali emas ke-1 OSN 2012) yang telah membantu dalam berbagai hal tek-
nis terkait buku KTO Matematika, mulai dari percetakan, penjualan hingga
distribusi buku tersebut. Kesuksesan buku tersebut tidak akan terjadi tanpa
antusiasme mereka yang sangat besar. Terima kasih juga saya ucapkan kepa-
da penerbit kami Graha Ilmu Mulia yang membantu kami dalam percetakan,
penyuntingan, dan berbagai hal sehingga buku tersebut bisa menjadi nyata.
Selain itu, saya ingin mengucapkan terima kasih kepada seluruh panitia
KTO Matematika yang meliputi tim soal, koreksi, quality check, desain, dan
official account LINE. Tanpa kerja mereka yang saling bahu-membahu, KTO
Matematika tidak akan berjalan dan buku ini juga tidak mungkin terbuat.
Saya juga ingin mengucapkan banyak terima kasih kepada semua orang yang
sudah berkontribusi ke buku ini sehingga buku ini bisa terwujudkan.
Akhir kata, kami yakin bahwa buku ini masih jauh dari sempurna. Ka-
mi mohon maaf apabila terdapat kekeliruan dalam bentuk apa pun di buku
ini: kesalahan pengetikan, kesalahan pemilihan kata, kata-kata yang kurang

v
Buku KTO Matematika 1

berkenan bagi pembaca, dan lain-lain. Oleh karena itu, apabila Anda memili-
ki kritik ataupun saran yang bersifat membangun untuk perbaikan buku ini,
Anda bisa mengirimkan surat elektronik ke mail@ktom.tomi.or.id.

Jakarta, 10 Desember 2016

Herbert Ilhan Tanujaya


Ketua Tim KTO Matematika

vi
Mengenai Buku Ini

Buku yang sedang Anda baca ini berisikan kompilasi soal tujuh bulan penye-
lenggaraan Kontes Terbuka Olimpiade Matematika (KTO Matematika) diser-
tai solusi dan komentar. Lahir pada bulan Juni 2015, KTO Matematika adalah
inisiatif dari para alumnus olimpiade matematika di Indonesia yang bertujuan
untuk memperkenalkan olimpiade matematika kepada semua orang Indone-
sia. Melalui diadakannya kontes dalam jaringan non-profit ini, diharapkan
siswa-siswi maupun guru-guru mendapatkan pengetahuan tentang olimpiade
matematika, mulai dari tipe-tipe soalnya sampai dengan cara penilaiannya.
Selain itu, peserta yang mengirimkan jawaban untuk soal uraian berkesem-
patan untuk mendapatkan umpan balik dari para pemenang olimpiade mate-
matika tingkat nasional (Olimpiade Sains Nasional) maupun internasional (In-
ternational Mathematical Olympiad). Saran yang bersifat konstruktif tersebut
diharapkan meningkatkan kemampuan peserta dalam menjawab soal uraian
dengan tepat menggunakan pernyataan yang logis, terperinci, dan jelas. Se-
lain itu, saran yang diberikan bisa dijadikan bahan pelajaran bagi para guru
untuk dapat membina peserta didiknya dalam membahasakan solusi mereka.
Buku KTOM 1 telah mendapatkan respons positif dari khalayak, terbukti
dengan cetakan pertamanya yang sudah terjual habis. Oleh karena itu, kami
menerbitkan cetakan kedua untuk buku ini. Isi dari buku cetakan kedua ini
pada umumnya sama dengan buku pendahulunya, hanya saja beberapa keke-
liruan yang terdapat pada buku cetakan pertama (sebagai contoh kesalahan
solusi, penulisan, pemilihan kata, juga frasa ambigu) sudah diperbaiki pada
cetakan kedua. Saya berharap bahwa perbaikan ini dapat meningkatkan kua-
litas buku ini dan semakin mendekatkan kami kepada tujuan dasar berdirinya
KTOM: memfasilitasi pembelajaran olimpiade matematika bagi semua anak
Indonesia.

vii
Buku KTO Matematika 1

Solusi dan komentar untuk soal-soal ini ditulis oleh para pemenang Inter-
national Mathematical Olympiad, kompetisi matematika paling bergengsi un-
tuk tingkat sekolah menengah. Kami berusaha untuk menyajikan solusi yang
sistematis dengan bahasa yang mudah dimengerti. Selain itu, kami juga me-
nyediakan komentar untuk setiap soal. Pemberian komentar bertujuan agar
pembaca tidak hanya mengerti cara mengerjakan soal, seperti yang didapat
dari melihat solusi, tetapi juga dapat mengerti dasar pemikiran yang meng-
arah kepada setiap solusi, kesalahan-kesalahan umum yang mungkin dilaku-
kan, dan mendapatkan pengetahuan tambahan dengan diberikannya beberapa
referensi buku maupun artikel. Dengan demikian, pembaca dapat memperluas
cakrawala pengetahuan dan menghindari kekeliruan-kekeliruan dalam mela-
kukan pendekatan saat menjawab soal.
Struktur penulisan buku ini adalah sebagai berikut:

1. Bagian pertama sampai ketujuh berisi soal-soal dan pembahasan (serta


komentar) KTO Matematika bulan Juni 2015 sampai dengan Desember
2015, berturut-turut. Bagian soal dipisahkan dengan bagian solusi agar
pembaca tidak ‘tergoda’ untuk melihat solusi. Hal ini sangatlah esensial
karena salah satu kunci sukses olimpiade matematika adalah mencoba,
mencoba, dan mencoba.

2. Bagian kedelapan berisi petunjuk untuk setiap soal. Petunjuk dikelom-


pokkan menjadi tiga kategori, yakni level 1000, level 2000, dan level
3000, berdasarkan tingkat kerumitan. Setiap soal pasti memiliki petun-
juk level 1000, sedangkan hanya soal-soal tertentu yang memiliki petun-
juk level 2000 ataupun petunjuk level 3000. Pemberian petunjuk bertu-
juan agar pembaca yang masih belum mempunyai ide untuk menjawab
soal setelah pemikiran yang cukup intens dapat mencari ‘pencerahan’
mengenai cara penyelesaian soal tersebut. Untuk mempermudah pem-
baca dalam mengutilisasi bagian petunjuk, sistem penulisan petunjuk
adalah sebagai berikut:

a) Setiap petunjuk merupakan bilangan empat-angka. Angka pertama


menunjukkan level dari petunjuk tersebut, sedangkan tiga angka
terakhir merupakan indeks dari petunjuk tersebut.
b) Pada bagian akhir dari setiap soal, diberikan kode yang menunjuk-
kan nomor dari petunjuk level 1000 yang bersesuaian dengan soal
tersebut. Jika pembaca ingin mengetahui petunjuk level 1000 dari

viii
Mengenai Buku Ini

sebuah soal, pembaca tinggal mencocokkan nomor petunjuk dengan


daftar petunjuk yang ada di bagian kedelapan.
c) Jika suatu soal memiliki petunjuk level 2000, nomor petunjuk level
2000 tersebut tertera pada bagian akhir dari petunjuk level 1000
untuk soal tersebut. Hal yang sama berlaku untuk soal yang memi-
liki petunjuk level 3000. Tujuan diciptakannya sistem penomoran
petunjuk ini adalah agar pembaca tidak dapat mengetahui tingkat
kesulitan dari suatu soal dengan melihat banyaknya petunjuk yang
tersedia.

Semoga buku ini dapat menjadi referensi yang berguna bagi pembaca untuk
memperdalam kemampuan mengerjakan soal-soal olimpiade matematika.

Jakarta, 11 Desember 2016

Kevin Christian Wibisono


Koordinator Buku KTO Matematika

ix
Daftar Isi

Kata Pengantar iii

Mengenai Buku Ini vii

Daftar Isi x

1 Kontes Juni 2015 1


1.1 Soal-Soal . . . . . . . . . . . . . . . . . . . . . . . . . . . . . . . . . 1
1.2 Pembahasan . . . . . . . . . . . . . . . . . . . . . . . . . . . . . . . 6

2 Kontes Juli 2015 29


2.1 Soal-Soal . . . . . . . . . . . . . . . . . . . . . . . . . . . . . . . . . 29
2.2 Pembahasan . . . . . . . . . . . . . . . . . . . . . . . . . . . . . . . 34

3 Kontes Agustus 2015 57


3.1 Soal-Soal . . . . . . . . . . . . . . . . . . . . . . . . . . . . . . . . . 57
3.2 Pembahasan . . . . . . . . . . . . . . . . . . . . . . . . . . . . . . . 61

4 Kontes September 2015 81


4.1 Soal-Soal . . . . . . . . . . . . . . . . . . . . . . . . . . . . . . . . . 81
4.2 Pembahasan . . . . . . . . . . . . . . . . . . . . . . . . . . . . . . . 85

5 Kontes Oktober 2015 103


5.1 Soal-Soal . . . . . . . . . . . . . . . . . . . . . . . . . . . . . . . . . 103
5.2 Pembahasan . . . . . . . . . . . . . . . . . . . . . . . . . . . . . . . 107

6 Kontes November 2015 125

x
Daftar Isi Daftar Isi

6.1 Soal-Soal . . . . . . . . . . . . . . . . . . . . . . . . . . . . . . . . . 125


6.2 Pembahasan . . . . . . . . . . . . . . . . . . . . . . . . . . . . . . . 128

7 Kontes Desember 2015 145


7.1 Soal-Soal . . . . . . . . . . . . . . . . . . . . . . . . . . . . . . . . . 145
7.2 Pembahasan . . . . . . . . . . . . . . . . . . . . . . . . . . . . . . . 149

8 Petunjuk 167
8.1 Petunjuk Level 1000 . . . . . . . . . . . . . . . . . . . . . . . . . . 167
8.2 Petunjuk Level 2000 . . . . . . . . . . . . . . . . . . . . . . . . . . 176
8.3 Petunjuk Level 3000 . . . . . . . . . . . . . . . . . . . . . . . . . . 181

xi
1 Kontes Juni 2015

1.1 Soal-Soal
Bagian A

Tuliskan jawaban akhir setiap soal di lembar jawaban. Setiap soal bernilai 1
angka.

1. (1001) Pada sebuah papan catur 8 × 8, setiap barisnya diberi label bi-
langan 1 sampai 8 dan setiap kolomnya diberi label huruf A sampai H.
Hitunglah banyaknya kotak pada papan catur yang berada pada baris
berlabel bilangan prima dan kolom berlabel huruf vokal.
n+3
2. (1008) Tentukan semua bilangan bulat n sehingga n−1 merupakan bi-
langan bulat.

3. (1015) Sebanyak 2015 koin dibagi menjadi 10 tumpukan. Tentukan ba-


nyak koin minimum pada tumpukan yang paling besar.

4. (1022) Diketahui bilangan bulat positif n merupakan hasil jumlah 2015


bilangan bulat berurutan. Tentukan nilai terkecil yang mungkin untuk
n.

5. (1029) Diberikan segi-8 beraturan ABCDEF H. Tentukan besar ∠BCH.

6. (1036) Diberikan sebuah segitiga dengan panjang sisi BC = 20, C A = 24,


dan AB = 12. Titik D pada segmen BC dengan BD = 5. Lingkaran luar
dari segitiga ABD memotong C A di E. Hitung panjang DE.

7. (1043) Diberikan dua buah dadu. Dadu pertama berbentuk kubus de-
ngan sisi berangka 1 hingga 6 dan peluang muncul setiap sisi adalah

1
Buku KTO Matematika 1

sama. Dadu kedua berbentuk limas dengan sisi berangka 1 hingga 4 dan
peluang muncul setiap sisi adalah sama. Kedua buah dadu dilemparkan
bersamaan. Berapakah peluang jumlah bilangan yang muncul genap?

8. (1050) Misalkan a > b > c > d bilangan real sehingga a + b + c + d = 1 dan


ab + bc + cd + da = −1. Tentukan nilai a − b + c − d.

9. (1057) Pada Sae Games 2015, setiap keping emas bernilai 4 poin, perak
bernilai 2 poin dan perunggu bernilai 1 poin. Negara Anggrek menda-
patkan poin 420. Total medali yang diraih negara Anggrek adalah 150.
Misalkan N menyatakan banyaknya medali emas yang mungkin dipero-
leh negara Anggrek jika diketahui informasi tersebut. Tentukan banyak-
nya nilai yang mungkin untuk N.

10. (1064) Diberikan jajar genjang ABCD yang luasnya 4 satuan. Misalkan
E titik tengah CD dan F titik tengah AB. Garis AE memotong garis DF
di K. Garis AC memotong garis DF dan BE berturut-turut di L dan M.
Hitunglah luas EK LM.

11. (1071) Misalkan x1 , x2 , . . . , x2015 menyatakan akar-akar dari persamaan

x2015 + 2x2014 + 3x2013 + · · · + 2015x + 2016 = 0.

Tentukan nilai
2015
X 2015
(xk )n .
X
n=0 k=1

12. [Soal ini tidak memiliki jawaban] Diberikan segilima siklis ABCDE
yang kelilingnya 36 satuan. Segitiga ABD merupakan segitiga sama sisi
dengan panjang sisi 10 satuan. Diketahui CE = 8 satuan. Jika F meru-
pakan titik potong AC dan BE, hitunglah panjang F A + FB.

13. (1085) Tentukan semua bilangan prima p sedemikian sehingga terdapat


bilangan bulat n yang memenuhi

n(n − 1)(n − 2)(n − 3) − 1678 ≤ p2 ≤ n(n − 1)(n − 2)(n − 3) − 1656.

14. (1092) Empat kakak beradik dengan nama Andi, Bayu, Candika, Da-
nang, Ellena, Fanny, Gina, Hani sedang bermain catur. Diketahui bah-
wa:

2
Soal-Soal Kontes Juni 2015

a) Ellena bermain dengan kakaknya Gina


b) Hani bermain dengan kakaknya Ellena
c) Fanny bermain dengan Andi
d) Bayu bermain dengan adiknya Candika
e) Candika bermain dengan adiknya Andi

Sebut lawan bermain Gina sebagai X . Tentukan saudara dari X .

15. (1099) Diketahui a, b ∈ R yang memenuhi


1 1
a+ = b − 4030 +
a + 2015 b − 2015
dan |a − b| > 5000. Tentukan nilai dari
ab
− a + b.
2015
(Notasi R menyatakan himpunan semua bilangan real.)

16. (1101) Diberikan sebuah segitiga dengan panjang sisi BC = 9, C A = 10,


dan AB = 11. Misalkan titik I adalah pusat lingkaran dalam dari segitiga
ABC. Misalkan pula I B adalah pusat lingkaran singgung luar segitiga
ABC terhadap B yang menyinggung segmen C A dan sinar garis AB dan
BC. Hitunglah panjang I I B .

17. (1103) Suatu tumpukan kartu terdiri dari 2016 kartu yang dinomori 1,
2, . . . , 2016. Tumpukan kartu ini dikocok sehingga urutan awal kartu-
kartu tidak diketahui. Sebuah permainan dimainkan yang melakukan
dua langkah berikut bergantian hingga semua kartu habis: (1) kartu pa-
ling atas dipindahkan ke paling bawah tumpukan, (2) kartu paling atas
dikeluarkan dari tumpukan. Setelah semua kartu keluar dari tumpukan,
ternyata urutan kartu yang dikeluarkan adalah 1, 2, . . . , 2016. Tentukan
kartu mana yang berada di atas tumpukan pada urutan awal kartu ter-
sebut.

18. (1105) Diberikan sebuah fungsi f : N → Q dengan f (1) = 23 dan


y ´ x
³ µ ¶
f (x + y) = 1 + f (x) + 1 + f (y) + x2 y + x y + x y2 .
x+1 y+1
Tentukan nilai f (20). (Notasi N menyatakan himpunan semua bilangan
bulat positif dan Q menyatakn himpunan semua bilangan rasional.)

3
Buku KTO Matematika 1

19. (1107) Misalkan (a, b, c) merupakan pasangan bilangan asli yang meme-
nuhi persamaan
a2 + 2b2 + 4c2 = k(a + b + c).
Carilah nilai k terkecil sehingga persamaan tersebut memiliki minimal
dua solusi.

20. (1109) Misalkan a i adalah koefisien dari x i pada penjabaran (1 + 3x)2015


untuk setiap bilangan bulat i dengan 0 ≤ i ≤ 2015. Carilah banyak nilai
k di mana 0 ≤ k ≤ 2014 sedemikian sehingga a k < a k+1 .

21. (1111) Misalkan x, y, dan z bilangan real yang memenuhi x + y + z = 0 dan


x2 + y2 + z2 = 6. Carilah nilai maksimum dari |(x − y)(y − z)(z − x)|.

4
Soal-Soal Kontes Juni 2015

Bagian B

Tuliskan jawaban beserta langkah pekerjaan Anda pada lembar jawaban.


Selain jawaban akhir, Anda perlu menuliskan argumentasi atau langkah-
langkah untuk memperoleh jawaban akhir tersebut. Gunakan halaman yang
berbeda untuk setiap soal yang berbeda. Setiap soal bernilai 7 angka.

1. (1113) Diberikan segiempat PQRS sedemikian sehingga:

(i) segitiga PSR merupakan segitiga siku-siku dengan ∠PSR = 90°,


(ii) segitiga PRQ merupakan segitiga siku-siku dengan ∠PRQ = 90°.

Diketahui |PS | = 12, |SR | = 9, dan |PQ | = 25.

a) Tentukan luas segiempat PQRS.


b) Tentukan panjang diagonal QS.

2. (1120) Barisan {a n } dan { b n } diberikan dengan definisi a 1 = 625 dan


a n = 625a n−1 serta b 1 = 5 dan b n = 5b n−1 untuk setiap bilangan asli n > 1.
Tentukan bilangan asli k terkecil sehingga b k > a 625 .

3. (1127) Pada papan catur berukuran 4 × 4 akan diletakkan sebanyak 7 bu-


ah kuda identik. Tentukan banyak cara meletakkannya sehingga tidak
ada dua buah kuda yang saling menyerang.
(Keterangan terkait bagaimana kuda menyerang dapat dilihat pada ar-
tikel berjudul “Kuda catur” di Wikipedia bahasa Indonesia.)

5
Buku KTO Matematika 1

1.2 Pembahasan
Bagian A

1. Jawab: 8.
Perhatikan bahwa kita dapat menyatakan tiap kotak dengan pasangan
(a, b), di mana a adalah label baris dan b adalah label kolom yang memu-
at kotak tersebut. Sebagai contoh, kotak (3, A) merupakan kotak yang
terletak pada baris berlabel 3 dan kolom berlabel A. Kita akan menghi-
tung banyaknya kotak dengan label baris bilangan prima dan label ko-
lom huruf vokal. Himpunan semua bilangan prima dari {1, 2, . . . , 8} ada-
lah {2, 3, 5, 7}, dan himpunan semua huruf vokal dari { A, B, . . . , H } adalah
{ A, E }. Dari kedua fakta tersebut, kita bisa mendaftar semua kotak yang
memenuhi syarat, yaitu (2, A), (2, E), (3, A), (3, E), (5, A), (5, E), (7, A), dan
(7, E), sehingga terdapat 8 kotak yang demikian.
Secara alternatif, kita bisa mengutilisasi prinsip perkalian seperti beri-
kut ini: “Karena pelabelan baris dan kolom adalah dua kasus yang tidak
saling memengaruhi, banyaknya kotak yang memenuhi kedua syarat pa-
da soal (label baris bilangan prima dan label kolom huruf vokal) adalah
hasil kali banyaknya baris yang berlabel bilangan prima dan banyaknya
kolom yang berlabel huruf vokal, yang sama dengan 4 × 2 = 8.”
Komentar:

a) Berdasarkan fakta bahwa 89% peserta mampu menjawab soal ini


dengan benar, pantas dikatakan bahwa soal ini adalah salah satu
soal termudah dari semua soal pada kontes bulan Juni.
b) Prinsip perkalian (atau multiplication principle) merupakan satu
dari dua konsep counting (hal menghitung di bidang kombinatori-
ka) yang fundamental; satu lagi adalah prinsip penjumlahan (atau
addition principle). Soal ini adalah contoh dasar dari penggunaan
prinsip perkalian dalam menyelesaikan permasalahan kombinato-
rika. Namun, mengingat angka-angka yang digunakan pada soal
cenderung tidak besar, penghitungan tanpa mempergunakan prin-
sip perkalian bukanlah masalah yang besar.
c) Untuk mendapatkan pengetahuan lebih lanjut mengenai prinsip
penjumlahan, prinsip perkalian, dan teknik-teknik counting yang
lain, pembaca dapat merujuk pada buku Principles and Techniqu-

6
Pembahasan Kontes Juni 2015

es in Combinatorics karangan Chen Chuan-Chong dan Koh Khee-


Meng.

2. Jawab: -3, -1, 0, 2, 3, dan 5.


Pandang bahwa nn+ 3 4 n+3
−1 = 1 + n−1 . Kita peroleh n−1 adalah bilangan bulat
4
jika dan hanya jika n− 1 juga adalah bilangan bulat, yang ekuivalen de-
ngan n − 1 adalah faktor dari 4. Perhatikan bahwa semua faktor dari 4
adalah -4, -2, -1, 1, 2, dan 4. Sebagai konsekuensi, semua nilai n yang
memenuhi ialah -3, -1, 0, 2, 3, dan 5.
Komentar:

a) Menurut statistik, 68% peserta dapat menjawab soal nomor 2 de-


ngan benar. Akibatnya, soal ini dapat dikategorikan cukup mudah.
b) Cukup banyak peserta membuat kesalahan yang tidak perlu pada
soal ini. Beberapa kesalahan yang umum ditemukan adalah meng-
hitung banyaknya bilangan bulat n (sehingga menghasilkan jawab-
an 6), hanya meninjau nilai n yang positif (ataupun taknegatif), dan
melupakan fakta bahwa −4 dan 4 adalah faktor dari 4. Pembaca di-
harapkan menghindari tipe-tipe kesalahan demikian dengan mem-
baca soal hingga tuntas sebelum mengerjakannya.

3. Jawab: 202.
Pertama, akan ditunjukkan bahwa banyaknya koin pada tumpukan yang
paling besar pasti lebih dari 201. Asumsikan bahwa banyaknya koin
pada tumpukan yang paling besar kurang dari atau sama dengan 201.
Akibatnya, banyaknya koin secara keseluruhan haruslah kurang dari
atau sama dengan 10 × 201 = 2010, yang adalah tidak mungkin kare-
na 2015 > 2010. Sebagai akibat, banyaknya koin pada tumpukan yang
paling besar pasti lebih dari 201. Selanjutnya, akan didemonstrasikan
bahwa kondisi di mana banyaknya koin pada tumpukan yang paling be-
sar adalah 202 mungkin untuk dicapai. Untuk menunjukkannya, cukup
pandang pembagian 2015 koin menjadi 5 tumpukan yang berisi 201 koin
dan 5 tumpukan yang berisi 202 koin. Penggabungan kedua fakta ter-
sebut memberikan kesimpulan bahwa banyaknya koin minimum pada
tumpukan yang paling besar adalah 202.

7
Buku KTO Matematika 1

Komentar:

a) Statistik menunjukkan bahwa persentase peserta yang dapat men-


jawab soal ini dengan benar adalah 61%.
b) Observasi intuitif yang menjadi kunci untuk menyelesaikan soal ini
adalah bahwa kita dapat memperkecil banyaknya koin pada tum-
pukan yang paling besar dengan cara memperkecil selisih banyak-
nya koin antartumpukan. Pengamatan ini dapat mengarahkan ki-
ta kepada kasus di mana setiap tumpukan mengandung koin-koin
dengan banyak yang sama, yaitu 201,5 (yang kenyataannya tidak
mungkin). Angka tersebut memberikan sugesti bahwa banyaknya
koin minimum yang mungkin pada tumpukan paling besar adalah
202.
c) Setelah menebak bahwa jawaban untuk soal ini adalah 202, lang-
kah selanjutnya adalah melakukan validasi. Salah satu metode
yang umum untuk kasus seperti ini adalah membuktikan bahwa ni-
lai tersebut (dalam hal ini 202) mungkin untuk dicapai, dan semua
nilai yang lebih kecil dari nilai tersebut (202) tidak mungkin.

4. Jawab: 2015.
Misalkan median (nilai tengah) dari ke-2015 bilangan tersebut adalah a.
Akibatnya,

n = (a − 1007) + (a − 1006) + · · · + (a − 1) + a + · · · + (a + 1006) + (a + 1007)


= 2015a

Karena n > 0, haruslah a > 0, atau a ≥ 1. Nilai terkecil yang mungkin


untuk n adalah 2015 × 1 = 2015.
Komentar:

a) Melihat bahwa 74% peserta mendapatkan jawaban yang benar un-


tuk soal ini, soal ini bisa dikatakan mudah. Metode pengerjaan yang
digunakan pada soal ini juga sangat standar.
b) Untuk memperoleh 2015a pada penjumlahan 2015 suku di atas, ki-
ta bisa memanfaatkan rumus deret aritmetika maupun teknik Ga-
uss (menjumlahkan suku pertama dan suku terakhir, suku kedua
dan suku kedua terakhir, dan seterusnya). Kenyataanya, rumus de-
ret aritmetika bisa dibuktikan dengan penggunaan teknik Gauss.

8
Pembahasan Kontes Juni 2015

5. Jawab: 45°.

F E

G D

H C

A B

Cukup pandang bahwa ∠BCH = ∠BCD − ∠ HCD = 135° − 90° = 45°.

Komentar:

a) Soal ini dapat diselesaikan oleh 85% partisipan, sehingga dapat di-
golongkan sebagai soal mudah. Tidak ada hal yang perlu dijabarkan
lebih lanjut mengenai soal ini.

b) Pembaca dapat melakukan observasi mandiri dalam menghitung


besar sudut-sudut lain pada poligon bersisi lain.

6. Jawab: 7,5.

D
B

9
Buku KTO Matematika 1

Pandang bahwa 4CDE ∼ 4C AB (dengan orientasi demikian) karena


∠ECD = ∠BC A dan ∠CED = ∠CBA, mengingat ABDE adalah segiem-
pat tali busur. Akibatnya, kita peroleh relasi DE : AB = CD : C A. Meng-
gunakan fakta dari soal bahwa AB = 12, CD = BC − BD = 20 − 5 = 15, dan
C A = 24, bisa dilihat bahwa DE = 7, 5. Kita selesai.
Komentar:

a) Soal tersebut dapat dikategorikan sebagai soal rutin dengan mem-


pertimbangkan eksistensi soal bertipe demikian di kompetisi mate-
matika. Ide penyelesaiannya pun juga tergolong standar, mengingat
hanya menggunakan hubungan kesebangunan antardua segitiga.
Kendati demikian, hanya 34% peserta mampu mengerjakan soal ini
dengan tepat.
b) Relasi kesebangunan antarsegitiga, pada kebanyakan kasus, ada-
lah observasi yang krusial dalam mengerjakan soal geometri di
olimpiade matematika. Pembaca diharapkan untuk dapat memi-
lah segitiga-segitiga yang saling sebangun dalam sebuah gambar
geometri. Salah satu cara yang bisa dimanfaatkan untuk memper-
mudah pencarian tersebut adalah dengan menentukan besar sudut
pada sudut-sudut yang tampak di gambar tersebut. Metode terse-
but dikenal secara umum dengan sebutan angle chasing. Pembaca
diundang untuk membuktikan teorema Miquel dengan mengguna-
kan teknik angle chasing.

1
7. Jawab: 2.

Perhatikan bahwa jumlah bilangan yang muncul pada kedua dadu ada-
lah genap jika dan hanya jika paritas kedua bilangan yang muncul sama
(paritas adalah keadaan genap atau ganjilnya suatu bilangan). Dengan
demikian, dapat ditarik kesimpulan bahwa terdapat 6 × 4 = 24 kemung-
kinan keluarnya angka-angka pada pelemparan kedua dadu tersebut.
Selanjutnya, karena terdapat 3 bilangan ganjil pada dadu pertama dan
2 bilangan ganjil pada dadu kedua, terdapat 3 × 2 = 6 kemungkinan kelu-
arnya dua angka ganjil pada pelemparan kedua dadu tersebut. Dengan
cara yang sama, terdapat 6 kemungkinan keluarnya dua angka genap.
Dengan prinsip penjumlahan, diperoleh 6 + 6 = 12 kemungkinan pelem-
paran kedua dadu yang menghasilkan jumlah angka genap. Akibatnya,
peluang terjadinya keadaan tersebut adalah 12 1
24 = 2 .

10
Pembahasan Kontes Juni 2015

Komentar:

a) Mengamati bahwa 94% peserta mampu mendapatkan jawaban yang


benar, sudah jelas bahwa soal ini dapat dikategorikan sebagai soal
mudah.
b) Teknik pengerjaan soal adalah dengan mengutilisasi dua prinsip
fundamental dari counting (hal menghitung di bidang kombinatori-
ka), yakni prinsip penjumlahan dan prinsip perkalian, seperti yang
dilakukan pada soal nomor 1. Jika pembaca tidak mendapatkan ja-
waban yang sesuai pada soal ini, pembaca disarankan untuk belajar
lebih dalam mengenai kombinatorika. Salah satu buku yang baik
untuk dibaca bisa dilihat pada kolom komentar pada soal nomor 1.
p
8. Jawab: 5.
Misalkan a + c = x. Akibatnya, b + d = 1 − x. Perhatikan bahwa −1 =
ab + bc + cd + da = (b + d)(a + c) = x(1 − x), sehingga diperoleh persamaan
p
x2 − x − 1 = 0. Nilai x yang memenuhi persamaan tersebut adalah 1+2 5
p
1− 5
atau 2 . Namun, kita punya x = a + c > b + d = 1 − x, yang berakibat
p
kepada x > 21 . Konsekuensi dari pernyataan tersebut adalah x p = 1+2 5 ,
sehingga nilai a − b + c − d = (a + c) − (b + d) = x − (1 − x) = 2x − 1 = 5.
Komentar:

a) Soal ini dapat diselesaikan oleh 58% partisipan, sehingga dapat di-
golongkan sebagai soal cukup mudah.
b) Kunci dari penyelesaian soal ini adalah melihat kecenderungan a + c
dan b + d untuk mengelompok dan membuat pemisalan. Teknik
ini banyak dijumpai dalam penyelesaian soal-soal olimpiade mate-
matika. Pembaca diharapkan untuk dapat bersikap cekatan dalam
membuat pemisalan selagi diperlukan.
c) Banyak di antara peserta yang lupa mengutilisasi fakta bahwa
p a>
bp> c > d. Akibatnya, jawaban mereka berkisar antara − 5 atau
± 5. Pembaca sebaiknya menghindari kesalahan yang fatal seperti
ini dengan bersikap curiga bila ada fakta dalam soal yang belum
dimanfaatkan dalam solusi yang ditulis pembaca.

11
Buku KTO Matematika 1

9. Jawab: 31.

Misalkan banyaknya medali emas, perak, dan perunggu yang diraih ne-
gara Anggrek berturut-turut adalah x, y, dan z. Menggunakan informasi
pada soal, diperoleh dua persamaan, yakni 4x + 2y + z = 420 (sebut per-
samaan 1) dan x + y + z = 150 (sebut persamaan 2). Kurangkan 4 kali
persamaan 2 dengan persamaan 1, kita dapatkan 2y + 3z = 180. Per-
hatikan bahwa z harus merupakan bilangan genap, sehingga bisa kita
misalkan z = 2z0 . Akibatnya, y = 12 × (180 − 6z0 ) = 90 − 3z0 . Substitusi
ke persamaan 1, diperoleh x = 60 + z0 . Perhatikan bahwa x, y, dan z ha-
rus merupakan bilangan taknegatif. Akibatnya, didapat pertidaksamaan
0 ≤ z0 ≤ 30. Karena setiap bilangan bulat z0 yang memenuhi syarat ter-
sebut juga memenuhi soal, bisa disimpulkan bahwa banyaknya medali
emas yang mungkin didapat oleh negara Anggrek adalah semua bilang-
an bulat t yang memenuhi 60 ≤ t ≤ 90. Dengan kata lain, N = 31.

Komentar:

a) Soal ini dapat dikatakan cukup sulit berdasarkan data bahwa ha-
nya 39% peserta mampu menjawab soal ini dengan tepat.
b) Dari analisis jawaban-jawaban yang salah, terlihat bahwa jawaban
yang cukup banyak ditulis oleh para kontestan adalah 60 dan 90.
Dari jawaban-jawaban ini, bisa diobservasi bahwa sebagian peserta
gagal menginterpretasi soal dengan benar; mereka hanya menulisk-
an nilai minimum dan maksimum yang mungkin untuk banyaknya
medali emas yang diperoleh negara Anggrek.

1
10. Jawab: 2.

A D

L
F E

B C

12
Pembahasan Kontes Juni 2015

Misalkan [X ] menyatakan luas daerah X . Pandang bahwa [AEC] =


1 1 1
2 × [ADC] = 2 × 2 × [ABCD] = 1. Pandang pula bahwa 4 AMB seba-
AM AB
ngun dengan 4CME, sehingga MC = EC = 2. Akibatnya, AM 2
AC = 3 , dan
2 2
[AEM] = 3 × [AEC] = 3 . Selanjutnya, karena DF ∥ BE, kita punya K L ∥
AK AF
EM, sehingga 4 AK L sebangun dengan 4 AEM. Karena K E = DE = 1,
AK
kita punya AE = 2 , yang berakibat kepada [AK L] = 4 ×[AEM] = 4 × 3 = 16 .
1 1 1 2

Dari sana kita dapatkan [EK LM] = 23 − 16 = 21 .


Komentar:

a) Berdasarkan statistik bahwa 48% peserta mampu menjawab soal


ini dengan tepat, soal ini tergolong memiliki tingkat kesulitan se-
dang.
b) Observasi yang krusial dalam penyelesaian soal ini adalah relasi
kesebangunan antarsegitiga dan perbandingan luas antarsegitiga
dengan tinggi yang sama. Teknik pengerjaan soal yang demikian
cukup rutin digunakan pada soal-soal yang berkaitan dengan lu-
as daerah. Sebagai latihan tambahan, pembaca disarankan untuk
membuktikan teorema Varignon secara mandiri.
c) Beberapa ketidaktelitian yang ditemukan pada penyelesaian soal
ini antara lain melupakan fakta bahwa luas jajargenjang ABCD
adalah 4. Beberapa kontestan telah mendapatkan bahwa perban-
dingan luas segiempat EK LM dan ABCD adalah 81 . Namun, me-
reka menganggap bahwa luas segiempat ABCD adalah 1, sehingga
berkesimpulan bahwa luas segiempat EK LM adalah 18 . Ini adalah
kesalahan yang patut disesalkan.

11. Jawab: -2015.


Misalkan f (x) = x2015 + 2x2014 + 3x2013 + · · · + 2015x + 2016. Akibatnya,
2015
X 2015 2015
X 2015
(xk )n = (xk )n
X X
n=0 k=1 k=1 n=0
2015
X
µ
f (xk ) − 2016

= f (xk ) −
k=1 xk
2015
X 2016
=
k=1 xk
−2015
= 2016 ×
2016

13
Buku KTO Matematika 1

= −2015.

14
Pembahasan Kontes Juni 2015

Komentar:

a) Memandang bahwa 21% peserta mampu menjawab soal ini dengan


benar, soal ini jelas termasuk soal sulit.
b) Bagian yang cukup sulit dari penyelesaian soal ini adalah bagian
mentransformasi f (x) menjadi fungsi g(x) = x2015 + x2014 + · · · + x + 1.
Diperlukan keterbiasaan dan latihan yang intens untuk menda-
patkan insting tersebut. Selain itu, semuanya mengikuti.
c) Bagi pembaca yang tidak bisa membuat deduksi bahwa penjumlah-
2015
an dari kebalikan dari xk adalah − 2016 , silakan merujuk kepada
artikel Wikipedia yang berjudul Vieta’s formulas.

12. Jawab: soal salah.


Misalkan lingkaran luar segilima ABCDE berpusat di O dan memiliki
jari-jari r. Karena ABD merupakan segitiga sama sisi dengan panjang
sisi 10, kita peroleh jari-jari lingkaran luar segitiga ABD adalah p10 .
(3)
Dengan demikian, r = p10 .
(3)
Misalkan ∠BOC = 2K, ∠COD = 2L, ∠DOE = 2M, ∠EO A = 2N. Dipero-
leh K + L = M + N = 60°.
Perhatikan bahwa
K +L K −L
µ ¶ µ ¶
BC + CD = 2r(sin K + sin L) = 4r sin cos ≤ 2r
2 2
dan
M+N M−N
µ ¶ µ ¶
DE + E A = 2r(sin M + sin N) = 4r sin cos ≤ 2r.
2 2
Dengan demikian, kita peroleh
s
40 1600
AB + BC + CD + DE + E A ≤ 10 + 4r = 10 + p = 10 + < 34 < 36,
3 3

suatu kontradiksi. Jadi, tidak ada segilima ABCDE yang memenuhi


kondisi yang diberikan.
Komentar:

a) Soal ini dianulir karena tidak ada segilima yang memenuhi kondisi
yang diberikan.

15
Buku KTO Matematika 1

b) Kami ingin mengucapkan terima kasih kepada peserta KTO Juni


2016 atas nama Andrew Jonathan yang telah memberikan penje-
lasan terkait kesalahan pada soal ini.

13. Jawab: 2 dan 3.


Perhatikan bahwa untuk setiap bilangan bulat n, berlaku 24| n(n − 1)(n −
2)(n − 3). Oleh karena itu, kita punya 24k + 2 ≤ p2 ≤ 24k + 24 untuk suatu
bilangan bulat k. Padahal, jika p > 3, pernyataan p2 ≡ 1 (mod 24) pas-
ti berlaku, sehingga nilai p yang mungkin hanyalah 2 atau 3. Setelah
diperiksa, ternyata n = 8 memenuhi soal untuk kedua nilai p tersebut.
Sebagai akibat, semua nilai prima p yang memenuhi ialah 2 dan 3.
Komentar:

a) Sebanyak 44% dari semua partisipan mampu mengerjakan soal ini


dengan benar. Oleh karena itu, soal ini memiliki tingkat kesulitan
sedang.
b) Dua observasi kunci dari soal ini adalah fakta bahwa hasil kali em-
pat bilangan bulat berurutan habis dibagi 24 dan p2 ≡ 1 (mod 24)
untuk setiap bilangan prima p > 3. Pengetahuan dan latihan soal
mengenai teori bilangan dapat mendukung pembaca untuk mene-
mukan kebenaran ‘yang tersembunyi’ dalam soal-soal seperti ini.
c) Pembaca direkomendasikan untuk membuktikan kedua pernyata-
an tersebut. Pembaca juga disarankan untuk menunjukkan gene-
ralisasi dari pernyataan pertama, yakni hasil kali n bilangan bulat
berurutan habis dibagi oleh n!.
d) Pembaca dapat merujuk kepada bagian Number Theory dari buku
Problem Solving Strategies karya Arthur Engel untuk mempelajari
dasar-dasar teori bilangan.

14. Jawab: Danang.


Perhatikan kalimat (c), (d), dan (e). Anggap M adalah adik Andi; kita
akan mencari tahu siapa M. Karena Fanny bermain dengan Andi, kita
dapatkan bahwa M bermain dengan Candika dan Bayu bermain dengan
adiknya Candika. Perhatikan bahwa tidaklah mungkin bahwa M adalah
Fanny, Andi, Candika, maupun Bayu. Akibatnya, kemungkinan yang
tersisa untuk M adalah Hani, Ellena, Gina, dan Danang.

16
Pembahasan Kontes Juni 2015

Bila M itu Ellena, perhatikan bahwa Hani bermain dengan kakaknya


Ellena, yakni Andi. Padahal, Fanny bermain dengan Andi, sehingga di-
peroleh sebuah kontradiksi. Bila M itu Gina, perhatikan bahwa Ellena
bermain dengan kakaknya Gina, yakni Andi. Hal ini tidak mungkin ka-
rena Fanny-lah yang bermain dengan Andi.

Bila M itu Danang, kita tinjau adik Candika, sebut saja N. Pandang bah-
wa N tidak mungkin Ellena ataupun Gina sebab lawan bermain Can-
dika yang merupakan kakak N adalah Danang. Padahal, bila N adalah
Ellena atau Gina, lawan bermain Candika adalah Hani atau Ellena, kon-
tradiksi. Bila N itu Hani, perhatikan bahwa Hani bermain dengan Bayu.
Pandang bahwa sudah ada tiga permainan yang didapatkan (Fanny - An-
di, Danang - Candika, Hani - Bayu). Akibatnya, Ellena dan Gina pasti
bermain. Namun, hal ini tidak mungkin sebab Ellena bermain dengan
kakaknya Gina, dan bukan Gina.

Akibatnya, M adalah Hani. Bisa diperiksa bahwa Ellena adalah adik


Candika sekaligus lawan bermain Bayu, dan Gina adalah adik Bayu se-
kaligus lawan bermain Danang. Maka, saudara X adalah Danang.

Komentar:

a) Soal awal pada kompetisi meminta kita untuk mencari kakak, bu-
kan saudara, dari X. Namun, karena Danang bisa merupakan ka-
kak ataupun adik Fanny, soal ini dianulir. Jika pertanyaannya ada-
lah “siapa saudara X?”, barulah soal ini memiliki jawaban, yakni
Danang.

b) Soal ini bisa dijawab dengan lebih mudah dengan adanya visualisasi
seperti ini:

Hani ?

Ellena ?

Gina

17
Buku KTO Matematika 1

Fanny Andi

M Candika

N Bayu

Di sini, garis horizontal menyatakan permainan dan garis vertikal


menandakan hubungan kakak-adik.
1
15. Jawab: 2015 + 2015 .
Perhatikan bahwa
1 1
a+ = b − 4030 +
a + 2015 b − 2015
1 1
⇐⇒ − = b − a − 4030
a + 2015 b − 2015
b − a − 4030
⇐⇒ = b − a − 4030.
(a + 2015)(b − 2015)

Karena |a − b| > 5000, maka b − a 6= 4030, sehingga (a + 2015)(b − 2015) = 1.


ab 1
Akibatnya, 2015 − a + b = 2015 + 2015 .
Komentar:

a) Karena 44% dari semua partisipan dapat mengerjakan soal ini de-
ngan tepat, soal ini dapat dikatakan memiliki tingkat kesulitan se-
dang.
b) Teknik pengerjaan soal ini bisa digolongkan sebagai cukup standar,
yaitu dengan mengelompokkan bentuk a + 2015 dan b − 2015, yang
terdapat pada informasi soal dan bentuk yang ingin kita tentukan
nilainya. Pada dasarnya, soal ini hanyalah manipulasi bentuk alja-
bar.

18
Pembahasan Kontes Juni 2015

p
16. Jawab: 2 33.

A
IB

X
I

B
C

Karena BI dan BI B adalah garis bagi dari ∠ ABC, ketiga titik B, I,


dan I B kolinear. Misalkan X adalah titik singgung dari lingkaran da-
lam 4 ABC pada sisi AB, dan Y adalah titik singgung dari lingka-
ran singgung luar 4 ABC pada sisi AB. Perhatikan bahwa 4BI X ∼
4BI B Y . Dengan menggunakan rumus jari-jari lingkaran p
dalam dan
L 30 2
p
jari-jari lingkaran singgung luar, diperoleh I X = s = 15 = 2 2 dan
p p
I B Y = s−LAC = 305 2 = 6 2. Akibatnya, BI BI
B
= IIBXY = 13 . Selanjutnya,
menggunakan faktaq bahwa BX = s − AC = 5 dan teorema Pythagoras,
p 2 p
kita dapatkan BI = 52p + (2 2) = 33. Dari sana kita peroleh bahwa
I I B = BI B − BI = 2BI = 2 33. Kita selesai.
Komentar:

a) Secara mengejutkan, hanya 18% dari total peserta mampu menja-


wab soal ini dengan benar. Dengan demikian, bisa dikatakan bahwa
soal ini tergolong sulit, walaupun cara pengerjaannya sebenarnya
cukup rutin.
b) Dengan argumen yang sama, panjang segmen I I A dan I I C juga da-
pat ditentukan. Pembaca dipersilakan untuk mencobanya secara
mandiri.

19
Buku KTO Matematika 1

17. Jawab: 1985.


Perhatikan bahwa soal ini sebenarnya sama saja dengan mengganti pe-
nomoran (renumber) ke-2016 kartu menjadi demikian: kartu ke-i dari
atas diberi nomor i, dan kita harus mencari pada pengambilan ke berapa
terambil kartu bernomor 1. Definisikan sebuah set pengambilan seba-
gai sekumpulan pengambilan kartu sebelum nomor kartu yang terambil
menjadi lebih kecil. Sebagai contoh, jika terdapat 8 kartu 1, 2, . . . , 8, ter-
dapat empat set kartu pada pengambilan bergiliran sesuai prosedur soal,
yakni {2, 4, 6, 8}, {3, 7}, {5}, dan {1}. Perhatikan bahwa kartu bernomor 1
(pada penomoran baru) terjadi tepat setelah pengambilan kartu dengan
angka terbesar pada set pertama yang banyak anggotanya ganjil (silak-
an verifikasi sendiri). Dari sini, kita bisa melihat bahwa kartu bernomor
1 terambil pada urutan ke 1008 + 504 + 252 + 126 + 63 + 31 + 1 = 1985.
Komentar:

a) 29% dari keseluruhan peserta mampu mengerjakan soal ini dengan


tepat, sehingga kita dapat menggolongkan soal ini sebagai soal su-
lit.
b) Jika dilihat, pengerjaan soal ini tidak membutuhkan pengetahuan
akan teorema tertentu. Namun, letak kesulitan soal ini adalah pa-
da ‘pemerolehan’ ide. Pembaca sangat direkomendasikan untuk me-
mverifikasi bagian pada solusi yang disajikan.

18. Jawab: 4305.


Perhatikan bahwa
y ´ x
³ µ ¶
f (x + y) = 1 + f (x) + 1 + f (y) + x2 y + x y + x y2
x+1 y+1
f (x + y) f (x) f (y)
⇐⇒ = + + x y.
x+ y+1 x+1 y+1
f ( x)
Misalkan g(x) = x+1 untuk setiap x ∈ N. Akibatnya, g(x + y) = g(x) + g(y) +
x y, sehingga

g(20) = g(19) + g(1) + 19


= g(18) + 2g(1) + 19 + 18
..
.

20
Pembahasan Kontes Juni 2015

= 20g(1) + (19 + 18 + · · · + 1)
= 205.

Akibatnya, f (20) = 21g(20) = 4305.


Komentar:

a) Melihat bahwa 56% peserta dapat mengerjakan soal ini dengan be-
nar, soal ini tergolong cukup mudah.
b) Pengamatan yang penting dalam menjawab soal ini adalah pemisal-
f ( x)
an fungsi g(x) = x+1 . Observasi tersebut didasari oleh fakta bahwa
bentuk x + y + 1 cenderung muncul secara berkelompok dalam per-
samaan fungsi yang disajikan pada soal.
31
19. Jawab: 7 .
Perhatikan bahwa nilai k = 317 memenuhi soal karena terdapat dua triple
yang mungkin, yaitu (5, 1, 1) dan (3, 3, 1). Selanjutnya, akan ditunjuk-
kan bahwa k = 31
7 adalah yang terkecil. Menurut ketaksamaan Cauchy-
Schwarz,
1 1
µ ¶
(a2 + 2b2 + 4c2 ) 1 + + ≥ (a + b + c)2 ,
2 4
yang ekuivalen dengan

a2 + 2b2 + 4c2 4
k= ≥ (a + b + c).
a+b+c 7

Misalkan terdapat nilai k lain yang lebih kecil daripada 31 7 dan juga
memenuhi soal. Akibatnya, untuk setiap triple (a, b, c) yang mungkin,
31 4
7 > 7 (a + b + c), atau a + b + c ≤ 7. Verifikasi langsung menunjukkan bah-
wa untuk setiap triple (a, b, c) yang memenuhi a + b + c ≤ 7, semua nilai
k yang dihasilkan, selain k = 31 7 adalah unik (hal memverifikasi diserah-
kan kepada pembaca). Akibatnya, nilai terkecil dari k yang membuat
persamaan pada soal memiliki minimal dua solusi adalah 31 7 .
Komentar:

a) Soal ini sangat layak untuk dikatakan sebagai soal isian singkat
tersulit pada kontes bulan Juni, mengingat hanya satu peserta yang
mampu menjawab soal ini dengan benar.

21
Buku KTO Matematika 1

b) Bagian yang menjadi kunci dari penyelesaian soal ini adalah me-
nebak bahwa nilai k = 31
7 adalah yang terkecil. Cara yang dilaku-
kan untuk mendapatkan nilai tersebut adalah dengan melakukan
penebakan untuk nilai-nilai (a, b, c) yang relatif kecil. Setelah itu,
barulah dilakukan pembuktian bahwa nilai k tersebut adalah yang
terkecil.
c) Pembaca yang belum familiar dengan ketaksamaan Cauchy-
Schwarz dapat merujuk kepada artikel artofproblemsolving.com
yang berjudul Cauchy-Schwarz Inequality.

20. Jawab: 1511.


Lihat bahwa a i = 2015
¡ ¢ i
i (3) untuk setiap bilangan bulat i dengan 0 ≤ i ≤
2015. Oleh karena itu, a i+1 = i+1 (3) , sehingga aai+i 1 = 6045
¡2015¢ i+1 −3 i
i +1 (pen-
a i+1
jabaran diserahkan kepada pembaca). Perhatikan bahwa a i > 1, atau
a k+1 > a k jika dan hanya jika 6045 − 3k > k + 1, atau k < 1511. Akibatnya,
banyaknya nilai k yang memenuhi adalah 1511.
Komentar:

a) Ide penyelesaian dari soal ini sebenarnya cukup standar, yakni


dengan menggunakan penjabaran Binomial Newton dan memban-
dingkan nilai a i dan a i+1 untuk setiap i. Kenyataan bahwa hanya
32% peserta kontes yang mendapatkan jawaban benar menunjuk-
kan bahwa para kontestan cenderung kurang familiar dengan Bi-
nomial Newton.
b) Pembaca dapat membuktikan identitas-identitas kombinasi dengan
Binomial Newton agar lebih terbiasa dalam menghadapi soal-soal
yang melibatkan penjabaran tersebut.
p
21. Jawab: 6 3.
Karena bentuk |(x − y)(y − z)(z − x)| memiliki sifat simetrik (penukaran
antarnilai x, y, dan z tidak mengubah hasil dari ekspresi tersebut), dapat
dimisalkan, tanpa mengurangi keumuman, bahwa x ≥ y ≥ z. Pandang
bahwa

(x − y)2 + (y − z)2 + (z − x)2 = 2(x2 + y2 + z2 ) − 2(x y + yz + zx)


= −(x + y + z)2 + 3(x2 + y2 + z2 )
= 18.

22
Pembahasan Kontes Juni 2015

Selanjutnya, menurut ketaksamaan Rataan Kuadratik-Rataan Aritme-


tik (QM-AM), diperoleh

18 = (x − y)2 + (y − z)2 + (z − x)2


(x − y + y − z)2
≥ + (x − z)2
2
3
= (x − z)2
2
p
⇒ 2 3 ≥ x − z.

Namun, sekali lagi menurut AM-GM, kita peroleh

|(x − y)(y − z)(z − x)| = (x − y)(y − z)(z − x)


³ x − y + y − z ´2
≤ (x − z)
2
(x − z)3 p
= ≤ 6 3.
4
p p p
Karena triple (x, y, z) = ( 3, 0, − 3) memenuhi |(x − y)(y − z)(z − x)| = 6 3,
bisa kita
p simpulkan bahwa nilai maksimum dari ekspresi pada soal ada-
lah 6 3.
Komentar:

a) Soal ini termasuk soal sulit mengingat hanya 27 % peserta yang


menjawab soal ini dengan benar. Salah satu hal yang berkontribu-
si kepada kesulitan soal ini adalah nilai kesamaannya yang tidak
mudah terlihat.
b) Latihan soal yang intensif adalah satu-satunya kunci untuk mengu-
asai materi ketaksamaan, mengingat natur soal ketaksamaan yang
biasanya sulit ditebak. Pembaca dapat merujuk kepada buku Se-
crets in Inequalities Volume 1 - Basic Inequalities karya Pham Kim
Hung untuk memperluas cakrawala dan menambah pengalaman
pembaca dalam mengerjakan soal-soal ketaksamaan.

23
Buku KTO Matematika 1

Bagian B
p
1. Jawab: 204; 769.

S R

Q
P

Perhatikan bahwa dengan teorema Pythagoras, didapat PR = 15 dan


PQ = 25. Oleh karena itu, luas segiempat PQRS adalah jumlahan luas
segitiga PSR dan PQR, yang adalah 12 ×12×9+ 12 ×15×20 = 54+150 = 204.
Kemudian, perhatikan bahwa perbandingan PS : SR : PR (panjang sisi-
sisi segitiga PSR) dan QR : RP : PQ (panjang sisi-sisi segitiga QRP)
adalah sama, yakni 4 : 3 : 5. Akibatnya, bisa dikatakan bahwa segi-
tiga PSR sebangun dengan segitiga QRP. Pandang bahwa ∠SPQ =
∠SPR + ∠RPQ = ∠RQP + ∠RPQ = 90°, sehingga panjang diagonal QS
dapat dicari dengan mengaplikasikan
p teorema Pythagoras pada segitiga
SPQ. Ternyata, hasilnya adalah 769. Kita selesai.
Komentar:

a) Penyelesaian soal (a) cenderung mudah karena hanya menuntut


penggunaan teorema Pythagoras, sedangkan penyelesaian soal (b)
agak sulit mengingat diperlukan relasi kesebangunan antardua se-
gitiga. Secara alternatif, soal (b) bisa juga diselesaikan dengan tri-
gonometri, tetapi melibatkan penghitungan yang lebih rumit.
b) Beberapa kemungkinan penyelesaian yang menghasilkan nilai
yang tidak sempurna adalah:
i. Membuat klaim bahwa PQRS adalah trapesium siku-siku de-
ngan ∠SPQ = 90° tanpa bukti; meskipun hal tersebut bisa ‘dili-
hat’ saat kita menggambar sesuai skala, bukti tetap diperlukan
untuk menunjang pengamatan Anda. Klaim tanpa bukti seper-
ti ini bisa berakibat penalti yang cukup besar pada nilai Anda.

24
Pembahasan Kontes Juni 2015

ii. Menganggap bahwa sin a = sin b berakibat kepada a = b; de-


ngan mengambil contoh a = 0 dan b = π, terlihat bahwa per-
nyataan ini salah. Jika terdapat informasi lain (sebagai contoh:
a dan b keduanya lancip), barulah ada kemungkinan bahwa
konsekuensi tersebut valid. Pembaca diharapkan untuk bersi-
kap kritis dan teliti dalam membuat klaim dengan mempertim-
bangkan keseluruhan kasus yang mungkin.

2. Jawab: 626.
Pertama, akan kita buktikan dengan induksi matematika bahwa 4a n <
b n+1 untuk setiap bilangan asli n. Untuk n = 1, 4a 1 = 2500 dan b 2 = 3125,
sehingga ketaksamaan tersebut berlaku. Sekarang, misalkan 4a k < b k+1
untuk suatu bilangan asli n. Akan kita buktikan bahwa 4a k+1 < b k+2 .
Lihat bahwa karena 4a k dan b k+1 keduanya adalah bilangan asli, kita
peroleh 4a k + 1 ≤ b k+1 , sehingga 54a k +1 ≤ 5b k+1 . Namun, kita juga punya
4 × 54a k < 54a k +1 . Akibatnya, 4 × 625a k < 5b k+1 , yang ekuivalen dengan
4a k < b k+1 untuk setiap bilangan asli k.
Selanjutnya, akan dibuktikan bahwa a k+1 < b k+2 untuk setiap bilangan
asli k. Untuk menunjukkannya, cukup lihat bahwa 625a k = 54a k < 5b k+1 ,
yang ekuivalen dengan a k+1 < b k+2 untuk setiap bilangan asli k. Meng-
ingat b 626 > a 625 dan b 625 < a 625 , nilai k terkecil yang memenuhi soal
adalah 626.
Komentar:

a) Observasi bahwa nilai k terkecil yang memenuhi soal adalah 626 bi-
sa dilakukan dengan mengamati kasus-kasus yang lebih kecil. Da-
ri sana, baru kita buktikan dengan induksi matematika identitas-
identitas yang mengarah kepada solusi yang menunjukkan bahwa
memang nilai k tersebut adalah yang terkecil.
b) Salah satu kemungkinan penyelesaian yang menghasilkan nilai
yang tidak sempurna adalah menggunakan pola tanpa dibuktik-
an; perlu diingat bahwa penggunaan pola hanya berfungsi untuk
memberi kita clue mengenai nilai terkecil yang memenuhi, dan ha-
sil yang didapat tidak dapat menggantikan solusi. Sifat yang ter-
dapat pada pola tersebut mesti ditunjukkan kebenarannya, salah
satunya adalah dengan induksi matematika; jika tidak, sebuah ja-
waban dapat dikatakan tidak valid.

25
Buku KTO Matematika 1

3. Jawab: 48.
Untuk mempermudah penulisan, kita nomori keempat baris dengan 1,
2, 3, 4 dan kita beri indeks keempat kolom dengan A, B, C, D. Selanjut-
nya, kita beri indeks setiap kotak dengan string yang merupakan peng-
gabungan nomor baris dan indeks kolom. Sebagai contoh, kotak yang
terletak pada baris ketiga dan kolom C diberi indeks 3C.
Sekarang, tinjau empat kotak di bagian tengah. Jika ada dua kotak ter-
pilih yang terletak pada baris atau kolom yang sama (tanpa mengurangi
keumuman, misalkan kedua kotak tersebut 2B dan 2C), kotak-kotak 1A,
1D, 3A, 3D, 4A, 4B, 4C, dan 4D tidak boleh terisi. Akibatnya, kita hanya
bisa memilih 5 di antara 6 kotak. Perhatikan bahwa kotak mana pun
yang kita buang selalu menghasilkan dua kotak yang membentuk lin-
tasan kuda. Jadi, tidak boleh ada dua kotak terpilih yang terletak pada
baris atau kolom yang sama. Dari sini kita dapatkan bahwa banyaknya
kotak terpilih pada empat kotak di bagian tengah haruslah lebih kecil
atau sama dengan 2.
Jika terdapat dua kotak di bagian tengah yang terisi, kedua kotak terse-
but haruslah merupakan diagonal kotak 2 × 2 di tengah. Tanpa mengura-
ngi keumuman, misalkan kedua kotak tersebut adalah 2B dan 3C. Aki-
batnya, kotak-kotak yang mungkin terisi adalah 1A, 1C, 2D, 3A, 4B, dan
4D. Perhatikan bahwa untuk setiap kombinasi kotak yang terpilih, tidak
ada dua kotak yang membentuk lintasan kuda. Akibatnya, terdapat 6
kemungkinan. Namun, karena kita bisa menempatkan kedua kotak di
tengah dengan 2 cara, terdapat 6 × 2 = 12 cara secara keseluruhan.
Sekarang, kita akan meninjau kasus di mana terdapat satu kotak di bagi-
an tengah yang terisi. Asumsikan kotak tersebut adalah 2B. Akibatnya,
kotak-kotak yang mungkin terisi adalah 1A, 1B, 1C, 2A, 2D, 3A, 4B, dan
4D. Lihat bahwa maksimal satu dari pasangan kotak (1B, 3A) dan (1C,
2A) yang bisa terisi. Akibatnya, kotak-kotak 1A, 2D, 4B, dan 4D harus
terisi. Dengan demikian, kotak yang terpilih dari pasangan (1B, 3A) dan
(1C, 2A) adalah 3A dan 1C. Akibatnya, hanya ada 1 kemungkinan. Na-
mun, karena kita bisa menempatkan satu kotak di tengah dengan 4 cara,
terdapat 1 × 4 = 4 cara secara keseluruhan.
Kasus terakhir yang harus kita tinjau adalah saat keempat kotak di te-
ngah tidak terisi sama sekali. Tinjau dua cycle langkah kuda berikut: 1B-
2D-4C-3A-1B dan 1C-3D-4B-2A-1C. Perhatikan bahwa pada setiap cycle

26
Pembahasan Kontes Juni 2015

paling banyak terisi 2 kotak, sehingga pada gabungan cycle maksimal


terisi empat kotak. Dari sini bisa kita peroleh dua subkasus: terdapat
tiga kotak yang terisi pada gabungan cycle dan semua kotak pada ga-
bungan cycle terisi.
Untuk subkasus pertama, semua kotak di pojok terisi kuda. Kita tinggal
memilih tiga kotak yang terisi pada gabungan cycle, yang banyak cara-
nya adalah 2 × 4 × 2 = 16. Angka 2 pertama diperoleh dari penentuan
cycle yang mengandung 1 dan 2 kuda. Angka 4 diperoleh dari penentuan
kotak yang terisi pada cycle yang berisi 1 kuda. Angka 2 terakhir diper-
oleh dari penentuan dua kotak yang terisi pada cycle yang berisi 2 kuda.
Untuk subkasus kedua, banyaknya cara adalah 4 × 2 × 2 = 16. Angka 4
diperoleh dari pengisian 3 dari 4 kotak di pojok. Masing-masing angka 2
diperoleh dari penentuan dua kotak yang terisi pada cycle, yang kedua-
nya berisi masing-masing 2 kuda. Akibatnya, terdapat 16 + 16 = 32 cara
secara keseluruhan.
Menggabungkan semua kasus, diperoleh bahwa terdapat 12 + 4 + 32 = 48
cara peletakan 7 kuda pada papan catur 4 × 4 sehingga tidak ada dua
kuda yang saling menyerang. Kita selesai.
Komentar:

a) Observasi yang menjadi kunci dari penyelesaian soal ini adalah


pembagian kasus di mana terdapat 2, 1, atau 0 kuda pada kotak
2 × 2 di bagian tengah.
b) Beberapa kemungkinan penyelesaian yang menghasilkan nilai
yang tidak sempurna adalah:
i. Tidak melakukan peninjauan pada seluruh kasus; kesalahan
ini klasik dalam penyelesaian soal bertipe demikian. Untuk
menghindari kesalahan ini, pembaca disarankan untuk meme-
riksa ulang apakah pembagian kasus yang dilakukan sudah
meliputi seluruh kemungkinan yang ada. Pada solusi persa-
maan ini, masing-masing kemungkinan yang bisa didapat pas-
ti tergolong dalam salah satu kasus yang dibuat; inilah cara
pembagian kasus yang valid.
ii. Lupa melakukan perkalian 2 dan 4 pada dua kasus pertama;
perlu diingat bahwa faktor pengali tersebut dibutuhkan dika-
renakan kita sudah terlebih dahulu membuat asumsi-asumsi

27
Buku KTO Matematika 1

terkait dengan lokasi kotak yang terisi pada kotak 2 × 2 di te-


ngah.

28
2 Kontes Juli 2015

2.1 Soal-Soal
Bagian A

Tuliskan jawaban akhir setiap soal di lembar jawaban. Setiap soal bernilai 1
angka. Jawaban Anda harus berupa bilangan bulat dari 0 sampai 999. So-
al yang diberi (*) memiliki jawaban tidak bulat atau tidak di dalam
batas [0, 999].

1. (1002) Diberikan segitiga siku-siku ABC di mana M adalah titik tengah


sisi miring AC. Titik P, Q, R, S, T, U diberikan di luar segitiga ini de-
mikian sehingga ABPQ, BCRS, dan C ATU merupakan bujur sangkar.
Diketahui bahwa hasil jumlah luas ketiga bujur sangkar ini adalah 968.
Tentukan panjang MB.

2. (1009) Tentukan banyaknya bilangan asli di dalam himpunan


{1, 2, . . . , 1000} yang hasil jumlah digit-digitnya habis dibagi 3. Seba-
gai contoh, bilangan 12 jumlah digitnya habis dibagi 3, sementara 20
tidak.

3. (1016) Diberikan segitiga lancip ABC dengan titik pusat lingkaran da-
lam I. Diketahui bahwa ∠BIC = 110°. Tentukan besar ∠BAC.

4. (1023) Misalkan r, s, t adalah akar-akar dari polinom p(x) = x3 − x − 127.


Tentukan nilai dari
1 1 1
µ ¶µ ¶µ ¶
r+ s+ t+ .
s t r

29
Buku KTO Matematika 1

5. (*) (1030) Di Banjarmasin, nomor plat kendaraan diawali dengan huruf


DA, diikuti dengan sebuah bilangan 4 digit yang tidak diawali dengan
angka 0, dan diakhiri dengan 2 huruf dengan syarat: (i) bilangan dari
4 digit tersebut tidak habis dibagi 5 dan (ii) dua huruf terakhir tidak
boleh konsonan keduanya. Misalkan banyaknya kemungkinan nomor
§ n ¨
plat kendaraan adalah n. Tentukan 1000 .
Catatan: Notasi d xe adalah yang menyatakan bilangan bulat terkecil
yang lebih dari atau sama dengan x. Contoh: dπe = 4, d69e = 69.

6. (1037) Misalkan ABCD merupakan sebuah persegi dengan titik pusat


p di dalam persegi ABCD sehingga ∠OPB = 45°.
O. Titik P diberikan
Diketahui PO = 40 2 dan P A = 20. Tentukan panjang PB.

7. (1044) Suatu segitiga memiliki panjang sisi berupa bilangan bulat positif.
Diketahui bahwa 5 dan 10 merupakan panjang salah dua dari sisinya
dan s merupakan panjang sisi yang satu lagi. Tentukan hasil jumlah
semua nilai s yang mungkin.

8. (1051) Terdapat 17 kota yang dapat dituju dari kota Jakarta dengan Olim
Air. Diketahui bahwa jika terdapat k orang yang akan berangkat dari ko-
ta Jakarta ke 17 kota tujuan ini, pasti ada dua kota tujuan yang banyak
penumpangnya sama (bisa jadi tidak terdapat penumpang sama sekali;
dalam hal ini banyaknya penumpang adalah 0). Tentukan nilai terbesar
k yang mungkin.

9. (*) (1058) Diberikan persegi ABCD. Garis singgung dari titik C terhadap
lingkaran luar segitiga ACD bertemu dengan perpanjangan garis AB di
E, dan titik F adalah titik pertemuan kedua lingkaran luar segitiga BCD
dengan garis DE. Tentukan nilai EF × DF, jika diketahui |EC | = 21.

10. (1065) Misalkan x merupakan bilangan real sehingga 2 x + 4 x + 8 x = 1.


Tentukan nilai dari 2 x+1 − 24 x .

11. (1072) Bilangan asli k disebut keras jika memenuhi sifat bahwa: tidak
terdapat bilangan asli a, b sehingga a + b + ab = k. Tentukan banyaknya
bilangan keras di dalam himpunan {1, 2, 3, . . . , 100}.

12. (1079) Di toko buah Piade Mart, Anda ingin membeli 35 buah. Toko buah
tersebut menjual 5 macam buah: pepaya, nanas, semangka, apel, melon.
Ada aturan-aturan ketika melakukan pembelian.

30
Soal-Soal Kontes Juli 2015

a) pepaya yang dibeli harus sebanyak kelipatan 5,


b) nanas yang dibeli maksimal 4,
c) semangka yang dibeli harus berjumlah genap,
d) apel yang dibeli maksimal 1.

Berapa banyak cara membeli buah-buahan jika semua aturan di atas


terpenuhi?

13. (1086) Diberikan titik P(−2, r) dan R(4, 4) pada bidang-x y dengan r suatu
bilangan real positif. Jika titik Q terletak pada sumbu-x, maka nilai ter-
kecil yang mungkin untuk |PQ |+|QR | adalah 10. Tentukan hasil jumlah
semua nilai r yang mungkin.

14. (1093) Pada sebuah segitiga ABC, titik D, E dipilih pada segmen garis
BC sehingga AD = AE dengan titik D, E dalam urutan B, D, E, C. Di-
ketahui AB = 43, AC = 27, dan BD − CE = 20. Tentukan panjang BC.

15. (1100) Sebuah turnamen tenis diikuti 6 pemain sehingga setiap pemain
berhadapan satu sama lain tepat sekali. Untuk setiap permainan, pasti
salah satu pemain menang dan yang satu lagi kalah. Misalkan N adalah
banyaknya kemungkinan hasil semua pertandingan sehingga tidak ada
pemain yang tidak terkalahkan. Tentukan tiga digit terakhir dari N.

16. (1102) Ada 21 petak yang berurutan dari kiri ke kanan, bernomor
0, 1, . . . , 20. Seekor kelinci, mulai dari petak ke-0, ingin mencapai petak
ke-20, dimana ia hanya dapat melompat 1 atau 2 petak ke depan saja.
Misalkan n banyaknya cara kelinci tersebut mencapai petak ke-20, tan-
pa menginjak petak ke-15. Hitung sisa n ketika dibagi 1000.

17. (1104) Diberikan sebuah kertas berukuran persegi panjang. Jika ker-
tas tersebut dilipat terhadap diagonalnya, akan terbentuk suatu segilima
yang luasnya 1116 dari luas persegi panjang awal. Jika lebar dari persegi
panjang tersebut adalah 13, tentukan luas persegi panjang tersebut.

18. (1106) Diberikan suatu himpunan S = {1, 11, 111, 1111, . . . }, yaitu himpun-
an semua bilangan asli yang digit-digitnya adalah 1. Suatu bilangan di-
sebut bersifat kuat jika bilangan tersebut membagi habis setidaknya satu
dari anggota himpunan S. Ada berapakah bilangan kuat yang merupa-
kan anggota himpunan {1, 2, . . . , 1000}?

31
Buku KTO Matematika 1

19. (*) (1108) Diketahui suatu bilangan real a, b, c, dan d memenuhi

a−b a−c
=2 dan = 3.
c−d b−d
Tentukan hasil jumlah semua nilai yang mungkin untuk

a−d
.
b−c

20. (1110) Diberikan segitiga ABC dengan BC = 20 dan misalkan D merupa-


kan titik tengah BC. Lingkaran dalam segitiga ABC memotong segmen
garis AD menjadi tiga potong yang sama panjang. Misalkan a merupa-
kan luas dari segitiga ABC. Tentukan tiga digit terakhir dari a2 .

21. (1112) Tentukan hasil penjumlahan dari semua bilangan asli n yang
memenuhi n − ϕ(n) = 15. Notasi ϕ(n) menyatakan banyaknya bilang-
an asli yang kurang dari n dan relatif prima dengan n. Sebagai contoh,
ϕ(12) = 4.

32
Soal-Soal Kontes Juli 2015

Bagian B

Tuliskan jawaban beserta langkah pekerjaan Anda pada lembar jawaban.


Selain jawaban akhir, Anda perlu menuliskan argumentasi atau langkah-
langkah untuk memperoleh jawaban akhir tersebut. Gunakan halaman yang
berbeda untuk setiap soal yang berbeda. Setiap soal bernilai 7 angka.

1. (1114) Untuk setiap bilangan asli n, misalkan d(n) menyatakan banyak-


nya bilangan asli yang habis membagi n. Sebagai contoh, jika n = 6,
a a a
maka d(n) = 4. Jika n memiliki bentuk aritmetika p 1 1 p 2 2 · · · p k k de-
ngan p 1 , p 2 , . . . , p k merupakan bilangan prima berbeda dan a 1 , a 2 , . . . , a k
merupakan bilangan asli, dapat ditunjukkan bahwa d(n) = (a 1 + 1)(a 2 +
1) · · · (a k + 1).

a) Tentukan nilai dari d(20152015 ).


b) Misalkan m merupakan bilangan asli sehingga d(m) = 20152015 . Be-
rikan satu buah contoh m yang memenuhi.
c) Misalkan m merupakan bilangan asli sehingga d(m) = 20152015 .
Haruskah m merupakan bilangan kuadrat sempurna?

2. (1121) Di sebuah turnamen sepakbola, setiap tim bertanding melawan


tim lainnya tepat sekali, kemenangan diganjar 3 poin, seri diganjar 1
poin, dan kekalahan tidak diganjar poin.
Diketahui di akhir turnamen, Euclid United menjadi juara 1 dengan 8
poin, Trigonspor menjadi juara 2 dengan 7 poin, Analit FC menjadi juara
3 dengan 4 poin, dan Real Kompleks menempati peringkat 4 dengan 4
poin.
Tentukan semua kemungkinan banyak tim sepakbola di turnamen ini.

3. (1128) Misalkan x dan y merupakan bilangan real dengan x y 6= −1 demi-


kian sehingga
x5 y + x y5
= 2.
1 + x 3 y3
Tentukan nilai terkecil yang mungkin untuk x2 + y2 .

33
Buku KTO Matematika 1

2.2 Pembahasan
Bagian A

1. Jawab: 11.

C
R
M

S
B A

P Q

Perhatikan bahwa segitiga ABC adalah siku-siku, sehingga menurut te-


orema Pythagoras, berlaku AB2 + BC 2 = C A 2 . Perhatikan pula bahwa
jumlah luas ketiga persegi adalah AB2 + BC 2 + C A 2 = C A 2 + C A 2 = 2 ×
C A 2 = 936. Akibatnya, C A 2 = 484 dan C A = 22. Karena dalam segitiga
siku-siku titik tengah sisi miring juga merupakan titik pusat dari lingka-
ran luar segitiga, sehingga berlaku M A = MB = MC = M A +2 MC = AC2 = 11.
Komentar:

a) Soal ini tergolong cukup mudah mengingat dapat diselesaikan oleh


82% peserta.
b) Tidak diperlukan pengetahuan mendalam mengenai geometri un-
tuk dapat menyelesaikan soal ini.
c) Kunci mengerjakan soal ini adalah menghubungkan luas persegi
dengan properti Pythagoras yang dimiliki segitiga siku-siku.

2. Jawab: 333.
Perhatikan bahwa untuk setiap bilangan asli n dengan k + 1 digit (sebut
n = a k a k−1 · · · a 0 ), jumlah dari digit-digitnya adalah a k + a k−1 + · · · + a 0 ,

34
Pembahasan Kontes Juli 2015

sementara nilai bilangan tersebut adalah 10k · a k + 10k−1 · a k−1 + · · · + a 0 .


Oleh karena itu, selisih nilai mereka adalah (10k − 1) · a k + (10k−1 − 1) ·
a k−1 + · · · + 9 · a 1 . Karena 10n − 1 ≡ 1n − 1 ≡ 0 (mod 3) untuk sembarang
bilangan cacah n, kita punya pernyataan berikut: jumlah digit-digit dari
sebuah bilangan habis dibagi 3 jika dan hanya jika bilangan tersebut
habis dibagi 3. Akibatnya, banyaknya bilangan dari 1 sampai 1000 yang
habis dibagi 3 adalah 333.
Komentar:

a) Berdasarkan fakta bahwa 92% menjawab soal ini dengan benar, bisa
dikatakan bahwa soal ini sangat mudah.
b) Kunci penyelesaian soal ini adalah kesadaran akan fakta bahwa se-
buah bilangan habis dibagi 3 jika dan hanya jika jumlah digit-digit
penyusunnya juga habis dibagi 3.
c) Dapat ditunjukkan dengan cara serupa bahwa sebuah bilangan ha-
bis dibagi 9 jika dan hanya jika jumlah digit-digit penyusunnya juga
habis dibagi 9. Pembaca dipersilakan mencobanya secara mandiri.
d) Soal ini menggunakan prinsip keterbagian. Untuk mendapatkan
pengetahuan lebih lanjut mengenai keterbagian, pembaca dapat
merujuk ke artikel Wikipedia bahasa Inggris yang berjudul Divisi-
bility rule. Pembaca sangat direkomendasikan untuk membuktikan
validitas setiap pernyataannya.

3. Jawab: 40°.

110°
B
I

35
Buku KTO Matematika 1

Kita tahu bahwa jumlah sudut-sudut dalam sebuah segitiga adalah 180°.
Mengaplikasikan properti tersebut pada segitiga BIC, kita bisa menda-
patkan bahwa ∠ IBC + ∠ ICB = 180° − ∠BIC = 180° − 110° = 70°. Ka-
rena I merupakan titik pusat lingkaran dalam, I merupakan perte-
muan garis-garis bagi segitiga ABC (silakan verifikasi secara mandi-
ri). Akibatnya, ∠ ABC = 2∠ IBC dan ∠ ACB = 2∠ ICB, sehingga kita
dapatkan bahwa ∠ ABC + ∠ ACB = 2(∠ IBC + ∠ ICB) = 2 · 70° = 140° dan
∠BAC = 180° − ∠ ABC − ∠ ACB = 180° − 140° = 40°.
Komentar:

a) Berdasarkan fakta bahwa 42% peserta mampu mengerjakan soal


ini dengan benar, soal ini bisa dikategorikan sedang-sulit menurut
tingkat kesulitannya.
b) Salah satu kunci dalam menyelesaikan soal ini adalah pemahaman
tentang titik-titik spesial dalam segitiga, seperti titik pusat lingka-
ran dalam (biasa dinotasikan I), titik pusat lingkaran luar (biasa
dinotasikan O), titik berat (biasa dinotasikan G), dan titik tinggi
(biasa dinotasikan H). Untuk mempelajari lebih lanjut dalam hal
ini, pembaca dapat merujuk kepada buku Geometry Revisited karya
Coxeter dan Greitzer.
c) Kesalahan yang sering muncul adalah mengasosiasikan titik I se-
bagai titik O. Akibatnya, ∠BAC = 12 ∠BOC disalahartikan menjadi
1
2 ∠BIC.

4. Jawab: 127.
Perhatikan bahwa dengan formula Vieta, kita bisa mendapatkan bahwa
r + s + t = 0, rs + st + tr = −1, dan rst = 127. Akibatnya,

1 1 1 1 1 1 1
µ ¶µ ¶µ ¶ µ ¶
r+ s+ t+ = rst + (r + s + t) + + + +
s t r r s t rst
rs + st + tr + 1
= rst + (r + s + t) +
rst
−1 + 1
= 127 + 0 +
127
= 127.

36
Pembahasan Kontes Juli 2015

Komentar:

a) Berdasarkan fakta bahwa 79% peserta mampu mengerjakan soal


ini, bisa dikatakan bahwa soal ini tergolong mudah.
b) Salah satu kunci dalam menyelesaikan soal ini adalah pemaham-
an tentang formula Vieta untuk polinomial. Selain itu, semuanya
hanyalah manipulasi aljabar yang rutin.

5. Jawab: 1692.
Kita akan menggunakan aturan perkalian untuk mengerjakan soal ini.
Perhatikan bahwa digit pertama dari plat tidak boleh 0, sehingga ada
9 kemungkinan pengisian digit pertama. Selanjutnya, untuk digit ke-
dua dan digit ketiga masing-masing terdapat 10 kemungkinan pengisi-
an. Karena bilangan empat-digit tersebut tidak boleh habis dibagi 5,
terlihat bahwa digit terakhirnya tidak boleh bernilai 0 atau 5. Akibat-
nya, terdapat 8 kemungkinan pengisian digit terakhir. Akibatnya, terda-
pat 9 · 10 · 10 · 8 = 7200 kemungkinan pemilihan bilangan. Sekarang, kita
akan menghitung banyaknya kemungkinan pemilihan 2 huruf terakhir.
Bila tidak ada syarat, dipunyai bahwa ada 26 · 26 = 676 kemungkinan.
Akan tetapi, syarat bahwa dua hurufnya tidak boleh keduanya konsonan
mengharuskan kita untuk mengeliminasi 21 · 21 = 441 kemungkinan sa-
at dua huruf tersebut konsonan. Menurut prinsip komplemen, terdapat
676 − 441 = 235 kemungkinan yang diperbolehkan. Akibatnya, terdapat
7200 · 235 = 1692000 kemungkinan plat nomor. Jawaban yang diinginkan
adalah 1692000
§ ¨
1000 = 1692.
Komentar:

a) 42% dari keseluruhan peserta dapat mengerjakan soal ini dengan


benar, menandakan bahwa soal ini termasuk soal bertingkat kesu-
litan sedang-sulit.
b) Dalam mengerjakan soal menghitung menggunakan prinsip per-
kalian, biasanya terjadi kebingungan mengenai aspek perhitungan
mana yang harus didahulukan. Hal yang harus diingat adalah se-
lalu perhitungkan aspek yang paling bersyarat terlebih dahulu, da-
lam hal ini adalah “digit terakhir tidak boleh bernilai 0 atau 5”.
c) Tidak semua soal perhitungan bisa diselesaikan hanya dengan prin-
sip perkalian. Terkadang, beberapa prinsip dasar lain, seperti prin-

37
Buku KTO Matematika 1

sip komplemen dan prinsip inklusi-eksklusi sangat diperlukan. Hal


ini akan dijelaskan di beberapa soal selanjutnya.
d) Pembaca dapat merujuk kepada referensi yang diberikan di solusi
nomor satu isian singkat KTO bulan Juni untuk menambah penge-
tahuan mengenai prinsip-prinsip berhitung dalam kombinatorika.

6. Jawab: 100.

D C

A B

Perhatikan bahwa karena ∠OPB = 45° dan ∠O AB = 45°, kita dapatkan


ABOP adalah segiempat tali busur. Dengan menerapkan teorema Ptole-
my pada segiempat ABOP, kita dapatkan AB × OP + BO × P A = AO × BP.
Karena O adalah titik pusat persegi ABCD, tanpa p mengurangipkeumum-
an bisa dimisalkan O A = OB =p r. Akibatnya, AB = r 2 + r 2 = r 2. Selan-
jutnya, fakta bahwa PO = 40 2 dan P A = 20 membuat kita dapat mela-
kukan substitusi ke persamaan awal dan mendapatkan 80r + 20r = BP · r.
Dari sana, bisa diperoleh bahwa BP = 80 + 20 = 100.
Komentar:

a) Berdasarkan fakta bahwa 27% mampu menjawab soal ini dengan


benar, soal ini bisa dikatakan cukup sulit.
b) Untuk mengerjakan soal ini, diperlukan kecermatan dalam menya-
dari bahwa ABOP adalah segiempat siklis. Banyak soal geometri
melibatkan pencarian segiempat siklis sebagai ide awalnya.
c) Kunci penyelesaian soal ini ialah penggunaan teorema Ptolemy pa-
da segiempat tali busur. Bunyi teorema ini adalah sebagai beri-

38
Pembahasan Kontes Juli 2015

kut: untuk sembarang segiempat tali busur konveks ABCD, ber-


laku AB × CD + BD × AC = AC × BD. Pembaca direkomendasikan
untuk mencoba membuktikan teorema ini.

7. Jawab: 90.
Menurut ketaksamaan segitiga, tiga sisi dapat membentuk segitiga jika
dan hanya jika setiap sisinya lebih pendek daripada jumlah dua sisi la-
innya. Akibatnya, s harus lebih kecil dari 5 + 10 = 15. Akan tetapi, kita
juga tahu bahwa 10 harus lebih kecil dari s + 5, yang berarti s harus lebih
besar dari 5. Akibatnya, s dapat bernilai 6, 7, 8, . . . , 14. Dapat diverifi-
kasi bahwa semua nilai s tersebut dapat membentuk segitiga, sehingga
semua nilai s yang mungkin adalah 6 + 7 + 8 + 9 + 10 + 11 + 12 + 13 + 14 = 90.
Komentar:

a) Karena 62% peserta mampu mengerjakan soal ini, bisa dikatakan


bahwa soal ini tergolong mudah.
b) Penggunaan ketaksamaan segitiga adalah kunci penyelesaian soal
ini. Ketaksamaan ini mengharuskan tiga bilangan x, y, z memben-
tuk sisi-sisi segitiga jika dan hanya jika jumlah panjang setiap dua
sisi lebih besar dari panjang sisi lainnya. Dengan ketaksamaan ini,
kita bisa melakukan pembatasan terhadap nilai s, sehingga kita da-
pat hasil yang diinginkan.

8. Jawab: 135.
Misalkan a 1 , a 2 , . . . , a 17 adalah 17 bilangan yang menyatakan banyaknya
penumpang ke 17 kota tujuan. Pandang bahwa kita menginginkan agar
17 bilangan ini semuanya berbeda. Perhatikan bahwa jika ke-17 bilang-
an ini adalah 17 bilangan cacah terkecil (yaitu 0, 1, 2, . . . , 16, dan jumlah-
nya 136), kita mendapatkan kasus di mana k = 136 dan tidak terdapat
dua kota yang banyak penumpangnya sama. Jika banyaknya penum-
pang lebih dari 136, dapat ditemukan skenario di mana a 1 , a 2 , . . . , a 16
adalah 0, 1, 2, . . . , 15, dan penumpang yang tersisa semua menuju kota ke-
17. Untuk setiap skenario yang demikian, kita mendapatkan 17 bilang-
an yang saling berbeda. Padahal, yang kita inginkan adalah kepastian
di mana ada 2 bilangan yang sama. Sebagai konsekuensi, kita dapat-
kan k < 136. Sekarang, perhatikan bahwa jika k = 135, kita tidak da-
pat mengambil 17 bilangan cacah berbeda yang jumlahnya 135. Hal ini
benar karena 17 bilangan cacah berbeda memiliki jumlah minimal 136.

39
Buku KTO Matematika 1

Akibatnya, nilai k terbesar sehingga pasti ada dua kota yang banyak pe-
numpang tujuannya sama adalah 135.

40
Pembahasan Kontes Juli 2015

Komentar:

a) Berdasarkan fakta bahwa 23% mampu menjawab soal ini dengan


benar, soal ini bisa dikategorikan sebagai soal sulit.
b) Kunci penyelesaian soal ini adalah bagaimana mendapatkan bi-
langan 135. Ide di balik didapatnya bilangan ini adalah mengobse-
rvasi bahwa 0 + 1 + 2 +· · ·+ 16 = 136. Jika k = 136, kita dapat membu-
at ketujuh belas bilangan tersebut berbeda. Setelah itu, kita tinggal
menunjukkan bahwa saat k = 135, pasti terdapat dua bilangan yang
sama di antara ketujuh belasan tersebut.

1323
9. Jawab: 5 .

B
D

F
E
C

Perhatikan bahwa ∠ECB = ∠CDB = 45°. Akibatnya, ∠CEB = 45° dan


BCE adalah segitiga sama kaki yang siku-siku. Lihat pula bahwa pan-
21
jang BC = p , yang sekaligus merupakan panjang sisi persegi. Kemudi-
2 p
21 21
an, panjang BE = BC = p dan panjang E A = 2 × p = 21 2. Berdasar-
2 2
21
kan properti kuasa titik terhadap lingkaran, EF × ED = EB × E A = p ×
p 2
2 2
21 2 = 441. Selanjutnya, menurut teorema Pythagoras,
p
ED = p
EA +
2 441 2205 21 10 21 10
AD = 2 + 882 = 2 . Oleh karena itu, ED = 2 dan EF = 5 . Ke-
p p p
63 10 21 10 63 10 1323
mudian, DF = ED − EF = 10 , sehingga EF × DF = 5 × 10 = 5 .

Komentar:

a) Soal ini dianulir karena jawabannya tidak bulat.

41
Buku KTO Matematika 1

b) Perhatikan bahwa karena ABCD adalah sebuah persegi, dapat di-


lihat bahwa lingkaran luar segitiga ACD atau BCD sama saja de-
ngan lingkaran luar ABCD. Ide ini penting di soal ini.
c) Teorema kuasa titik pada lingkaran yang digunakan dalam soal ini
berbunyi: jika ABCD adalah segiempat tali busur, dan AB dan CD
berpotongan di E, kita punya E A × EB = EC × ED. Teorema ini bi-
sa dibuktikan menggunakan properti kesebangunan. Pembaca yang
belum pernah membuktikannya direkomendasikan untuk membuk-
tikan teorema ini secara mandiri.

10. Jawab: 1.
Lihat bahwa kita punya persamaan 2 x + 4 x + 8 x = 1. Dengan mengalikan
1 − 2 x ke kedua ruas, kita dapatkan 2 x − 4 x + 4 x − 8 x + 8 x − 16 x = 1 − 2 x ,
yang mengakibatkan 2 x − 16 x = 1 − 2 x . Dengan memindahkan 2 x dari
ruas kanan ke kiri, kita punya 2 x+1 − 24 x = 1.
Komentar:

a) Sebanyak 77% peserta dapat menjawab soal ini dengan benar, me-
nyugestikan bahwa soal ini cenderung mudah.
b) Perhatikan bahwa ide mengalikan 1 − 2 x pada kedua ruas muncul
dari motivasi untuk membuat terjadinya banyak eliminasi suku di
ruas kiri. Ide ini sangat penting dalam penyelesaian soal ini.

11. Jawab: 26.


Dengan menambahkan 1 ke kedua ruas, kita punya 1 + a + b + ab = 1 +
k, yang ekuivalen dengan (1 + a)(1 + b) = (1 + k). Pandang bahwa setiap
bilangan tak prima memiliki faktor selain 1 dan bilangan itu sendiri.
Akibatnya, bilangan tersebut dapat dinyatakan sebagai perkalian dua
bilangan, yang masing-masing lebih besar dari 1. Oleh karena itu, jika
k + 1 bukan prima, terdapat (a, b) yang memenuhi persamaan. Observasi
tersebut memberikan informasi bahwa soal ekuivalen dengan mencari
banyaknya k dari {1, 2, . . . , 100} sedemikian sehingga k+1 adalah bilangan
prima. Perhatikan bahwa di antara {2, 3, . . . , 101} terdapat 26 bilangan
prima, sesuai yang diinginkan soal.
Komentar:

a) Berdasarkan fakta bahwa 38% peserta mampu menyelesaikan soal


ini, bisa dikatakan bahwa soal ini cukup sulit.

42
Pembahasan Kontes Juli 2015

b) Ide menambahkan 1 ke kedua ruas penting dalam mengerjakan so-


al ini. Ide tersebut didasari oleh keinginan untuk memfaktorkan
ruas kiri agar didapatkan sifat dari 1 + k. Setelah itu, kunci lain
dalam pengerjaan soal ini adalah membuat definisi lain untuk bi-
langan kuat, dalam hal ini bilangan k sedemikian sehingga k + 1
adalah bilangan prima. Redefinisi biasanya berguna dalam penye-
lesaian beberapa soal olimpiade matematika, sebagai contoh pada
soal uraian kombinatorika pada KTO bulan Oktober.

12. Jawab: 36.


Misalkan x sebagai jumlah pepaya yang dibeli dan y sebagai jumlah na-
nas yang dibeli. Misalkan juga a sebagai jumlah melon yang dibeli, b
sebagai jumlah semangka yang dibeli dan c sebagai jumlah apel yang
dibeli. Perhatikan bahwa 35 − x − y = a + b + c dan setiap angka dari 0
sampai 35 dapat dinyatakan sebagai 35 − x − y untuk nilai x dan y yang
unik. Hal ini dikarenakan x kelipatan 5 dan nilai y maksimal adalah
4. Akibatnya, jumlah cara membeli buah-buahan ekuivalen dengan ba-
nyaknya pasangan (a, b, c) dengan b genap dan 0 ≤ c ≤ 1 yang memenuhi
0 ≤ a + b + c ≤ 35 atau ekuivalen dengan a + b + c = r dengan 0 ≤ r ≤ 35.
Perhatikan bahwa untuk sembarang nilai a ada nilai b dan c yang unik
sehingga a + b + c = r. Hal ini dikarenakan b adalah bilangan genap dan
nilai yang mungkin dari c hanya 0 dan 1. Hal ini mengakibatkan jum-
lah pasangan (a, b, c) ekuivalen dengan jumlah a. Dari sini, cukup kita
hitung banyaknya nilai a untuk seluruh nilai r. Perhatikan bahwa nilai
a yang mungkin untuk suatu r adalah 0, 1, 2, . . . , r − 1, r atau sebanyak
r + 1. Karena 0 ≤ r ≤ 35, Diperoleh banyaknya nilai a yang mungkin ada
sebanyak 1 + 2 + 3 + · · · + 35 + 36 = 666.
Komentar:

a) Pengerjaan soal ini tidak membutuhkan pengetahuan akan teorema


tertentu. Namun, letak kesulitan soal ini adalah pada pemerolehan
ide.
b) Pembaca sangat disarankan untuk membuktikan pernyataan “seti-
ap angka dari 0 sampai 35 dapat dinyatakan sebagai 35 − x − y untuk
nilai x dan y yang unik. Hal ini dikarenakan x kelipatan 5 dan nilai
y maksimal adalah 4”.

43
Buku KTO Matematika 1

13. Jawab: 4.

y
P(−2, r) R(4, 4)

Q
x

P0

Misalkan P 0 adalah pencerminan P terhadap sumbu-x. Karena Q ber-


ada di sumbu-x, dipunyai |PQ | = |P 0 Q |. Berdasarkan ketaksamaan se-
gitiga, |P 0 Q | + |QR | ≥ |P 0 R |. Kesamaan mungkin terjadi, yakni ketika
kita mengambil titik Q sebagai perpotongan segmen P 0 R dengan sumbu-
x. Akibatnya, |P 0 R | adalah batas minimum, yang sama dengan 10.
Karena P 0 = (−2, − r), berdasarkan teorema Pythagoras, kita dapatkan
(4 − (−2))2 + (4 − (− r))2 = 102 , yang menyebabkan r = 4. Akibatnya, jumlah
dari semua nilai r yang mungkin adalah 4.
Komentar:

a) Berdasarkan fakta bahwa 21% peserta mampu mengerjakan soal


ini, bisa disimpulkan bahwa soal ini tergolong sulit.
b) Hal yang utama dalam penyelesaian soal ini ialah penggunaan ke-
taksamaan segitiga. Utilisasi ketaksamaan ini sangat umum dalam
soal geometri yang melibatkan optimisasi.
c) Perlu diingat bahwa kita hanya perlu mencari nilai r yang positif.
Ketidaktelitian membaca soal dapat menyebabkan nilai r yang ne-
gatif ikut dimasukkan dalam perhitungan, yang tidak diperbolehk-
an.

44
Pembahasan Kontes Juli 2015

14. Jawab: 56.

C E F D B

Misalkan F adalah titik pada BC sehingga AF tegak lurus dengan BC.


Perhatikan segitiga ADE. Segmen AF adalah garis tinggi dari A pada
segitiga ADE. Karena AD = AE, kita punya DF = FE. Akibatnya, 20 =
BD − CE = BD + DF − FE − CE = BF − FC. Tanpa mengurangi keumuman,
misalkan FC = x dan BF = x + 20. Berdasarkan teorema Pythagoras,
AF 2 = AB2 − BF 2 = AC 2 − CF 2 . Akibatnya, 432 − (x + 20)2 = 272 − x2 , yang
ekuivalen dengan 40x = 720. Sebagai kesimpulan, x = 18, dan panjang
BC adalah (x + 20) + x = 38 + 18 = 56.
Komentar:

a) Berdasarkan fakta bahwa 42% peserta mampu mengerjakan soal ini


dengan benar, soal ini bisa dikatakan bertingkat kesulitan sedang.
b) Penggunaan teorema Pythagoras menjadi kunci dalam penyelesai-
an soal ini.

15. Jawab: 624.


Kita akan menggunakan prinsip komplemen. Kita akan menghitung ba-
nyaknya kemungkinan hasil pertandingan secara keseluruhan, kemudi-
an menguranginya dengan banyaknya kemungkinan hasil pertandingan
di mana ada pemain yang tidak terkalahkan. Karena terdapat 6 pemain,
secara keseluruhan ada 62 = 15 pertandingan. Kemudian, setiap pertan-
¡ ¢

dingan memiliki 2 kemungkinan hasil yang bergantung pada siapa yang


memenangi pertandingan tersebut. Akibatnya, total ada 215 = 32768 ke-
mungkinan. Jika ada pemain yang tidak terkalahkan, perhatikan bahwa

45
Buku KTO Matematika 1

hanya bisa ada maksimal 1 orang yang tidak terkalahkan. Lihat bahwa
terdapat 6 kemungkinan pemilihan pemain yang tidak terkalahkan ter-
sebut. Pemain tersebut memenangi kelima pertandingan melawan pe-
main lain. Untuk 10 pertandingan lain, terdapat 210 kemungkinan hasil.
Selanjutnya, menurut prinsip perkalian, terdapat 6 × 210 = 6144 kemung-
kinan hasil pertandingan di mana ada pemain yang tidak terkalahkan.
Jadi, ada 32768 − 6144 = 26624 kemungkinan hasil pertandingan di ma-
na tidak ada pemain yang tidak terkalahkan, dan tiga digit terakhirnya
adalah 624.
Komentar:

a) Berdasarkan fakta hanya 6% peserta yang mampu menyelesaikan


soal ini, soal ini layak untuk dikatakan sangat sulit.
b) Soal ini diselesaikan dengan menggunakan gabungan prinsip kom-
plemen dan prinsip perkalian. Pendekatan seperti ini sering dite-
mukan dalam pengerjaan soal-soal kombinatorika berhitung. Prin-
sip komplemen biasa digunakan dalam kasus yang mana penghi-
tungan banyaknya kemungkinan sebuah kondisi tidak terjadi lebih
mudah dibandingkan penghitungan banyaknya kemungkinan sebu-
ah kondisi terjadi.

16. Jawab: 50.


Perhatikan bahwa ada 1 cara untuk mencapai petak ke-1 (lompat 1 petak
dari petak ke-0), sementara ada 2 cara untuk mencapi petak ke-2 (lompat
2 petak dari petak ke-0 atau lompat 1 petak sebanyak dua kali). Untuk
petak ke-p dengan p > 2 (kecuali p = 15), banyak cara mencapai petak
tersebut sama dengan jumlah banyak cara mencapai 2 petak sebelum-
nya (masing-masing) karena hanya dari 2 petak tersebut kelinci dapat
mencapai petak ke-p. Akibatnya, kita dapat menuliskan satu demi satu
banyaknya cara mencapai setiap petak secara rekursif dari petak ke-1
sampai petak ke-14, yakni 1, 2, 3, 5, 8, 13, 21, 34, 55, 89, 144, 233, 377,
610. Dari sini dapat diselidiki bahwa terdapat 610 cara untuk mencapai
petak ke-14. Namun, setelah itu kelinci tersebut harus melompati 2 pe-
tak sekaligus dengan tujuan menghindari petak ke-15. Akibatnya, ada
610 cara untuk mencapai petak ke-16. Dengan cara yang sama (relasi
rekurensi), dapat ditunjukkan bahwa terdapat 610 cara untuk mencapai
petak ke-17. Melanjutkan relasi rekurensi tersebut, kita peroleh 3050

46
Pembahasan Kontes Juli 2015

cara untuk mencapai petak ke-20. Jawaban yang diinginkan adalah 50,
yakni sisa pembagian bilangan tersebut dengan 1000.
Komentar:

a) Berdasarkan fakta bahwa 27% peserta mampu mengerjakan soal


ini dengan benar, bisa dikatakan bahwa soal ini tergolong sulit.
b) Soal ini menggunakan teknik relasi rekurensi. Teknik ini sering di-
gunakan dalam soal kombinatorika berhitung. Untuk mendapatkan
pengetahuan lebih lanjut mengenai teknik tersebut, pembaca dapat
merujuk kepada buku Principles and Techniques in Combinatorics,
buku yang sama dengan buku rujukan untuk soal nomor 1 isian
singkat KTO Juni.
c) Sifat rekursif seperti ini juga ditunjukkan oleh bilangan Fibonacci.

17. Jawab: 338.

B A B E A

C D C

Misalkan persegi panjang yang dimaksud adalah ABCD, dengan AB


dan CD adalah sisi yang lebih panjang. Tanpa mengurangi keumum-
an, misalkan persegi panjang tersebut dilipat terhadap garis AC. Mi-
salkan E adalah perpotongan AB dan CD. Dari soal, kita tahu bahwa
11
luas daerah yang terbentuk sekarang adalah 16 dari luas awal. Aki-
5
batnya luas segitiga AEC adalah 16 dari luas persegi panjang. Kare-
na luas segitiga ABC adalah setengah dari luas persegi panjang awal,
3
diperoleh luas segitiga BEC adalah 16 dari luas persegi panjang, sehing-
ga kita bisa dapatkan AE : EB = 5 : 3. Karena AE = EC, kita temukan
EC : EB = 5 : 3. Dengan menggunakan teorema Pythagoras, kita juga
dapatkan EC : EB : BC = 5 : 3 : 4. Akibatnya, AB : BC = (5 + 3) : 4 = 2 : 1.

47
Buku KTO Matematika 1

Dari sini, kita peroleh bahwa panjang persegi panjang tersebut adalah
2 × 13 = 26 dan luasnya 26 × 13 = 338.
Komentar:

a) Fakta bahwa 37% peserta mampu mengerjakan soal ini dengan be-
nar menyarankan bahwa soal ini tergolong cukup sulit.
b) Tidak diperlukan teknik pengerjaan soal geometri yang mendalam
untuk dapat mengerjakan soal ini; hanya diperlukan kecermatan
dalam mencari relasi antarluas daerah dan antarpanjang sisi, yang
harus menjadi pusat perhatian dalam pengamatan.

18. Jawab: 400.


Perhatikan bahwa karena setiap bilangan pada himpunan S adalah gan-
jil dan memiliki digit satuan 1, bisa disimpulkan bahwa tidak ada bi-
langan yang habis dibagi 2 ataupun 5. Akibatnya, untuk sembarang bi-
langan dari {1, 2, . . . , 1000}, bila bilangan tersebut memiliki faktor 2 atau
5, bilangan tersebut tidak akan membagi semua bilangan di S. Hal
ini menandakan bahwa bilangan tersebut tidak kuat. Untuk bilangan-
bilangan dari {1, 2, . . . , 1000} yang tidak memiliki faktor 2 ataupun 5, ki-
ta bisa memisalkan bilangan tersebut adalah k. Perhatikan bahwa 10
relatif prima dengan 9k karena 2 dan 5 tidak membagi 9k. Dengan
menggunakan teorema Euler, ada bilangan t sehingga 10 t ≡ 1 (mod 9k),
yang berarti 9k habis membagi 999 . . . 999 dengan 9 sebanyak t kali,
atau k membagi 111 . . . 111 dengan 1 sebanyak t kali. Akibatnya, ba-
nyaknya bilangan kuat sama dengan banyaknya bilangan yang tidak
habis dibagi 2 dan tidak habis dibagi 5. Karena banyaknya bilangan
yang habis dibagi 2 adalah 1000 2 = 500 dan banyaknya bilangan yang ha-
bis dibagi 5 adalah 1000 5 = 200, banyaknya bilangan yang habis dibagi 2
dan 5 = 1000
10 = 100. Akibatnya, banyaknya bilangan kuat sama dengan
1000 − 500 − 200 + 100 = 400.
Komentar:

a) Berdasarkan fakta bahwa 29% peserta dapat menjawab soal ini de-
ngan benar, bisa dikatakan bahwa soal ini tergolong cukup sulit.
b) Kita menggunakan teorema Euler untuk menyelesaikan soal ini,
yang berbunyi: untuk sembarang bilangan a, b yang relatif prima,
terdapat bilangan t sedemikian sehingga t = φ(b), di mana φ adalah
fungsi Euler-phi, sehingga a t = 1 (mod b).

48
Pembahasan Kontes Juli 2015

c) Kesalahan yang sering terjadi adalah kekeliruan dalam menghi-


tung banyaknya bilangan yang tidak habis dibagi 2 maupun 5.
Untuk menghitungnya, kita dapat menggunakan prinsip inklusi-
eksklusi. Penting untuk diingat bahwa ada bilangan-bilangan yang
terhitung dua kali, yakni yang habis dibagi 2 dan 5.

19. Jawab: -5.


Pandang bahwa kita punya 2 persamaan. Persamaan pertama ekuivalen
dengan a − b = 2(c − d), yang ekuivalen dengan a + 2d = b + 2c, yang eku-
ivalen dengan 4(a + 2d) = 4(b + 2c). Persamaan kedua ekuivalen dengan
a − c = 3(b − d), yang ekuivalen dengan a + 3d = 3b + c, yang ekuivalen
dengan 3(a + 3d) = 3(3b + c). Jika kita kurangkan persamaan pertama
dengan persamaan kedua, didapatkan a − d = −5b + 5c = −5(b − c). Aki-
batnya, ab−−dc = −5.
Komentar:

a) Berdasarkan fakta bahwa 44% peserta mampu mengerjakan soal


ini, bisa dikata-kan bahwa soal ini tergolong bertingkat kesulitan
sedang.
a− b
b) Ternyata, dapat pula dibuktikan bahwa c− d + ba−−dc + ab−−dc = 0.

20. Jawab: 64.

F Y

B E D C

Tanpa mengurangi keumuman, misalkan AB < AC. Misalkan pula pan-


jang AB = c, BC = a, C A = b, dan s adalah setengah keliling segitiga
ABC.
Misalkan lingkaran dalam segitiga ABC bertemu BC dan AB masing-
masing di E dan F. Selain itu, misalkan pula AD bertemu lingkaran

49
Buku KTO Matematika 1

dalam tersebut di titik X dan Y , dengan X lebih dekat ke A daripada Y .


Kemudian, misalkan A X = X Y = Y D = x. Menurut teorema titik kuasa
terhadap lingkaran, DE 2 = DY · D X = x · 2x = 2x2 . Dengan cara yang
sama, dipunyai AF 2 = 2x2 . Akibatnya, DE 2 = AF 2 .
Diketahui bahwa CE = s − c dan AF = s − a. Sebagai akibat, DE = CE −
CD = s − c − 10. Karena DE 2 = AF 2 , kita punya (s − c − 10)2 = (s − a)2 . Ini
berarti s − c − 10 = ±(s − a), sehingga a = c + 10 (dari sini, diperoleh c = 10
karena a = 20) atau 2s − a − c = 10 =⇒ b = 10. Dengan asumsi c < b dan
ketaksamaan segitiga (b + c > a = 20), bisa didapatkan bahwa c = 10.
Kemudian, menurut teorema Stewart, dipunyai 10b2 + 10c2 = 180x2 +
2 2
2000. Dari sini diperoleh bahwa 2x2 = b + c9−200 . Namun, dari fakta bah-
2 2
wa AF 2 = 2x2 , kita dapat (s − a)2 = 2x2 , sehingga (s − a)2 = b + c9−200 . Ka-
rena s = a+2b+ c = 15 + 2b dan c = 10, diperoleh bahwa b2 − 36b + 260 = 0.
Dari sini, nilai b yang mungkin adalah 10 atau 26. Namun, karena c < b,
didapatkan b = 26.
p
Menggunakan
p rumus Heron, luas segitiga tersebut adalah 28 · 8 · 18 · 2 =
24 14, sehingga a = 8064, dan tiga digit terakhirnya ialah 64.
Komentar:

a) Pada kontes sebenarnya, soal ini dianulir dikarenakan di formulasi


aslinya tertulis “Lingkaran dalam segitiga ABC memotong segmen
garis BD menjadi tiga potong yang sama panjang”.
b) Solusi ini menggunakan fakta bahwa CE = s − c dan AF = s − a, yang
bisa didapat dengan melihat bahwa BF = BE dan seterusnya.

21. Jawab: 94.


Misalkan f (n) = n − φ(n). Akibatnya, f (n) dapat diartikan sebagai ba-
nyaknya bilangan dari 1, 2, . . . , n yang tidak relatif prima dengan n, atau
dengan kata lain memiliki faktor persekutan dengan n. Kita akan men-
cari banyaknya bilangan asli n sehingga f (n) = 15. Perhatikan bahwa
jika n merupakan bilangan prima, bilangan asli yang memiliki faktor
persekutuan dengan n dan lebih kecil dari atau sama dengan n hanya-
lah n itu sendiri. Akibatnya, f (n) = 1, yang tidak memenuhi soal. Dari
sini bisa didapatkan bahwa n memiliki setidaknya 2 faktor prima. Un-
tuk sembarang bilangan asli n, misalkan n = pq, dengan p adalah faktor
prima terkecil dari n. Perhatikan bahwa p ≤ q karena jika q < p, terda-
pat faktor prima dari q yang lebih kecil dari p. Kemudian, perhatikan

50
Pembahasan Kontes Juli 2015

bahwa q, 2q, . . . , pq semuanya memiliki faktor persekutuan q dengan n.


Jika p > 15, akan kita dapatkan f (n) > 15. Akibatnya, haruslah p ≤ 15.
Dengan cara yang serupa, kita tahu bahwa p, 2p, . . . , q p semuanya me-
miliki faktor persekutuan q dengan n. Jika q > 15, akan kita dapatkan
f (n) > 15. Akibatnya, haruslah q ≤ 15. Kita dapat simpulkan bahwa
1 < p ≤ q ≤ 15. Perhatikan bahwa terdapat 6 bilangan prima yang lebih
kecil dari 15, yaitu 2, 3, 5, 7, 11, 13. Kita akan membagi kasus berdasar-
kan kemungkinan nilai p.

a) Jika p = 2, karena f (n) = n − φ(n) < n, maka nilai q setidaknya 8.


Dapat diverifikasi untuk q = 8, 9, 10, 11, 12, 13, 14, 15 tidak ada solusi
yang memenuhi.
b) Jika p = 3, karena q hanya dapat memiliki faktor-faktor prima yang
lebih besar dari atau sama dengan 3, maka q = 3, 5, 7, 11, 13 (prima)
atau 3 · 3, 3 · 5. Dapat diverifikasi bahwa hanya q = 13 yang meme-
nuhi kondisi soal.
c) Jika p = 5, perhatikan bahwa jika p ≥ 5, q hanya memiliki faktor-
faktor prima yang lebih besar dari atau sama dengan 5. Akibat-
nya, q tidak bisa memiliki lebih dari dua faktor prima karena akan
mengakibatkan q ≥ 5 · 5 = 25, yang bertentangan dengan q ≤ 15. Ja-
di, q harus merupakan bilangan prima, dan q yang mungkin adalah
5, 7, 11, 13. Dapat diverifikasi bahwa hanya q = 11 yang memenuhi
kondisi soal.
d) Jika p = 7, q yang mungkin adalah 7, 11, 13. Dapat diverifikasi
bahwa tidak ada yang memenuhi kondisi soal.
e) Jika p = 11, q yang mungkin adalah 11, 13. Dapat diverifikasi bah-
wa tidak ada yang memenuhi kondisi soal.
f) Jika p = 13, q yang mungkin adalah 13. Dapat diverifikasi bahwa
tidak ada yang memenuhi kondisi soal.

Dari analisis kasus di atas, didapatkan bahwa semua nilai n yang meme-
nuhi adalah 3 · 13 = 39 dan 5 · 11 = 55. Jumlah dari kedua bilangan yang
memenuhi adalah 39 + 55 = 94.
Komentar:

a) Berdasarkan fakta bahwa 25% peserta mampu mengerjakan soal ini


dengan benar, bisa dikatakan bahwa soal ini tergolong cukup sulit.

51
Buku KTO Matematika 1

b) Ide dasar dari pengerjaan soal ini adalah membatasi nilai p dan q
sehingga verifikasi satu-satu bisa dilakukan. Perlu juga ketelitian
untuk menghitung banyaknya solusi dari keseluruhan kasus.

52
Pembahasan Kontes Juli 2015

Bagian B

1. Jawab: (a) 20163 ; (c) harus.

a) Perhatikan bahwa 2015 dapat difaktorisasi prima menjadi 5 × 13 ×


31. Akibatnya, 20152015 = 52015 × 132015 × 312015 . Berdasarkan defi-
nisi yang diberikan di soal, d(20152015 ) = (2015 + 1)(2015 + 1)(2015 +
1) = 20163 .
b) Perhatikan bahwa kita menginginkan d(n) = (a 1 + 1)(a 2 + 1) · · · (a k +
1) = 20152015 . Akibatnya, kita dapat mengambil k = 2015 dan
a i + 1 = 2015 untuk setiap i dari 1 sampai 2015, sehingga a i =
2014. Untuk p 1 , p 2 , . . . , p 2015 , kita dapat menggunakan 2015 bi-
a a a 2015
langan prima terkecil yang berbeda. Maka, n = p 1 1 p 2 2 · · · p 2015 =
2014
(p 1 p 2 · · · p 2015 ) memenuhi kondisi soal.
c) Harus. Misalkan m bukan merupakan bilangan kuadrat. Untuk
setiap faktor p dari m, ada q sehingga pq = m. Perhatikan bahwa
q pasti berbeda dari p karena jika sama, m pasti merupakan bi-
langan kuadrat. Akibatnya, kumpulan semua faktor dari m dapat
dibagi menjadi pasangan-pasangan sehingga perkalian dua bilang-
an yang bersesuaian adalah m. Dari sini, bisa disimpulkan bahwa
banyaknya faktor dari m haruslah genap. Padahal, 20152015 adalah
bilangan ganjil. Akibatnya, terjadi kontradiksi, yang mengharusk-
an m adalah bilangan kuadrat.

Komentar:

a) Pembaca yang belum mengerti mengapa banyaknya faktor bulat


positif dari sebuah bilangan asli n dapat dihitung menggunakan
formula pada soal dipersilakan mencoba membuktikannya sendiri
menggunakan prinsip perkalian.
b) Untuk 1(a), kesalahan yang sering terjadi adalah lupa untuk mela-
kukan faktorisasi terlebih dahulu. Perhatikan bahwa definisi fungsi
d(n) mengharuskan faktorisasi prima untuk dilakukan.
c) Untuk 1(b), dapat dibuktikan lebih lanjut bahwa untuk sembarang
bilangan asli k, terdapat bilangan asli n sehingga d(n) = k. Untuk
membuktikannya, pembaca dapat menggunakan cara yang serupa
(melakukan faktorisasi k, kemudian mengambil prima-prima ber-
beda untuk n).

53
Buku KTO Matematika 1

d) Untuk 1(c), perhatikan bahwa secara intuitif, kita dapat memba-


gi faktor-faktor sebuah bilangan menjadi pasangan-pasangan yang
hasil kalinya bilangan itu sendiri (sebagai contoh, (1,12), (2, 6), (3,
4) adalah pemasangan faktor-faktor untuk n = 12). Jika n adalah
sebuah bilangan kuadrat, misal 25, perhatikan bahwa 5 harus ber-
pasangan dengan 5 itu sendiri. Hal ini memberikan sugesti intuitif
bahwa n adalah bilangan kuadrat jika dan hanya jika banyak fak-
tornya adalah ganjil.

2. Jawab: 5.
Misalkan banyaknya tim adalah n. Perhatikan bahwa ada n2 = n(n2−1)
¡ ¢

pertandingan yang dilaksanakan karena setiap dua tim bertemu tepat


sekali. Dalam setiap pertandingan, ada 2 poin yang dibagikan (saat ke-
dua tim seri, masing-masing mendapat 1 poin) atau 3 poin yang dibagik-
an (saat salah satu tim menang dan mendapatkan 3 poin). Oleh karena
itu, pada setiap pertandingan ada minimal 2 poin yang diberikan. Aki-
batnya, total poin seluruh tim lebih besar atau sama dengan n(n2−1) · 2 =
n(n − 1). Padahal, karena poin setiap tim di bawah posisi ke-4 semuanya
kurang dari atau sama dengan 4, bisa diperoleh bahwa total nilai setiap
tim kurang dari atau sama dengan 8 + 7 + 4 + 4 + (n − 4) · 4 = 4n + 7. Kita
dapatkan n(n − 1) ≤ 4n + 7, atau n(n − 5) ≤ 7. Jika n > 6, kita dapatkan
n(n − 5) ≥ 7 × 2 > 7. Akibatnya n yang mungkin hanyalah 4, 5, atau 6.
Kita akan menganalisis setiap kasus.

a) Jika n = 4, tim Euclid United mendapat 8 poin hanya dengan me-


lawan 3 tim lain. Hal ini tidak memungkinkan karena 8 tidak bisa
dinyatakan sebagai penjumlahan 3 bilangan asli yang dipilih dari
himpunan {3, 1, 0}.
b) Jika n = 5, untuk mempermudah penulisan, misalkan Tim A adalah
Euclid United, Tim B adalah Trigonspor, Tim C adalah Analit FC,
Tim D adalah Real Kompleks, dan Tim E adalah tim lain. Jika
A mengalahkan C dan D dan seri dengan B dan E, kemudian B
mengalahkan C dan D dan B dikalahkan oleh E, kemudian C seri
dengan D dan keduanya mengalahkan E, didapatkan poin A = 8,
poin B = 7, dan poin C, D, E masing-masing 4 poin. Konfigurasi
tersebut memenuhi kondisi soal.
c) Jika n = 6, terdapat 62 = 15 pertandingan yang dimainkan. Perha-
¡ ¢

tikan bahwa nilai tim di peringkat ke-5 dan ke-6 tidak bisa melebihi

54
Pembahasan Kontes Juli 2015

nilai tim Real Kompleks yang ada di peringkat 4. Akibatnya, total


maksimal nilai semua tim adalah 8 + 7 + 4 + 4 + 4 + 4 = 31. Jika semua
pertandingan berakhir seri, akan ada total 30 poin yang dibagikan
ke semua tim. Karena total poin paling banyak adalah 31, terdapat
paling banyak satu pertandingan yang berakhir tidak seri. Padahal,
agar mendapatkan 8 poin dalam 5 pertandingan, tim Euclid United
harus menang 2 kali, seri 2 kali, dan kalah 1 kali, yang tidak mung-
kin terjadi. Dengan demikian kita dapatkan bahwa kasus di mana
banyak peserta adalah 6 tim tidak memungkinkan.

Jadi, banyaknya tim yang mungkin hanyalah 5.


Komentar:

a) Dalam penyelesaian soal ini, observasi yang penting adalah bahwa


setiap pertandingan minimal menghasilkan 2 poin, dan banyaknya
tim yang mungkin hanyalah 4, 5, atau 6.
b) Kesalahan umum yang dilakukan para peserta adalah sebagai ber-
ikut:
i. Tidak menuliskan contoh kasus dengan 5 tim. Hal ini sangat
fatal karena jika tidak dibuktikan, hal di mana tidak ada kasus
yang memenuhi syarat soal untuk 5 tim mungkin saja terjadi.
ii. Lupa menyadari bahwa kasus pertandingan seri hanya meng-
hasilkan total 2 poin untuk kedua tim.
1 1
3. Jawab: 3 2 2 3 .
Misalkan x y = a dan x2 + y2 = b. Akibatnya, soal ekuivalen dengan b2 =
4a3 +2
a (perhatikan bahwa a 6= 0). Perhatikan bahwa kita menginginkan
nilai minimum dari b. Namun, karena b ≥ 0, meminimumkan b sama
saja dengan meminimumkan b2 . Pertama, akan ditunjukkan bahwa a
haruslah positif. Misalkan a < 0. Karena (x + y)2 ≥ 0, diperoleh b ≥ −2a,
3
yang ekuivalen dengan b2 ≥ 4a2 . Oleh karena itu, 4aa+2 − 4a2 ≥ 0, yang
berakibat kepada a2 ≥ 0, sebuah kontradiksi mengingat a < 0. Jadi, a > 0.
Perhatikan bahwa menurut ketaksamaan AM-GM, diperoleh
1
4a3 + 1 + 1 3 · 4 3 · a 1
b2 = ≥ = 3·43 .
a a
Perhatikan bahwa kesamaan mungkin terjadi. Akibatnya, nilai mini-
1 1
mum dari b adalah 3 2 2 3 . Kita selesai.

55
Buku KTO Matematika 1

Komentar:

a) Teknik pengerjaan soal ini adalah memisalkan x y = a dan x2 + y2 =


b. Hal ini intuitif karena ekspresi-ekspresi pada soal mengandung
kedua bentuk tersebut.
b) Kesalahan umum yang dilakukan oleh banyak sekali peserta yang
mengerjakan soal ini adalah tidak mengecek bahwa terdapat x dan
y yang bersesuaian dengan nilai a dan b yang diperoleh dengan ke-
taksamaan AM-GM. Hal tersebut sangat fatal karena terdapat ke-
mungkinan di mana tidak ada x dan y yang memenuhi, contohnya
saat a = 2 dan b = 1.

56
3 Kontes Agustus 2015

3.1 Soal-Soal
Bagian A

Tuliskan jawaban akhir setiap soal di lembar jawaban. Setiap soal bernilai 1
angka. Jawaban Anda harus berupa bilangan bulat dari 0 sampai 999. So-
al yang diberi (*) memiliki jawaban tidak bulat atau tidak di dalam
batas [0, 999].

1. (1003) Diketahui p, q, r bilangan prima yang memenuhi p + q = r. Jika


p anggota {1, 2, . . . , 100} tentukan nilai terbesar p yang mungkin.

2. (1010) Diberikan bilangan real positif a, b, c yang memenuhi

ab + a + b = 5
ac + a + c = 9
bc + b + c = 14.

Carilah nilai a + b + c.

3. (1017) Tentukan banyaknya pasangan terurut bilangan asli (k, l, m) de-


mikian sehingga k + 2l + m = 2k + l − 2m = 2015.

4. (1024) Jika a merupakan bilangan real terbesar yang memenuhi persa-


maan
1
x2 + = 2,
x
tentukanlah nilai a2 + a + 1.

57
Buku KTO Matematika 1

5. (1031) Diberikan persegi ABCD. Misalkan E dan F berturut-turut titik


tengah dari sisi AD dan AB dan misalkan pula G merupakan titik potong
antara garis CE dan DF. Diketahui bahwa luas segitiga DEG adalah 1.
Hitung luas persegi ABCD.

6. (*) (1038) Fungsi f memenuhi persamaan

f (2 x ) + x f (2− x ) = 1

untuk sembarang bilangan real x. Hitunglah nilai


1 1
³ p ´ + ³ p ´.
f 21 + 2 f 23 + 7

7. (*) (1045) Diberikan segitiga ABC dengan A 1 , B1 , dan C 1 berturut-turut


merupakan titik tengah sisi BC, C A, dan AB. Misalkan D adalah kaki
tegak lurus dari titik A ke sisi BC di mana D terletak di antara B dan
A 1 . Diketahui bahwa B1 D tegak lurus dengan A 1 C 1 . Diketahui bahwa
A 1 C 1 = 10 dan luas segitiga ABC adalah 150. Tentukan luas segitiga
A 1 C 1 D.

8. (1052) Tentukan banyaknya pasangan terurut (a, b, c) demikian sehingga

a − bc2 ≡ 1 (mod 13)


ac + b ≡ 4 (mod 13)

di mana 0 ≤ a < 13, 0 ≤ b < 13, and 0 ≤ c < 13.

9. (1059) Diberikan himpunan A = {1, 2, 3, 4, 5, 6, 7}. Tentukan banyaknya


fungsi f : A → A yang memenuhi f ( f (x)) = x untuk setiap x ∈ A.
p
10. (1066) Definisikan barisan a n sebagai a n = (9 + 69)n . Tentukan ba-
§ ¨

nyak bilangan dari a 1 , a 2 , . . . , a 1000 yang habis dibagi 9.

11. (1073) Misalkan ABCD merupakan trapesium demikian sehingga A,


B, C, D terletak pada sebuah lingkaran yang pusatnya di O dan sisi
AB sejajar sisi CD. Diagonal AC dan BD berpotongan di titik M dan
∠ AMD = 60°. Diketahui MO = 10. Diketahui bahwa selisih panjang AB
p
dan CD dapat ditulis dalam bentuk m n di mana m dan n adalah bi-
langan asli dan n tidak habis dibagi oleh kuadrat suatu bilangan prima.
Hitunglah nilai m + n.

58
Soal-Soal Kontes Agustus 2015

12. Tentukanlah banyaknya permutasi (a 1 , a 2 , . . . , a 10 ) dari 1, 2, . . . , 10 yang


memenuhi
|a 1 − 1| + |a 2 − 2| + · · · + |a 10 − 10| = 4.

13. [Soal ini tidak memiliki jawaban] Misalkan N bilangan asli terbesar
yang tidak dapat dinyatakan sebagai 2a + 11b, dimana a dan b bilangan
¥N¦
bulat nonnegatif. Tentukan 10 .

14. (1094) Misalkan I adalah titik pusat lingkaran dalam segitiga ABC dan
O adalah titik pusat excircle terhadap titik B. Jika BI = 12, OI = 18, dan
BC = 15, hitunglah panjang AB.

59
Buku KTO Matematika 1

Bagian B

Tuliskan jawaban beserta langkah pekerjaan Anda pada lembar jawaban.


Selain jawaban akhir, Anda perlu menuliskan argumentasi atau langkah-
langkah untuk memperoleh jawaban akhir tersebut. Gunakan halaman yang
berbeda untuk setiap soal yang berbeda. Setiap soal bernilai 7 angka.

1. (1115) Di Zimbabwe, ada 3 jenis penyakit singa: sakit perut, sakit gigi,
dan sakit kepala. Ternyata, di sebuah kebun binatang di Zimbabwe, ada
24 ekor singa yang sakit perut, 25 ekor singa yang sakit gigi, dan 26 ekor
singa yang sakit kepala. Namun, ada keajaiban. Jika 2 singa dengan
penyakit berbeda bertemu, maka penyakit mereka berubah menjadi pe-
nyakit yang ketiga yang berbeda dari penyakit awal kedua singa terse-
but. Apakah kita dapat mengatur cara bertemu para singa agar semua
singa memiliki penyakit yang sama? Jika ya, tunjukkan caranya. Jika ti-
dak, buktikan secara matematis (tidak bisa hanya dengan mencoba-coba
dengan beberapa contoh saja).

2. (1122) Diberikan dua buah lingkaran Γ1 dan Γ2 yang berpotongan di dua


buah titik X dan Y . Titik P dan Q berturut-turut terletak pada Γ1 dan
Γ2 sehingga PQ adalah garis singgung persekutuan luar Γ1 dan Γ2 dan
PQ lebih dekat dengan X daripada dengan Y . Garis X Y memotong garis
PQ di titik M.

(a) (Teorema Secant-Tangent) Tunjukkan bahwa M X · MY = MP 2 .


(b) Tunjukkan bahwa M merupakan titik tengah segmen garis PQ.
(c) Diberikan titik Y sehingga QY adalah merupakan diameter dari
lingkaran Γ2 dan berlaku PQ = 2 · QY . Tentukanlah besar sudut
∠PQ X .
3. (1129) Apakah terdapat bilangan asli yang habis dibagi 2015 dan hasil
jumlah digit-digitnya adalah 2015?

4. (1134) Diberikan barisan f 0 , f 1 , . . . yang didefinisikan dengan f 0 = 0, f 1 =


1, dan f n+1 = f n + f n−1 untuk setiap bilangan asli n ≥ 1. Tunjukkan bahwa
untuk setiap bilangan asli n, berlaku
à p !4
1 1 1 1 1 1+ 5
+ + +···+ < .
f1 f2 f3 fn 2 2

60
Pembahasan Kontes Agustus 2015

3.2 Pembahasan
Bagian A

1. Jawab: 71.
Perhatikan paritas ketiga bilangan tersebut. Jika ketiga bilangan terse-
but semuanya ganjil, lihat bahwa jumlah dua bilangan ganjil haruslah
genap, sehingga diperoleh kontradiksi. Akibatnya, salah satu di antara
p, q, dan r haruslah genap. Perhatikan pula bahwa satu-satunya bilang-
an prima yang genap adalah 2. Jika r = 2, dapat dilihat bahwa tidak
ada dua bilangan prima p dan q yang memenuhi. Oleh karena itu, salah
satu di antara p dan q haruslah 2. Karena soal meminta nilai p ter-
besar yang mungkin, tanpa mengurangi keumuman bisa diasumsikan
bahwa q = 2, yang mengakibatkan p + 2 = r. Untuk mencari nilai p ter-
besar yang mungkin, kita harus mencari bilangan prima terbesar yang
merupakan anggota {1, 2, . . . , 97} dan berselisih 2 dengan bilangan prima
lainnya. Properti ini dipenuhi oleh 71 karena 73 = 71 + 2 juga merupakan
bilangan prima. Akibatnya, nilai p terbesar yang memenuhi adalah 71.
Komentar:

a) Karena 63% peserta menjawab soal ini dengan benar, soal ini bisa
digolongkan sebagai soal mudah.
b) Kunci utama dari soal ini adalah menyadari bahwa bilangan prima
kecuali 2 adalah ganjil dan memandang paritas dari p, q, dan r.
c) Sebagian besar kesalahan ditemukan karena tidak mengecek apa-
kah p, q, r ketiganya prima. Kesalahan seperti ini menghasilkan 97
sebagai jawaban. Padahal, perlu diingat bahwa 97 + 2 = 99 bukanlah
prima.

2. Jawab: 7.

ab + a + b = 5 ab + a + b + 1 = 6 (a + 1)(b + 1) = 6
ac + a + c = 9 ac + a + c + 1 = 10 (a + 1)(c + 1) = 10
bc + b + c = 14 bc + b + c + 1 = 15 (b + 1)(c + 1) = 15.

Dengan mengalikan ketiga persamaan di kanan atas, kita punya

(a + 1)2 (b + 1)2 (c + 1)2 = 6 · 10 · 15 = 900.

61
Buku KTO Matematika 1

p
Akibatnya, (a + 1)(b + 1)(c + 1) = 900 = 30. Dengan membagi hasil ini
dengan persamaan pertama, kita mendapatkan c + 1 = (a+(a1)( b+1)( c+1)
+1)( b+1) =
30
6 = 5. Dengan cara yang sama, kita mendapatkan b + 1 = 3 dan a + 1 =
2. Jumlahkan ketiga persamaan yang sudah didapat, dan kita punya
(a + 1) + (b + 1) + (c + 1) = 5 + 3 + 2 = 10. Jadi, a + b + c = 7.
Komentar:

a) Menurut statistik, 76% peserta dapat menjawab soal nomor 2 de-


ngan benar. Akibatnya, soal ini dapat dikategorikan sebagai salah
satu soal termudah.
b) Ide dari soal ini (menambahkan 1 ke kedua sisi agar ekspresi pa-
da soal bisa difaktorkan) sebenarnya cukup unik dan tidak intuitif.
Namun, ide ini termasuk rutin dan sering ditemukan dalam soal.
c) Karena ketiga persamaan tersebut simetris, cara termudah untuk
mendapatkan a, b, c adalah dengan mengalikan ketiganya. Cara la-
in untuk menyelesaikan persamaan tersebut adalah dengan mem-
bagi satu persamaan dengan persamaan lainnya.
d) Solusi yang disajikan di sini cukup tidak intuitif tetapi cantik, Sebe-
narnya, ada solusi lain yang lebih bersifat brute force (kotor, meng-
kuli, kurang elegan) tetapi mudah dipikirkan, yaitu dengan menya-
takan a dan c dalam b (bisa dalam a atau c juga), kemudian mela-
kukan substitusi untuk mendapatkan penyelesaian.

3. Jawab: 168.
Dengan menyederhanakan k +2l + m = 2k + l −2m, kita memperoleh k = l +
3m. Kemudian, dengan menyubstitusikan k = l + 3m ke persamaan 2k +
l − 2m = 2015, kita peroleh 2(l + 3m) + l − 2m = 3l + 4m = 2015. Perhatikan
bahwa ini adalah persamaan Diophantine yang dipenuhi oleh l = 4x + 1
dengan 1 < 3(4x + 1) < 2015, atau 0 ≤ x ≤ 167, dan m = 2015−3(4
4
x+1)
= 503 −
3x. Di sini, k = l + 3m pasti merupakan bilangan asli karena l dan m
keduanya adalah bilangan asli. Akibatnya, banyaknya pasangan terurut
yang memenuhi ada 167 − 0 + 1 = 168.
Komentar:

a) Karena hanya sekitar 26% peserta menjawab soal ini dengan benar,
soal ini termasuk bertingkat kesulitan sedang.

62
Pembahasan Kontes Agustus 2015

b) Kesulitan utama dalam menyelesaikan soal ini terletak pada hal


menyelesaikan persamaan Diophantine setelah substitusi dilaku-
kan. Persamaan Diophantine adalah persamaan dengan solusi bi-
langan bulat. Biasanya, kita menghadapi persamaan Diophanti-
ne yang berbentuk ax + b y = c, yang disebut juga kombinasi linear.
Kombinasi linear tersebut mempunyai 0 atau takhingga banyaknya
solusi bilangan bulat (silakan verifikasi sendiri). Kunci penyelesai-
an soal ini adalah dengan memperhatikan bahwa kita bisa mempe-
roleh solusi lainnya dengan menambah atau mengurangi masing-
masing suku dengan K PK(a, b), yakni kelipatan persekutuan ter-
kecil dari a dan b.
c) Untuk mendapatkan pengetahuan lebih lanjut mengenai persama-
an Diophantine, pembaca dapat merujuk kepada artikel Introdu-
ction to Diophantine Equations karangan Tom Davis.

4. Jawab: 3.
Jika ada bilangan positif x yang memenuhi x2 + 1x = 2, menurut AM-GM
p p
kita punya x2 + 1x ≥ 2 x, yang ekuivalen dengan 2 = x2 + 1x ≥ 2 x. Dengan
menyederhanakan pertidaksamaan tersebut, kita peroleh x ≤ 1. Perha-
tikan pula bahwa x = 1 memenuhi x2 + 1x = 1 + 1 = 2. Akibatnya, nilai a
adalah 1 dan a2 + a + 1 = 12 + 1 + 1 = 3.
Komentar:

a) Berdasarkan fakta bahwa 84% peserta mampu menjawab soal ini


dengan benar, pantas dikatakan bahwa soal ini adalah salah satu
soal termudah dari semua soal pada kontes bulan Agustus.
b) Cara termudah untuk menyelesaikan soal ini adalah dengan meng-
alikan seluruh persamaan dengan x, kemudian mencoba memfak-
torkan polinomial berderajat 3, di mana a adalah faktor terbesar-
nya. Namun, karena kita menginginkan nilai terbesar dari x yang
memenuhi persamaan tersebut, kita bisa menggunakan prinsip per-
tidaksamaan AM-GM untuk mencari nilai x terbesar yang mungkin.

63
Buku KTO Matematika 1

5. Jawab: 20.

1 1
A 2a E 2a D

1
2a G

1
2a

B C

AF DE
Perhatikan bahwa AD = CD dan ∠F AD = 90° = ∠EDC, sehingga
4F AD ∼ 4EDC. Ini mengakibatkan ∠ ADF = ∠DCE dan ∠ AFD =
∠DEC. Sekarang, karena ∠EDG = ∠ ADF = ∠DCE dan ∠DEG =
∠DEC, kita punya 4DGE ∼ 4CDE. Misalkan sisi persegi adalah a;
karena E dan F terletak di tengah, didapatkan DE = AF = 12 a. La-
lu, denganp menerapkan teorema Pythagoras pada 4CDE, kita pero-
leh CE = 25 a. Karena 4DGE ∼ 4CDE, luas mereka berbanding se-
suai dengan kuadrat panjang sisi sebandingnya, yaitu Luas 4DGE :
¡ ¢2 ³ p ´2
Luas 4CDE = DE 2 : CE 2 = 21 a : 25 a = 1 : 5. Selanjutnya, karena
luas DEG adalah 1, kita peroleh luas CDE adalah 5. Perhatikan bahwa
persegi ABCD tersusun atas 4 buah segitiga CDE. Jadi, luas persegi
ABCD adalah 4 · 5 = 20.
Komentar:

a) Statistik menunjukkan bahwa persentase peserta yang dapat men-


jawab soal ini dengan benar adalah 61%, menjadikan soal ini seba-
gai soal mudah.
b) Soal ini bisa diselesaikan dengan banyak cara. Salah satu cara lain-
nya adalah dengan menarik garis tinggi dari segitiga-segitiga terse-
but dan menggunakan fakta bahwa garis-garis tinggi dari segitiga-
segitiga sebangun berbanding sesuai dengan panjang sisi-sisi segi-
tiga tersebut. Dengan memisalkan sisi persegi sebagai a, kita ke-
mudian bisa mencari nilai a dan juga luasnya.

64
Pembahasan Kontes Agustus 2015

c) Sebagian besar kesalahan yang ditemukan yakni menuliskan ja-


waban 5, yang kemungkinan terjadi karena peserta lupa bahwa
yang ingin dicari ialah luas ABCD, dan bukan CDE.
p p
6. Jawab: 14 5
3 + 2 2 + 3 7.
Dengan mensubstitusi x pada persamaan awal dengan − x, diperoleh

f (2− x ) − x f (2 x ) = 1.

Dengan melakukan eliminasi persamaan di atas dengan persamaan


awal, dapat diperoleh
1− x
f (2 x ) = .
1 + x2
Kita dapatkan p
1 4+2 2 p
p = p = 2+2 2
f (21+ 2 ) 2
dan p
1 17 + 6 7 8 5 p
p = p = + 7.
f (23+ 7 ) 2+ 7 3 3
Jadi,
1 1 14 p 5p
p + p = +2 2+ 7.
f (21+ 2 ) f (23+ 7 ) 3 3

Komentar:

a) Soal dianulir karena jawaban bukan merupakan bilangan bulat.


b) Teknik substitusi sangat bermanfaat dalam menyelesaikan tipe soal
seperti ini.

65
Buku KTO Matematika 1

25
7. Jawab: 2 .

C1 B1

B D A1 C

Perhatikan segitiga BA 1 C 1 dan BC A sebangun dengan perbanding-


an sisi-sisi bersesuaian adalah 1 : 2. Karena A 1 C 1 = 10, kita peroleh
AC = 20. Selanjutnya, perhatikan bahwa A 1 C 1 sejajar dengan AC dan
B1 D tegak lurus A 1 C 1 . Hal ini berakibat B1 D tegak lurus AC. Kare-
na B1 titik tengah AC, kita peroleh D AC segitiga sama kaki. Karena
∠ ADC = 90°, diperoleh 4 ADC merupakan segitiga siku-siku sama kaki,
sehingga berakibat 4 ADB1 juga segitiga siku-siku sama kaki. Diperoleh
DB1 = AB1 = 10 Misalkan E adalah kaki tegak lurus dari titik B ke sisi
AC. Karena luas 4 ABC = 150, diperoleh BE = 15. Misalkan BE dan
DB1 berturut-turut memotong A 1 C 1 di titik F dan G. Perhatikan bahwa
B1 G = EF = BE 2 = 7, 5. Dengan demikian, DG = DB1 − B1 G = 2, 5. Jadi,
luas 4 A 1 C 1 B = 21 A 1 C 1 × DG = 12, 5.
Komentar:

a) Soal dianulir karena jawaban bukan merupakan bilangan bulat.


b) Diperlukan pemahaman yang baik mengenai kesebangunan untuk
menyelesaikan soal ini.

8. Jawab: 23.

66
Pembahasan Kontes Agustus 2015

Pertama-tama, kita akan meninjau nilai c. Dengan memasukkan nilai


c, kita memperoleh dua persamaan dalam modulo 13. Seperti halnya
menyelesaikan persamaan linear dengan dua variabel, terdapat tiga ke-
mungkinan:

• Persamaan tersebut tidak punya solusi, yaitu apabila ada 0 ≤ k < 13


sehingga k(a − bc2 ) ≡ ac + b (mod 13) tetapi k · 1 6≡ 4 (mod 13) (hal ini
mengakibatkan k ≡ k(a− bc2 ) ≡ ac + b 6≡ 4 (mod 13)). Dari syarat per-
tama, dengan membandingkan koefisien dari a dan b dalam modulo
13, kita peroleh k ≡ c (mod 13) dan − kc2 ≡ 1 (mod 13). Substitu-
si k ≡ c (mod 13) ke persamaan kedua akan menghasilkan − c3 ≡ 1
(mod 13) yang dipenuhi oleh c ≡ 4, 10, 12 (mod 13). Dengan men-
cocokkan ke syarat kedua, persamaan tersebut tidak mempunyai
solusi untuk c ≡ 10, 12 (mod 13).
• Persamaan tersebut punya lebih dari satu solusi, yaitu ketika kedua
persamaan ekuivalen dalam modulo 13. Hal ini terjadi ketika ada
0 ≤ k < 13 sehingga k(a− bc2 ) ≡ ac + b (mod 13) dan k ·1 ≡ 4 (mod 13),
yang mengakibatkan kedua persamaan ekuivalen dan punya 13 so-
lusi (solusi b tergantung pada a dan a bisa mengambil nilai apa sa-
ja dalam rentang 0 ≤ a < 13. Syarat pertama mengakibatkan k ≡ c
(mod 13) dan − kc2 ≡ 1 (mod 13). Substitusi k ≡ c (mod 13) ke per-
samaan kedua akan menghasilkan − c3 ≡ 1 (mod 13) yang dipenu-
hi oleh c ≡ 4, 10, 12 (mod 13). Menyesuaikan dengan syarat kedua,
persamaan tersebut mempunyai 13 solusi untuk c ≡ 4 (mod 13).
• Persamaan tersebut punya tepat satu solusi, yang terjadi untuk se-
mua nilai c lainnya; ada sebanyak 13 − 2 − 1 = 10. solusi yang demi-
kian.

Akibatnya, total banyaknya solusi yang mungkin adalah 2 · 0 + 1 · 13 + 10 ·


1 = 23.
Komentar:

a) Karena 5% peserta menjawab soal ini dengan benar, soal ini bisa
digolongkan sebagai soal sulit.
b) Sebagian besar peserta yang tidak mendapatkan jawaban yang be-
nar memberikan jawaban yang kurang dari 23 pasangan terurut.
Kesalahan ini mudah sekali dicapai apabila kita menggunakan me-

67
Buku KTO Matematika 1

tode brute force. Metode ini sangat rawan akan kekeliruan perhitu-
ngan, sehingga sebaiknya dihindari.
c) Kunci utama dari soal ini adalah menyadari bahwa menyelesaikan
persamaan dalam modulo 13 tidak jauh berbeda dengan menyele-
saikan SPLDV (sistem persaman linear dua variabel). Namun, kita
harus berhati-hati dalam melihat kasus-kasus ujung (boundary).

9. Jawab: 232.
Perhatikan bahwa ada dua kemungkinan untuk f (x). Pertama, jika
f (x) = x, (kita sebut x adalah titik tetap), dipunyai f ( f (x)) = f (x) = x.
Kedua, jika f (x) = y dan y 6= x, kita harus punya f (y) = f ( f (x)) = x. Untuk
menghitung banyaknya fungsi yang memenuhi, kita harus membagi be-
berapa kasus berdasarkan banyaknya titik tetap. Karena f (x) = y meng-
akibatkan f (y) = x, banyaknya titik tetap haruslah ganjil agar kita bisa
memasangkan sisa bilangan lainnya di daerah asal. Jika banyaknya titik
tetap adalah 1, banyaknya cara memilih 1 titik tetap adalah 71 dan cara
¡ ¢

(6)(4)(2)
memasangkan 6 bilangan lainnya adalah 2 3!2 2 , sehingga banyaknya
¡ ¢ (6)(4)(2)
keseluruhan fungsi dengan 1 titik tetap adalah 71 · 2 3!2 2 = 105. Jika
banyaknya titik tetap adalah 3, lihat bahwa banyaknya cara memilih 3
titik tetap adalah 73 dan cara memasangkan 4 bilangan lainnya adalah
¡ ¢

(42)(22)
2! , sehingga total fungsi yang memenuhi dengan 3 titik tetap adalah
¡7¢ (42)(22)
3 · 2! = 105.¡ Jika banyaknya titik tetap ada 5, banyaknya keseluruhan

fungsi adalah 5 · 1 = 21. Terakhir, jika banyaknya titik tetap ada 7, kita
hanya punya satu fungsi. Dengan menjumlahkan semuanya, diperoleh
bahwa ada 105 + 105 + 21 + 1 = 232 fungsi yang memenuhi.
Komentar:

a) Karena 2% peserta menjawab soal ini dengan benar, soal ini bisa
digolongkan sebagai salah satu soal tersulit di bagian A.
b) Sebagian peserta menjawab 1 dan 7. Jawaban ini mungkin dipero-
leh dengan menganggap satu-satunya fungsi yang mungkin adalah
f (x) = x, sementara banyaknya fungsi yang memenuhi ada 7 mung-
kin diperoleh karena peserta tidak menghitung banyaknya fungsi
melainkan menghitung f (i) = i sebagai satu fungsi. Lebih jelasnya,
peta { f (1) = 1, f (2) = 2, . . . , f (7) = 7} adalah satu fungsi yang mung-
kin, peta { f (1) = 2, f (2) = 1, f (3) = 3, f (4) = 4, . . . , f (7) = 7} adalah satu

68
Pembahasan Kontes Agustus 2015

fungsi yang mungkin, tetapi f (1) = 1 sendirinya bukanlah satu fung-


si karena kita belum menetapkan bagaimana memetakan bilangan-
bilangan lainnya.
c) Sebenarnya penyelesaian soal ini hanya menggunakan prinsip ber-
hitung yang sangat mendasar jika kita sudah tahu kunci utamanya,
yakni menemukan f (x) = x atau f (x) = y, f (y) = x. Hal ini bisa diper-
oleh dengan memisalkan f (x) = y, sehingga kita langsung menda-
patkan f ( f (x)) = f (y) = x. Menghitung hal tersebut bisa dilakukan
dengan lebih mudah menggunakan pembagian kasus berdasarkan
banyaknya titik tetap, lalu menghitung sisanya menggunakan prin-
sip kombinasi.

10. Jawab: 999.


p ¡ p ¢n
Perhatikan bahwa 9 − 69 < 1, sehingga untuk n > 0, 9 − 69 < 1. Per-
hatikan
p pula¡ bahwa p ¢jika kita mengekspansikan koefisien Binomial dari
n
(9 + 69)n + 9 − 69 , kita peroleh
b n2 c
à ! à ! à !
n n p n n p X n n−2 i p 2 i
n− i i n− i i
X X
9 ( 69) + 9 (− 69) = 2 · 9 ( 69) .
i =0 i i =0 i i =0 2i
p
Perhatikan bahwa karena semua suku yang mempunyai 69 saling
menghilangkan, suku-suku yang tertinggal hanyalah yang bernilai p ¢bu-
¡ n
lat, sehinggap ekspresi dipatas adalah bilangan bulat. Karena 9 − 69 <
n
1, a n = (9 + 69)n + p
¡ ¢
9 − 69 , dimana semua suku a n kecuali suku yang
mengandung nn 90 ( 69)n habis dibagi 9. Suku ini tidak habis dibagi 9
¡ ¢
p
jika dan hanya jika ( 69)n tidak habis dibagi 9, yaitu ketika n ≤ 2, ka-
rena kita selalu punya n genap. Akibatnya, semua a n kecuali a 2 habis
dibagi 9, dan bisa disimpulkan bahwa banyaknya suku yang habis dibagi
9 adalah 1000 − 1 = 999.
Komentar:

a) Soal ini cukup sulit dengan mempertimbangkan statistik bahwa ha-


nya 5% peserta yang menjawab soal ini dengan benar.
b) p
Kunci utama soal ini adalah bagaimana cara menghilangkan faktor
69, karena itu bukan bilangan bulat dan mempersulit kita untuk
menentukan apakah sebuah
p bilangan adalah kelipatan 9. Untuk
menghilangkan faktor 69, salah satu cara yang terpikirkan seca-
ra intuitif adalah menyeimbangkan suku tersebut dengan sekawan

69
Buku KTO Matematika 1

p p
(komplemen) dari 9 + 69, yaitu 9 − 69, yang dengan indahnya le-
bih kecil dari 1, sehingga tidak memengaruhi nilai hasil fungsi cei-
ling.
c) Selain itu, dalam hal menentukan keterbagian dengan 9, kita hanya
tinggal melakukan ekspansi suku-suku yang terlihat mengandung
bilangan kelipatan 9. Cara ini cukup rutin dilakukan dalam prak-
tiknya.
d) Hal lebih lanjut mengenai teorema Binomial untuk mengekspansi
suku-suku bisa dilihat dalam buku An Introduction of The Theory of
Number oleh Ivan Niven, Herbert Z., Hugh M.

11. Jawab: 13.

A E B
y
M
x

D F C
O

Pertama, tarik sebuah garis tegak lurus dari O terhadap AB. Karena
AB sejajar dengan CD, garis tersebut juga tegak lurus terhadap CD.
Misalkan garis ini memotong AB dan CD masing-masing di E dan F.
Kemudian, karena AO = OB = OC = OD, bisa ditunjukkan bahwa segi-
tiga AEO kongruen dengan BEO; dengan cara yang sama, bisa dibuk-
tikan bahwa segitiga CFO kongruen dengan segitiga DFO. Akibatnya,
∠ AOD = ∠BOC, yang mengakibatkan segitiga AOD kongruen dengan
segitiga BOC (sisi-sudut-sisi). Sebagai akibat, ABCD adalah trapesium
sama kaki.
Tanpa mengurangi keumuman, misalkan CD lebih dekat ke O daripada
AB. Perhatikan bahwa segitiga AMD kongruen dengan segitiga BMC

70
Pembahasan Kontes Agustus 2015

sebab ∠ AMD = ∠BMC, ∠ ADB = ∠ ACB (keduanya menghadap sisi AB),


dan AD = BC karena ABCD adalah trapesium sama kaki. Akibatnya,
segitiga AMD kongruen dengan segitiga BMC (sudut-sisi-sudut), dan
DM = MC. Dari sini diperoleh bahwa MO membagi dua garis CD. Garis
OEF sebenarnya juga melewati M. Selain itu, segitiga DF M kongruen
dengan segitiga CF M dan ∠DMF = ∠CMF = 60° sebab ∠DMC = 180° −
∠ AMD = 120°.
Sekarang, misalkanp MF = x dan ME = y. Dari segitiga DMF, bisa dida-
pat bahwa
p DF = 3x, dan dari segitiga AME juga bisa diperoleh bahwa
AE =p 3y. Akibatnya, bila kita melihat
p segitiga DOF dan AOC, dipu-
nyai ( 3y)2 + (10 + y)2 = O A 2 dan ( 3x)2 + (10 − x)2 = OD 2 . Namun, karena
O A = OD, kita dapat 3y2 + (10 + y)2 = 3x2 + (10 − x)2 .
Dengan menjabarkan kedua ruas, didapat identitas berikut: y2 + 5y =
x2 − 5x. Dari sini, kita tambahkan 25 ke kedua ruas untuk mendapatkan
5 2 5 2
¡ 45 ¢
y + 2 = x − 2 . Akibatnya, y + 2 = ± x − 52 , sehingga dipunyai x −
¡ ¢ ¡ ¢ ¡ ¢

y = 5 atau x + y = 0, yang jelas tidak mungkin.


p
Dari segitiga
p MFD dan segitiga ME A kita tahu bahwa AE p 3y dan
=
DFp= 3x. Akibatnya, selisih AB dan CD sama dengan 2 3(x − y) =
10 3. Jadi, jawabannya 10 + 3 = 13.
Komentar:

a) Pada naskah soal asli tidaklah tertulis bahwa O merupakan pusat


lingkaran. Ini membuat soal dianulir.
b) Perhatikan bahwa solusi yang kami sajikan di atas hanya melibat-
kan kesebangunan dan teorema Pythagoras. Kedua hal ini sebe-
narnya sangat ampuh dalam menyelesaikan banyak soal geometri.
Namun, diperlukan latihan keras agar kita bisa melihat intuisi da-
lam mengerjakan soal-soal ini; di
p sini, kami
p memberikan variabel
ke panjang AB dan CD (yakni 2 3x dan 2 3y), kemudian melihat
cara untuk menghubungkan kedua variabel tersebut (x dan y).
c) Soal ini juga bisa diselesaikan dengan trigonometri. Misalkan saja
∠ ADB = α dan radius lingkarannya adalah r. Kita bisa dapatkan
ekspresi AB − CD dalam r dan α, yang bisa kita jabarkan. Dengan
mengobservasi segitiga DOM, kita juga bisa mendapatkan hubu-
ngan lain antara r dan α. Namun, bila kita tidak terbiasa, cara ini

71
Buku KTO Matematika 1

akan sangat menghabiskan waktu; dan akan menjadi masalah pula


bila Anda lupa akan identitas-identitas trigonometrinya.

12. Jawab: 44.


Dalam soal ini, kita ingin mencari banyaknya permutasi dari
(a 1 , a 2 , . . . , a 10 ) yang memenuhi |a 1 − 1| + |a 2 − 2| + · · · + |a 10 − 10| = 4.
Lihat bahwa untuk semua x, | x| ≥ 0. Akibatnya, setiap suku di se-
belah kiri pada persamaan di atas lebih besar atau sama dengan 0,
di mana semua suku akan sama dengan 0 jika a i = i. Langkah se-
lanjutnya adalah menemukan berapa banyak cara kita bisa menyata-
kan 4 sebagai jumlah bilangan-bilangan asli, yang adalah 4, karena
4 = 3 + 1 = 2 + 2 = 2 + 1 + 1 = 1 + 1 + 1 + 1. Untuk masing-masing kemung-
kinan, kita akan mencari banyaknya permutasi (a 1 , a 2 , . . . , a 10 ) yang
memenuhi.

• Jika 4 hanya terdiri dari satu suku a i dengan a i − i = 4, kita mene-


mukan sebuah kontradiksi karena a i = i + 4, dan ini mengakibatkan
a i+4 6= i + 4, sehingga a i+4 − (i + 4) > 0.
• Jika 4 = 3 + 1, haruslah ada i sehingga |a i − i | = 3 dan tepat satu j
lainnya dengan |a j − j | = 1. Namun, karena terdapat tepat 2 nilai i
dengan a i − i > 0, dalam kasus ini kita harus punya |a i − i | = |a i −
a j | = |a j − j |, dan terjadi kontradiksi.
• Jika 4 = 2 + 2, kita dapatkan hasil yang sama dengan kasus se-
belumnya, tetapi hal ini mungkin dan dipenuhi ketika ada tepat
2 nilai i dengan a i 6= i dan |a i − i | = |a i − a j | = |a j − j | = 2 atau
a i = i + 2 dan a i+2 = i. Akibatnya, pasangan (i, j) yang mungkin
adalah (1, 3), (2, 4), . . . , (8, 10), ada 8 pasang.
• Jika 4 = 2 + 1 + 1, kita harus mencari 3 bilangan berurutan sehingga
a i = i + 2, a i+1 = i dan a i+2 = i + 1. Pasangan (i, i + 1, i + 2) yang
memenuhi adalah (3, 1, 2), (4, 2, 3), . . . , (10, 8, 9), sehingga ditemukan
8 kemungkinan.
• Jika 4 = 1 + 1 + 1 + 1, kita harus mencari i dan j yang berbeda dengan
j > i + 1 sehingga a i = i + 1, a i+1 = i, a j = j + 1, a j+1 = j. Jika i = k,
ada 9 − (k + 2) + 1 nilai yang mungkin untuk j. Akibatnya, banyaknya
pasangan (i, j) yang memenuhi adalah 7k=1 8 − k = 28.
P

Jadi, banyaknya permutasi yang memenuhi syarat pada soal adalah 8 +


8 + 28 = 44.

72
Pembahasan Kontes Agustus 2015

Komentar:

a) Mengingat hanya 2% peserta menjawab soal ini dengan benar, soal


ini bisa digolongkan sebagai soal tersulit di bagian A.
b) Dalam soal ini perlu diperhatikan bahwa | x| ≥ 0, sehingga pilihan
kita terbatas di sini.
c) Sebagian besar peserta mendapatkan jawaban yang terlalu besar
dalam soal ini. Hal ini kemungkinan disebabkan karena melakukan
kelebihan penghitungan akibat tidak memperhatikan nilai mutlak.

13. Jawab: soal salah.


Perhatikan bahwa 2a tidak mungkin habis dibagi 11 sebab satu-satunya
faktor prima dari 2a adalah 2, dan tidak meliputi 11. Akibatnya, tidak
ada bilangan habis dibagi 11 yang bisa dinyatakan dalam bentuk 2a +
11b, sehingga N tersebut tidak ada.
Komentar:

a) Bila pertanyaannya diganti menjadi “sehingga N tidak habis dibagi


11”, nilai N tersebut ada, yakni 501. Perhatikan bahwa 20 , 21 , . . . , 29
masing-masing habis dibagi 1, 2, 4, 8, 5, 10, 9, 7, 3, dan 6. Hal ini
berarti bahwa semua bilangan yang kongruen 1 (mod 11) dan le-
bih besar atau sama dengan 20 pasti bisa dinyatakan dalam bentuk
2a + 11b, semua bilangan yang kongruen 2 (mod 11) dan lebih besar
atau sama dengan 21 bisa dinyatakan dalam bentuk 2a + 11b, dan
seterusnya.
Akibatnya, semua bilangan yang lebih besar atau sama dengan
29 = 512 pasti bisa dinyatakan dalam bentuk 2a + 11b ini. Perhati-
kan bahwa 502, 503, . . . , 511 kecuali 506 bisa juga dinyatakan dalam
bentuk 2a + 11b karena semuanya tidak habis dibagi 11 ataupun
bersisa 6 jika dibagi 11. Akibatnya, bilangan terbesar yang tidak
bisa dinyatakan adalah 501, yang kongruen 6 (mod 11), karena bi-
langan terkecil a sehingga 2a ≡ 6 (mod 11) adalah 9.

73
Buku KTO Matematika 1

14. Jawab: 24.

O
A

B C

Perhatikan bahwa B, I, dan O kolinear karena titik pusat lingkaran


singgung luar dan lingkaran dalam keduanya terletak pada garis bagi
∠ ABC. Misalkan ∠BAC = a, ∠ ABC = b, dan ∠ ACB = c. Lalu, lihat bah-
wa ∠ AOB = 180° − ∠ ABO − ∠BAO = 180° − 2b − 90° − a2 + a = 2c . Dari sana
¡ ¢

diperoleh bahwa ∠ ABO = 2b = ∠ IBC dan ∠ AOB = 2c = ∠BCI, sehing-


AB BI
ga 4 ABO ∼ 4 IBC. Hal ini mengakibatkan BO = BC , di mana BI = 12,
BC = 15, dan BO = BI + IO = 12 + 18 = 30. Akibatnya, AB = 3015 ·12
= 24.

Komentar:

a) Soal ini adalah soal tersulit di bagian A karena tidak ada peserta
yang menjawab soal ini dengan benar.
b) Hal pertama yang perlu diperhatikan adalah bahwa I dan O kedua-
nya terletak di garis bagi ∠ ABC. Hal ini bisa dilihat dengan mena-
rik radius-radius lingkaran dalam dan lingkaran singgung luar dari
segitiga tersebut yang tegak lurus dengan BA dan BC, juga mem-
perhatikan bahwa dua buah segiempat yang terbentuk kongruen.
Soal-soal dan teorema yang serupa bisa dicari dalam buku Kapita
Selekta Geometri oleh Nugroho, Teddy Widarto, Eka Susanto.
c) Hal lain yang penting dari soal ini adalah mencari segitiga-segitiga
sebangun.

74
Pembahasan Kontes Agustus 2015

d) Langkah pertama untuk menyelesaikan soal geometri adalah meng-


gambar dengan baik. Pak Soewono, dosen Universitas Telkom, Ban-
dung sekaligus salah satu anggota pembina tim Indonesia dalam
International Mathematical Olympiad, selalu menekankan bahwa
menggambar dengan rapi adalah langkah awal yang harus dilaku-
kan dalam mengerjakan soal geometri. Pembaca disarankan untuk
mengaplikasikan saran dari beliau demi keuntungan pembaca sen-
diri.

75
Buku KTO Matematika 1

Bagian B

1. Jawab: tidak dapat.


Tinjau banyak singa sakit perut, gigi, kepala dalam mod 3. Perhatikan
bahwa mula-mula banyak masing-masing penyakit adalah {0, 1, 2} dalam
mod 3. Perhatikan bahwa jika 2 singa bertemu, banyak masing-masing
penyakit tetap {0, 1, 2} dalam mod 3 (silakan verifikasi sendiri). Padahal,
kondisi yang ingin dituju adalah {0, 0, 0} mod 3, yaitu ketika 1 penyakit
menjangkiti 75 singa dan kedua penyakit lainnya 0 singa. Akibatnya, ki-
ta tidak bisa mempertemukan singa-singa tersebut sedemikian sehingga
semua singa terjangkit penyakit yang sama.
Komentar:

a) Soal ini menggunakan prinsip invarian, yaitu melihat kondisi yang


bernilai tetap setelah beberapa tahap operasi dilakukan. Di sini,
yang menjadi invarian adalah selisih dari masing-masing jenis pe-
nyakit dalam modulo 3. Hal yang paling menarik dari soal-soal in-
variant adalah mencari kondisi apa yang selalu tetap dan bisa dija-
dikan patokan setelah beberapa operasi.
b) Untuk mendapatkan pengetahuan lebih lanjut mengenai prinsip in-
varian, pembaca dapat merujuk kepada buku Problem Solving Stra-
tegies karangan Arthur Engel. Dalam buku ini, ada satu bab yang
memuat banyak soal menarik mengenai prinsip invarian.
c) Selain menggunakan cara ini, kita dapat juga menggunakan prinsip
sistem persamaan linear untuk membuktikan tidak adanya solusi.
Hal ini cenderung lebih rutin dengan menganggap singa dengan pe-
nyakit 1 dan penyakit 2 bertemu x kali, singa dengan penyakit 2
dan penyakit 3 bertemu y kali, dan singa dengan penyakit 1 dan pe-
nyakit 3 bertemu z kali, sehingga banyaknya penyakit 1 pada akhir
operasi ada 24 + 2y − x − z, penyakit 2 ada 25 + 2z − x − y, dan penyakit
3 ada 26 + 2x − y − z. Dengan menyamakan ketiga persamaan ter-
sebut dengan {0, 0, 75} dalam urutan yang berbeda-beda, kita tidak
akan memperoleh solusi bilangan bulat. Dengan demikian dapat
disimpulkan bahwa kondisi pada soal tidak mungkin tercapai.
d) Kelebihan dari cara kedua adalah apabila jawaban dari soal terse-
but adalah “ada”, kita bisa langsung menemukan nilai x, y, z yang
memenuhi. Di sisi lain, penggunaan prinsip invariant hanya akan

76
Pembahasan Kontes Agustus 2015

memberikan kita jawaban bahwa hal tersebut mungkin atau tidak


mungkin dicapai. Namun, cara kedua juga memiliki kekurangan,
yakni dibutuhkannya lebih banyak kalkulasi saat melakukan pe-
nyamaan ketiga persamaan dengan {0, 0, 75}.

2. Jawab: (c) 45°.

Γ1

Y
Γ2
X

P M Q

a. Kita tahu bahwa ∠ MP X = ∠PY X , menurut Alternate Segment the-


orem. Lalu, karena ∠PY X = ∠PY M dan ∠P M X = ∠Y MP, dipu-
MP
nyai 4 MP X ∼ 4 MY P menurut kriteria sudut-sudut. Jadi, M X =
MY 2
MP , dan MP = M X × MY .
b. Dengan teorema Secant-Tangent, diperoleh MP 2 = M X × MY dan
MQ 2 = M X × MY . Jadi, MP = MQ.
PQ 2QY
c. Dari soal (b), didapat MQ = 2 = 2 = QY . Karena QY adalah
diameter Γ2 , pusat dari Γ2 ada di QY . Akibatnya, ∠ MQY = 90°,
mengakibatkan bahwa 4 MQY adalah segitiga siku-siku sama kaki.
Akibatnya, ∠QY M = 45°. Namun, ∠QY M sama dengan setengah
busur Q X , yang sama dengan ∠PQ X . Jadi, ∠PQ X = 45°.

77
Buku KTO Matematika 1

Komentar:

a) Penyelesaian soal ini sebenarnya sangat sederhana, yakni dengan


menggunakan kesebangunan antarsegitiga.
b) Untuk membuktikan bahwa dua segitiga adalah sebangun, kita ha-
rus memiliki setidaknya dua pasang sudut yang sama.
c) Observasi kunci dari soal ini ialah bahwa sudut keliling busur ada-
lah setengah dari sudut pusat busur. Pembaca dipersilakan untuk
memverifikasi hal tersebut dengan teknik angle chasing (mencari
nilai sudut-sudut dan menemukan relasi antarsudut).
d) Soal ini sebenarnya sangat sederhana dengan menggunakan kese-
bangunan segitiga. Untuk membuktikan dua segitiga sebangun ki-
ta harus selalu ingat kita butuh setidaknya dua pasang sudut yang
sama. Soal ini akan lebih mudah bila kita tahu bahwa sudut keliling
dalam satu busur adalah setengan dari sudut pusat busur tersebut,
contohnya ∠ X Y Q sama dengan setengah busur Q X . Hal ini juga
berlaku untuk titik singgung seperti ∠ MQ X sama dengan setengah
busur Q X . Hal-hal lebih lanjut mengenai lingkaran bisa dilihat da-
lam dokumen pembelajaran Power of a Point oleh Yufei Zhao. Beli-
au juga menulis banyak seri geometri lainnya yang sangat menarik
dan menantang.
e) Kesalahan umum yang dilakukan oleh para peserta dalam menja-
wab soal ini adalah tidak menuliskan argumen yang jelas mengenai
alasan kedua segitiga sebangun. Walaupun alasannya cenderung
jelas, penulisan yang runtut dan tepat tetap dibutuhkan. Jika ti-
dak, pengurangan poin bukanlah hal yang tidak mungkin terjadi.

3. Jawab: ada.
Untuk setiap bilangan asli n, kita menggunakan notasi s(n) untuk me-
nyatakan hasil jumlah digit-digit n. Perhatikan bahwa s(2015) = 8 dan
s(2 · 2015) = s(4030) = 7. Selanjutnya, kita punyai lema berikut:
Lema. Diberikan sebuah bilangan asli n. Jika a dan b merupakan bilang-
an asli yang habis dibagi n, bilangan ab yang diperoleh dari konkatenasi
a dan b juga habis dibagi n.
Bukti. Misalkan a = kn dan b = ln untuk suatu bilangan asli k dan l.
Misalkan banyak digit b adalah m. Oleh karena itu, kita punyai bah-

78
Pembahasan Kontes Agustus 2015

wa ab = 10m a + b = 10m kn + ln + (10m k + l)n, yang juga habis dibagi n.


Terbukti.

Sekarang, kita bisa peroleh bahwa bilangan

2015201520152015201520152015 |403040304030
{z . . . 40304030}
281 buah 4030

memiliki jumlah digit 6· 8 + 281· 7 = 2015; dan berdasarkan lema, bilang-


an tersebut habis dibagi 2015.

Komentar:

a) Sebagian besar kesalahan dalam soal ini adalah menganggap bah-


wa konkatenasi ab dari dua bilangan a dan b yang habis dibagi c
sudah pasti habis dibagi c. Hal ini sebenarnya tidak jelas terlihat
dan perlu dibuktikan terlebih dahulu.

b) Kunci utama soal ini sebenarnya adalah memandang konkatenasi


ab.

4. Bukti.
P∞ 1
Misalkan S = i =1 F i . Akibatnya,

X∞ 1 X∞ 1 X∞ 1
S −2 = = = .
i =3 F i i =1 F i +2 i =1 F i +1 + F i

Dengan teorema AM-HM berbobot, diperoleh:

1+φ 1 1 φ
µ ¶
≤ +φ·
F i + φ · Fφi+1 1 + φ Fi F i+1

1 1 1 φ2 1 1 φ2
µ ¶ µ ¶
≤ + = + .
F i + F i+1 (1 + φ)2 F i F i+1 φ4 F i F i+1

79
Buku KTO Matematika 1

Akibatnya,

X 1
S −2 =
i =1 F i +1 + F i
∞ 1 µ 1 φ2

X
≤ +
i =1 φ
4 F F i+1
i

φ4 (S − 2) ≤ S + φ2 (S − 1)
(φ4 − φ2 − 1)S ≤ 2φ4 − φ2
2φ 4 − φ 2 φ4
S≤ = .
φ4 − φ2 − 1 2

Kesamaan terjadi jika F i+1 = φF i untuk setiap i ∈ N. Namun, ini tidak


φ4
mungkin karena F i+1 rasional, sedangkan φF i tidak. Akibatnya S < 2 .
Komentar:

a) Soal ini adalah soal tersulit di bagian B; hanya ada 1 peserta yang
menjawab soal ini dengan benar sepenuhnya.
b) Soal ini menggunakan teorema AM-HM (rataan aritmetik - rataan
harmonik) berbobot yang mungkin belum banyak diketahui. Teo-
rema ini sebenarnya tidak jauh berbeda dengan teorema AM-HM
biasa; versi berbobot dari ketaksamaan ini hanya sebuah penguat-
an. Hal lebih lanjut mengenai ketaksamaan ini bisa dilihat dalam
jurnal A Note on the Weighted Harmonic-Geometric-Arithmetic Me-
ans Inequalities oleh Gerard Maze, Urs Wagner.

80
4 Kontes September 2015

4.1 Soal-Soal
Bagian A

Tuliskan jawaban akhir setiap soal di lembar jawaban. Setiap soal bernilai 1
angka. Jawaban Anda harus berupa bilangan bulat dari 0 sampai 999.

1. (1004) Sebuah kue berbentuk persegi panjang akan dipotong beberapa


kali secara horizontal dan beberapa kali secara vertikal dengan jarak
setiap dua potongan berdekatan adalah sama (sehingga menghasilkan
potongan-potongan kue yang sama besar). Kue ini dipotong menjadi te-
pat 2015 potongan kue. Tentukan banyak potongan minimal yang diper-
lukan.

2. (1011) Tentukan tiga digit terakhir dari

1 + n2 + n4
1 + n + n2
untuk n = 2015.

3. (1018) Diadakan sebuah turnamen yang diikuti oleh 74 peserta. Pada


mulanya, setiap peserta memiliki kesempatan 3 kali kalah. Turnamen
ini dilakukan dengan mempertandingkan dua buah peserta yang masih
bertahan di turnamen; di setiap pertandingan, tepat satu peserta kalah.
Jika seorang peserta telah kalah tiga kali, ia keluar dari turnamen. Dike-
tahui bahwa setelah mengadakan tepat N pertandingan, hanya tersisa
1 orang pemain dan ia dinobatkan sebagai pemenang. Tentukan nilai
terkecil yang mungkin untuk N.

81
Buku KTO Matematika 1

4. (1025) Diketahui bahwa A(20, 10) dan C(−18, 2) adalah dua buah titik
sehingga AC merupakan diagonal dari suatu persegi ABCD. Titik B
dan D berturut-turut memiliki koordinat (− p, q) dan (r, − s) di mana p, q,
r, dan s merupakan bilangan asli. Hitunglah p + q + r + s.

5. (1032) Diketahui bahwa terdapat sebanyak n garis pada bidang di mana


setiap garis memotong tepat 20 garis lainnya. Tentukan hasil jumlah
semua kemungkinan nilai n.

6. (1039) Tentukan banyaknya pasangan bilangan asli (a, b) demikian se-


hingga a × b = 1515 .

7. (1046) Pada sebuah pesta, akan hadir 1000 tamu. Setiap tamu menge-
nal tepat 40 tamu lainnya dengan asumsi bahwa jika A mengenal B, B
mengenal A. Diketahui bahwa dari n tamu pertama yang hadir, tidak
ada dari mereka yang saling mengenal. Tentukan nilai terbesar n yang
mungkin.

8. (1053) C 1 dan C 2 adalah dua buah lingkaran yang bersinggungan luar


dengan diameter AB dan BC, titik pusat D dan E, berturut-turut. Mi-
salkan F adalah titik potong antara garis singgung C 2 dari A dan garis
singgung C 1 dari C di mana kedua garis singgung
p berada pada sisi yang
sama terhadap garis AC. Jika BD = BE = 2, luas segitiga AFC dapat
p
ditulis dalam bentuk m n di mana m dan n adalah bilangan asli dan n
tidak habis dibagi oleh kuadrat suatu bilangan prima. Hitunglah m + n.

9. (1060) Barisan a 1 , a 2 , . . . didefinisikan dengan a k = (k2 + k + 1)k! untuk


k = 1, 2, . . . . Misalkan

1 + a 1 + a 2 + · · · + a 24 m
=
a 25 n

dengan m, n adalah bilangan asli yang relatif prima. Hitunglah m + n.


p
10. (1067)pDiberikan segitiga ABC demikian sehingga AB = 1, AC = 2, dan
BC = 3. Diberikan titik P dan Q demikian sehingga PB = QB = 1, PC =
PA p p
QC = 2, dan P A > Q A. Nilai Q A dapat ditulis dalam bentuk m + n di
mana m dan n merupakan bilangan asli yang tidak habis dibagi kuadrat
suatu bilangan prima. Hitunglah m + n.

82
Soal-Soal Kontes September 2015

11. (1074) Untuk setiap bilangan asli n, misalkan t(n) menyatakan pembagi
ganjil terbesar dari n. Sebagai contoh, t(48) = 3 dan t(49) = 49. Hitunglah
tiga digit terakhir dari hasil jumlah

128
X
t(k).
k=1

12. (1081) Diketahui bahwa untuk setiap bilangan real x, berlaku ax2 + bx +
c ≥ 0 di mana a, b, c merupakan bilangan real yang tidak semuanya 0
dan a < b. Tentukan nilai terkecil yang mungkin dari

a + 5b + 3c
.
b−a

13. (1088) Misalkan E, F, G adalah titik-titik pada sisi AB, BC, CD dari
sebuah persegi panjang ABCD,
p berturut-turut,
p demikian sehingga BF =
FG, ∠FGE = 90°, BC = 54 3, dan EF = 5. Panjang BF dapat dituliskan
p p
a− b
dalam bentuk c di mana a, b, c merupakan bilangan asli dengan a
dan b tidak habis dibagi kuadrat suatu bilangan prima. Hitunglah nilai
a + b + c.

14. (1095) Misalkan ¹ º ¹ 2º ¹ 2015 º


7 7 7
N= + +···+
5 5 5
dan misalkan M adalah sisa pembagian 72016 oleh N. Tuliskan tiga digit
terakhir dari M.

83
Buku KTO Matematika 1

Bagian B

Tuliskan jawaban beserta langkah pekerjaan Anda pada lembar jawaban.


Selain jawaban akhir, Anda perlu menuliskan argumentasi atau langkah-
langkah untuk memperoleh jawaban akhir tersebut. Gunakan halaman yang
berbeda untuk setiap soal yang berbeda. Setiap soal bernilai 7 angka.

1. (1116) Pada papan catur berukuran 8 × 8, beberapa buah gajah akan di-
letakkan. Dua buah gajah dikatakan saling menyerang jika garis yang
menghubungkan kedua gajah sejajar dengan diagonal papan catur. Pa-
ling banyak, berapa gajahkah yang dapat diletakkan pada papan catur
tersebut sehingga tidak ada dua gajah yang saling menyerang?

2. (1123) Tentukan semua pasangan terurut bilangan real tak nol (a, b, c)
yang memenuhi ab + bc + ca = 1 dan

1 1 1
µ ¶ µ ¶ µ ¶
3 a+ =4 b+ =5 c+ .
a b c

3. (1130) Diberikan suatu bilangan asli n. Suatu pembagi positif d dari n


disebut spesial jika d + 1 juga merupakan pembagi positif dari n. Tun-
jukkan bahwa banyaknya pembagi spesial dari n tidak lebih dari sete-
ngah banyaknya pembagi positif dari n.

4. (1135) Diberikan segitiga lancip ABC. Misalkan D dan E berturut-turut


merupakan kaki garis tinggi dari titik A dan B. Misalkan pula A 0 dan
B0 merupakan titik tengah dari AD dan BE, berturut-turut. Misalkan
garis CD memotong BE di X . Misalkan pula CE memotong AD di Y .
Tunjukkan bahwa terdapat lingkaran yang melalui A 0 , B0 , X , dan Y .

84
Pembahasan Kontes September 2015

4.2 Pembahasan
Bagian A

1. Jawab: 94.
Misalkan dilakukan pemotongan vertikal sebanyak m kali dan pemo-
tongan horizontal sebanyak n kali. Dengan demikian, akan dihasilk-
an sebanyak (m + 1) × (n + 1) = 2015 potongan kue. Kita sekarang akan
mencari nilai terkecil yang mungkin untuk m + n. Perhatikan bahwa
2015 = 5 × 13 × 31. Dengan asumsi m ≤ n (tanpa mengurangi keumum-
an), ada sebanyak 4 kemungkinan yang perlu diperiksa:

a) m + 1 = 1 dan n + 1 = 5 × 13 × 31, maka m + n = 2014,


b) m + 1 = 5 dan n + 1 = 13 × 31, maka m + n = 406,
c) m + 1 = 13 dan n + 1 = 5 × 31, maka m + n = 106, dan
d) m + 1 = 31 dan n + 1 = 5 × 13, maka m + n = 94.

Akibatnya, banyak potongan minimal yang dibutuhkan adalah 94.


Komentar:

a) 61% peserta mampu menjawab soal ini dengan benar, menunjukkan


bahwa soal ini tergolong mudah.
b) Perhatikan bahwa 31 dan 65 adalah dua faktor dari 2015 yang hasil
kalinya 2015 dan selisihnya sekecil mungkin, Hal ini terjadi karena
4ab = (a + b)2 − (a − b)2 ; untuk meminimalkan nilai (a + b)2 dengan
ab yang tetap, nilai (a − b)2 haruslah diminimalkan.
c) Teknik yang serupa dapat digunakan untuk mencari potongan mi-
nimal yang dibutuhkan untuk membagi kue menjadi n potongan,
untuk setiap bilangan asli n.

2. Jawab: 211.
Perhatikan bahwa 1 + n2 + n4 = 1 + 2n2 + n4 − n2 = (1 + n2 )2 − n2 = (1 + n2 −
n)(1 + n2 + n), sehingga ekspresi pada soal bisa disederhanakan menjadi
n2 − n +1. Perhatikan bahwa 1−2015+20152 = 4058211. Konsekuensinya,
tiga digit terakhirnya adalah 211.
Komentar:

85
Buku KTO Matematika 1

a) Fakta bahwa 76% peserta mampu menjawab soal ini dengan benar
menunjukkan bahwa soal ini tidaklah sulit. Kenyataanya, soal ber-
tipe demikian muncul dengan frekuensi yang cukup besar di olim-
piade matematika.
b) Observasi yang menjadi kunci dalam penyelesaian soal ini adalah
pemfaktoran bentuk n4 + n2 + 1, yang sekilas terlihat tidak bisa di-
faktorkan menjadi polinomial dengan koefisien bilangan bulat. Tek-
nik pemfaktoran seperti ini penting untuk dikuasai mengingat ba-
nyaknya soal olimpiade matematika yang menggunakan teknik ma-
nipulasi demikian untuk memfaktorkan bentuk-bentuk aljabar.
c) Faktorisasi seperti pada solusi juga dapat diperoleh dengan meman-
dang fakta bahwa n3 − 1 = (n − 1)(n2 + n + 1) dan n2 − 1 = (n − 1)(n + 1).

3. Jawab: 219.
Pandang bahwa banyaknya pertandingan yang perlu dilakukan setidak-
nya adalah sebanyak 3 kali banyak peserta yang keluar dari pertanding-
an, yaitu N ≥ 3 × 73 = 219. Nilai ini dapat dicapai dengan memilih satu
pemain dan membuatnya menggugurkan setiap 73 pemain lainnya. Jadi,
nilai terkecil yang mungkin untuk N adalah 219.
Komentar:

a) 41% peserta mampu menjawab soal ini dengan benar, menyugestik-


an bahwa soal ini tergolong bertingkat kesulitan sedang.
b) Perhatikan bahwa dalam turnamen di atas, nilai N ekuivalen de-
ngan banyaknya kekalahan dalam turnamen tersebut. Akibatnya,
dengan logika yang serupa dapat disimpulkan bahwa banyaknya
turnamen terbesar yang mungkin dilakukan adalah 221 (pemain
terakhir tidak boleh kalah tiga kali).
c) Dalam mencari nilai minimum dari suatu hal, cara yang biasa dila-
kukan adalah menemukan suatu batas bawah (dalam hal ini 219),
kemudian membuktikan bahwa nilai tersebut mungkin untuk dica-
pai. Cara yang sama bisa diaplikasikan untuk mencari nilai maksi-
mum.

86
Pembahasan Kontes September 2015

4. Jawab: 46.

B(−3, 25) y

A(20, 10)
P(1, 6)
C(−18, 2)
x

D(5, −13)

Titik pusat persegi adalah titik tengah AC, yaitu P(1, 6). Dengan mero-
tasi A dengan pusat rotasi P sebesar 90°, diperoleh bahwa titik B ada-
lah (−3, 25). Dengan merotasi C dengan pusat rotasi P sebesar 90°, di-
peroleh bahwa titik D adalah (5, −13). Dengan demikian, p + q + r + s =
3 + 25 + 5 + 13 = 46.
Komentar:

a) Fakta bahwa 52% peserta mampu menjawab soal ini dengan be-
nar memberikan kesimpulan bahwa soal ini tergolong sedang dalam
tingkat kesulitan.
b) Kunci dari penyelesaian soal ini adalah mencari koordinat titik ha-
sil rotasi. Materi tersebut sudah dipelajari di tingkat sekolah mene-
ngah atas (SMA).
c) Pendekatan lain yang bisa dimanfaatkan adalah pendekatan alja-
bar; namun, cara ini cenderung rumit dan melibatkan cukup ba-
nyak manipulasi aljabar.

5. Jawab: 162.
Misalkan n garis tersebut dibagi menjadi k buah kelompok berdasarkan
hubungan kesejajaran: dua garis berada dalam sebuah kelompok jika
dan hanya jika kedua garis tersebut sejajar. Akibatnya, a 1 + a 2 +· · ·+ a k =
n, dengan a t menyatakan banyaknya garis pada kelompok t. Perhati-
kan bahwa setiap garis pada kelompok i berpotongan dengan tepat n − a i

87
Buku KTO Matematika 1

garis yang lainnya, sehingga n − a i = 20, atau a i = n − 20, untuk seti-


ap i. Sebagai konsekuensi, n = a 1 + a 2 + · · · + a k = k(n − 20), sehingga
k = n−n20 = 1 + n−
20
20 . Dari sini kita punya bahwa n − 20 harus merupakan
faktor positif dari 20. Perhatikan tabel berikut:

n − 20 1 2 4 5 10 20
n 21 22 24 25 30 40

Akibatnya, bisa kita simpulkan bahwa jumlah semua kemungkinan nilai


n adalah 21 + 22 + 24 + 25 + 30 + 40 = 162.
Komentar:

a) Hanya 6% peserta mampu menjawab soal ini dengan benar, per-


sentase terendah dari semua soal isian singkat bulan ini. Hal ini
menunjukkan bahwa masih banyak peserta yang belum familiar de-
ngan teknik pengerjaan soal yang demikian.
b) Observasi yang sangat penting dalam menjawab soal ini ialah bah-
wa kita bisa mengelompokkan garis-garis berdasarkan hubungan
kesejajaran. Pengamatan ini didasari oleh fakta bahwa dua garis
tak berimpit yang berpotongan pasti tidak sejajar, dan dua garis
tak berimpit yang sejajar pasti tidak berpotongan.

6. Jawab: 256.
Karena a dan b merupakan bilangan asli yang hasil kalinya 1515 , pem-
bagi prima dari a dan b haruslah anggota himpunan {3, 5}. Akibatnya,
kita bisa menuliskan a = 3a0 5a1 dan b = 3b0 5b1 di mana a 0 , b 0 , a 1 , b 1 me-
rupakan bilangan cacah. Dengan demikian, agar ab = 1515 , haruslah
a 0 + b 0 = 15 dan a 1 + b 1 = 15. Banyaknya tupel bilangan cacah (x, y) yang
memenuhi x + y = 15 adalah 16 (silakan verifikasi sendiri). Jadi, banyak-
nya tupel (a 0 , b 0 , a 1 , b 1 ) yang memenuhi a 0 + b 0 = 15 dan a 1 + b 1 = 15
adalah 16 × 16 = 256. Dengan kata lain, banyaknya bilangan asli (a, b)
yang memenuhi ab = 1515 adalah 256.
Komentar:

a) Sebanyak 59% peserta mampu menjawab soal ini dengan benar, se-
hingga soal ini bisa dikatakan mudah.
b) Pengetahuan dasar akan teorema faktorisasi bilangan bulat sudah
cukup untuk mendapatkan jawaban benar pada soal ini.

88
Pembahasan Kontes September 2015

7. Jawab: 500.
Akan kita tunjukkan bahwa n ≤ 500. Misalkan A adalah himpunan n
orang pertama dan B himpunan sisa orang yang menghadiri pesta terse-
but. Dari soal kita tahu bahwa terdapat 40n buah hubungan pengenalan
dari A ke B. Kita juga memiliki fakta bahwa himpunan B hanya bo-
leh mempunyai 40(1000 − n) pengenalan karena setiap dua orang pada
himpunan A tidak saling mengenal. Akibatnya, 40n ≤ 40(1000 − n) ⇐⇒
n ≤ 1000 − n ⇐⇒ n ≤ 500. Contoh konfigurasi dengan n = 500 sebagai
berikut: orang ke-i yang hadir mengenal orang ke-(i + 500), (i + 501), . . . ,
(i + 539), dengan orang ke-1000 + i menunjukkan orang yang sama de-
ngan orang yang ke-500 + i. Jadi, nilai n terbesar yang mungkin adalah
500.
Komentar:

a) 13% peserta mampu menjawab soal ini dengan benar, menandakan


bahwa soal ini tergolong cukup sulit.
b) Pengamatan yang esensial dalam proses menjawab soal ini adalah
menemukan suatu batas bawah. Batas bawah tersebut dapat dicari
dengan melakukan percobaan untuk banyak tamu yang lebih kecil,
lalu dilakukan generalisasi untuk kasus yang lebih besar.
c) Dalam mencari nilai maksimum dari suatu hal, cara yang biasa di-
lakukan adalah menemukan suatu batas bawah (dalam hal ini 500),
kemudian membuktikan bahwa nilai tersebut mungkin untuk dica-
pai. Cara yang sama bisa diaplikasikan untuk mencari nilai mini-
mum.

89
Buku KTO Matematika 1

8. Jawab: 4.

M1
M2
F

A B C
D r G E R

Misal kedua garis singgung yang dimaksud adalah AM2 and CM1 , dan
misal G adalah proyeksi titik F pada AC. Jika jari-jari lingkaran dengan
pusat D adalah r danp jari-jari lingkaran dengan pusat E adalah
p R, kita
p
GA AM2 (2 r +R )2 −R 2 2 r 2 +Rr GC CM1 (2R + r )2 − r 2
peroleh FG = M2 E = R = R dan FG = M2 D = r =
p
2 R 2 +Rr
r . Dengan mengutilisasi fakta G A + GC = AC = 2(R + r) dan men-
jumlahkan persamaan di atas, kita mendapatkan
p p p
2(R + r) p R R+r r Rr R + r
=2 R+r ⇐⇒ FG = p p .
FG Rr R R+r r
AC Rr (R + r )3/2
p
Sekarang, [ACF] = 2 · FG = (R + r)FG = R 3/2 + r 3/2
. Karena R = r = 2,
kita memperoleh
p
2 · (2 2)3/2 p
[ACF] = p = 23/2 = 2 2,
2 · ( 2)3/2
sehingga m + n = 2 + 2 = 4.
Komentar:

a) Statistik menunjukkan bahwa 35% peserta mampu menjawab so-


al ini dengan benar. Akibatnya, soal ini bisa digolongkan sebagai
cukup sulit.

90
Pembahasan Kontes September 2015

b) Tidak diperlukan pengetahuan geometri yang tinggi dalam men-


jawab soal ini. Observasi yang penting adalah bahwa kita dapat
menghitung panjang segmen-segmen pada gambar dalam R dan r.

9. Jawab: 676.
Perhatikan bahwa a k = (k + 1)2 k! − k2 (k − 1)!. Jadi, a 1 + a 2 + · · · + a 24 =
252 · 24! − 1. Dari situ diperoleh

1 + a 1 + a 2 + · · · + a 24 252 · 24! 25
= 2
= ,
a 25 (25 + 25 + 1)· 25! 651

mengakibatkan m + n = 676.
Komentar:

a) Soal ini tergolong cukup mudah mengingat 43% peserta mampu


menjawabnya dengan benar.
b) Pengamatan yang cukup penting dalam menjawab soal ini adalah
pengubahan bentuk a k menjadi bentuk yang dapat dilakukan teles-
koping. Pendekatan lain yang bisa dilakukan ialah mencari bentuk
umum dari a 1 + a 2 + · · · + a k dengan memasukkan nilai k yang kecil.

10. Jawab: 5.

Q A

B
C

Pandang bahwa lingkaran-lingkaran (B, BP), (C, CP) berpotongan di P, Q


dengan PQ adalah diameter lingkaran (B, BA). Kita juga p PQ ⊥BC.
p punya
PA
Perhatikan bahwa 2∠QP A = ∠ ACB. Akibatnya, Q A = 2 + 3, dan m +
n = 5.

91
Buku KTO Matematika 1

Komentar:

a) Sebanyak 33% peserta mampu menjawab soal ini dengan benar, me-
nandakan bahwa soal ini tergolong cukup sulit.

11. Jawab: 462.


Untuk menjawab soal ini, kita menggunakan sifat bahwa
2n
t(k) = n2 .
X
s(n) =
k= n+1

Untuk membuktikannya, perhatikan bahwa

2n
X 2n
X n
X
t(k) = t(k) − t(k)
k= n+1 k=1 k=1
Xn n
X n
X
= t(2k − 1) + t(2k) − t(k)
k=1 k=1 k=1
Xn n
X n
X
= 2k − 1 + t(k) − t(k)
k=1 k=1 k=1
Xn
= 2k − 1
k=1
2
=n .

Dengan demikian, kita punyai

2048
X
t(k) = t(1) + s(1) + s(2) + s(4) + s(8) + s(16) + s(32) + s(64)
k=1
= 1 + 12 + 22 + 42 + 82 + 162 + 322 + 642
= 1 + 1 + 4 + 16 + 64 + 256 + 1024 + 4096
= 5462.

Akibatnya, tiga digit terakhir dari ekspresi pada soal adalah 462.

92
Pembahasan Kontes September 2015

Komentar:

a) 30% peserta mampu menjawab soal ini dengan benar, menunjukkan


bahwa soal ini cenderung sulit bagi para kontestan.
b) Hal yang penting untuk digarisbawahi adalah identitas s(n) = n2 .
Pemahaman akan identitas tersebut memberikan kemudahan yang
sangat berarti dalam penyelesaian soal ini.

12. Jawab: 11.


Perhatikan bahwa
a + 5b + 3c 3(a + b + c)
= − 2,
b−a b−a
sehingga kita cukup mencari nilai terkecil yang mungkin untuk
a+b+c
.
b−a
Misalkan f (x) = ax2 + bx + c. Akibatnya, a + b + c = f (1) dan 3(b − a) =
f (1) − f (−2). Karena f (−2) ≥ 0,
a+b+c 3 f (1) 3 f (1)
= ≥ = 3.
b−a f (1) − f (−2) f (1)
Kesamaan terjadi saat f (−2) = 0. Sebagai contoh, saat a = 1, b = 4, dan
c = 4, diperoleh
a+b+c
= 3.
b−a
Jadi, nilai minimum ekspresi pada soal adalah 11.
Komentar:

a) Fakta bahwa 9% peserta mampu menjawab soal ini dengan benar


bisa dijadikan dasar untuk berkesimpulan bahwa soal ini tidaklah
mudah.
b) Soal ini dapat dipandang juga dengan cara berikut: Andaikan b =
2
ka. Dengan³ 2 ´ sifat diskriminan b − 4ac ≤ 0 dapat dibuktikan bah-
wa c ≥ a k4 . Akibatnya soal ini ekuivalen dengan mencari nilai
minimum dari
4 + 20k + 3k2 1 27
µ ¶
f (k) = = 26 + 3(k − 1) +
4(k − 1) 4 k−1

93
Buku KTO Matematika 1

untuk k > 1. Dengan ketaksamaan AM-GM (rataan aritmetik-


rataan geometrik),
27
3(k − 1) + ≥ 18,
k−1
sehingga f (k) ≥ 11, sesuai dengan jawaban di atas.
c) Cara pada komentar (b) cenderung lebih mudah dipikirkan, tetapi
melibatkan penghitungan yang lebih rumit. Penebakan bahwa 11
adalah nilai minimum cukup membantu dalam menjawab soal ini.

13. Jawab: 16.

D G C

A E B

Misalkan H adalah proyeksi B pada GE dan I adalah proyeksi F pada


BH Perhatikan bahwa 4FBI ≡ 4GFC, GH = F I = GC, dan BH = BC.
Misalkan BF = x. Pembaca diberi kesempatan untuk memverifikasi
hubungan-hubungan berikut:

GC 2 = x2 − (BC − x)2 = 2BCx − BC 2


HE 2 = (GE − GH)2
BE 2 − BH 2 = (BE − GC)2 = BE 2 − 2BE.GC + GC 2
BC 2 + GC 2 = 2BE.GC, BCx = BE.GC
(BCx)2 = BE 2 .GC 2 = (5 − x2 )(2BCx − BC 2 )

94
Pembahasan Kontes September 2015

p p p
11− 3
Akibatnya, diperoleh x3 − 5x + 2 3 = 0, x = 2 , sehingga a + b + c =
11 + 3 + 2 = 16.
Komentar:

a) 9% peserta mampu menjawab soal ini dengan benar, menandakan


tingkat kesulitan soal ini yang sangat tinggi.
b) Trik pengerjaan soal ini ialah menyatakan panjang BF dengan pan-
jang sisi-sisi lainnya. Pembaca dipersilakan untuk melakukan veri-
fikasi untuk setiap persamaan yang ada di solusi ini.

14. Jawab: 241.


Perhatikan bahwa jak a − [a (mod 5)]
= .
5 5
Karena 74 ≡ 1 (mod 5), diperoleh 74k+1 , 74k+2 , 74k+3 , 74k+4 (mod 5)
berturut-turut bernilai 2, 4, 3, 1. Akibatnya,

X 7i
2015 X 7i − 7i
2015 (mod 5)
¹ º
=
i =1 5 i =1 5
 
1 72016 − 1 1 
= · − 2+4+3+1+2+
{z4 + · · · + 2 + 4 + 3}

5 6 5 |
2015
2016
7 − 30241
= .
30

Dengan demikian, dipunyai 30N + 30241 = 72016 . Tentunya, N > 30241,


sehingga M = 30241. Jadi, tiga digit terakhir dari M adalah 241.
Komentar:

a) 13% peserta mampu menjawab soal ini dengan benar.


b) Observasi penting pada soal ini adalah menyatakan a5 dengan
¥ ¦

bentuk lain yang melibatkan sisa pembagian a dengan 5. Setelah


itu, semuanya mengikuti.

95
Buku KTO Matematika 1

Bagian B

1. Jawab: 14.
Pilih salah satu diagonal l, yaitu yang membentang dari ujung kiri atas
ke ujung kanan bawah pada papan catur tersebut. Kita katakan dua
buah kotak sediagonal jika garis yang menghubungkan kedua buah ko-
tak tersebut sejajar dengan l. Pandang kotak pada ujung kiri bawah X
dan ujung kanan atas Y . Selain dua kotak ini, 62 kotak lain pada papan
catur ini dapat dikelompokkan menjadi 13 kelas sedemikian sehingga
setiap kotak pada masing-masing kelas semuanya sediagonal.
Sekarang, di setiap kelas ini hanya bisa diletakkan satu buah gajah kare-
na jika ada dua gajah pada dua kotak yang sekelas, kedua gajah ini pasti
saling menyerang. Jadi, di 62 kotak ini terletak paling banyak maksimal
13 gajah. Sekarang, tidak mungkin kedua kotak X dan Y diletakkan
gajah, karena mereka akan saling menyerang (pandang bahwa mereka
akan sejajar dengan diagonal yang tegak lurus l. Jadi, maksimal ada 14
gajah yang tidak saling menyerang yang dapat diletakkan pada papan
catur.
Sekarang, kita perlu mencari tahu apakah 14 gajah dapat diletakkan
dalam papan catur tersebut. Ternyata, hal tersebut mungkin dicapai.
Konfigurasi 14 gajah tersebut adalah sebagai berikut. Perhatikan bahwa
tidak ada dua gajah yang saling menyerang.

96
Pembahasan Kontes September 2015

Komentar:

a) Perlu ditekankan bahwa memberikan konfigurasi gajah seperti di


atas merupakan bagian penting dari solusi, karena ini membuktik-
an bahwa mungkin meletakkan 14 gajah di papan catur sehingga
mereka tidak saling menyerang.
b) Dapat dibuktikan pula bahwa 8 buah menteri, 8 buah benteng, atau
32 buah kuda dapat diletakkan pada papan catur sehingga tidak
ada buah catur yang saling menyerang.
c) Kesalahan yang banyak dilakukan oleh para kontestan ialah menja-
wab 14 tampa membuktikan bahwa nilai tersebut adalah yang ter-
besar. Hal tersebut tidak diperbolehkan karena soal meminta kita
untuk menentukan banyak gajah terbesar yang mungkin diletak-
kan. Klaim benar yang tidak disertai argumen pembuktian yang
valid akan berakibat kepada pengurangan nilai.

2. Jawab: 12 , 1, 13 dan − 21 , −1, − 13 .


¡ ¢ ¡ ¢

Perhatikan bahwa jika (a, b, c) merupakan solusi, (−a, − b, − c) juga solusi.


Dengan demikian, tanpa mengurangi keumuman, bisa dimisalkan bah-
wa r 2 adalah bilangan real yang sama dengan ekspresi 3(a + a1 ), 4(b + 1b ),
dan 5(c + 1c ). Karena ab + bc + ca = 1, (a + b)(a + c) = a2 + ab + bc + ca = a2 + 1.
Jadi,
ar 2
(a + b)(a + c) =
3
br 2
(b + c)(b + a) =
4
cr 2
(c + a)(c + b) = .
5
Kalikan ketiganya, diperoleh
abcr 6
(a + b)2 (b + c)2 (c + a)2 =
60
p
abcr 3
(a + b)(b + c)(c + a) =
60
sehingga
s
5r 3 ab 5r 3 p
a+b = ca + bc = abc
60 c 60

97
Buku KTO Matematika 1

s
3r 3 bc 3r 3 p
b+c = ab + ac = abc
60 a 60
4r 3 4r 3 p
r
ca
c+a= bc + ab = abc.
60 b 60
Jumlahkan ketiga persamaan di ruas kanan, diperoleh
12 3 p
2(ab + bc + ca) = r abc
60
p
r 3 abc = 10.

Jadi,
5
ca + cb =
6
1
ab + ac =
2
2
bc + ab = .
3
Dari sini, diperoleh ab = 61 , bc = 12 , dan 13 . Akhirnya, diperoleh (a, b, c) =
¡1 1
¢ ¡ 1 1
¢
2 , 1, 3 dan − 2 , −1, − 3 . Perhatikan bahwa kedua pasangan terurut ini
merupakan solusi yang dicari.
Komentar:

a) Observasi bahwa (a, b, c) adalah solusi jika dan hanya jika


(−a, − b, − c) cukup penting karena membuat kita bisa berasumsi,
tanpa mengurangi keumuman, bahwa a + a1 merupakan bilangan
positif.
b) Observasi bahwa (a + b)(a + c) = a a + a1 (dan bentuk lain yang se-
¡ ¢

tara untuk b dan c) juga penting.


c) Kesalahan yang banyak dilakukan oleh para kontestan adalah:
i. Membuat pemisalan yang tidak umum mengenai hubungan a,
b, dan c, contohnya a < b < c. Perhatikan bahwa pemisalan
seperti demikian tidak berlaku untuk semua soal. Pada soal
ini, kesimetrisan tidak terlihat, sehingga kita tidak boleh me-
misalkan a < b < c tanpa fakta pendukung lain.
ii. Menggunakan teknik mencoba-coba, atau trial and erroruntuk
mencari semua solusi. Metode ini tidak valid karena kita di-
minta untuk mencari semua pasangan yang memenuhi.

98
Pembahasan Kontes September 2015

iii. Tidak memasukkan kembali solusi yang didapat ke persama-


an awal. Hal ini sangatlah fatal dan dapat mengakibatkan pe-
ngurangan poin yang sebenarnya bisa dihindari. Contoh klasik
yang menunjukkan pentingnya substitusi balik adalah: tentu-
kan semua pasangan bilangan real (a, b) sehingga a2 + b2 = 0
dan a + b = 1; dengan melihat persamaan pertama didapat
a = b = 0, yang tidak memenuhi persamaan kedua, sehingga
sistem persamaan tersebut tidak memiliki solusi.

3. Bukti.
Perhatikan lema berikut:
n
Lema. Jika d adalah pembagi positif spesial sehingga d(d + 1) 6= n, d +1
bukan pembagi positif spesial.
Bukti lema. Perhatikan bahwa d | n dan d + 1| n. Misalkan n = (d + 1)k un-
tuk suatu bilangan asli k. Akibatnya, d | dk + k, sehingga d | k. Sekarang,
jika dn+1 merupakan pembagi spesial, dn+1 + 1 = k + 1| n, mengakibatkan
k + 1|(d + 1)k, atau k + 1| d + 1. Akibatnya, d = k, sehingga n = (d + 1)d.
Kontradiksi. Dengan demikian, lema terbukti.
Jika n = d(d + 1), perhatikan bahwa d adalah pembagi spesial dan d + 1
bukan pembagi spesial karena pastilah (d + 1) + 1 bukan pembagi spesial.
Jadi, terbukti bahwa banyaknya pembagi spesial dari n tidak lebih dari
setengah banyaknya pembagi positif dari n.
Komentar:

a) Kunci dari penyelesaian soal ini adalah mendapatkan pernyataan


seperti pada lema. Pernyataan tersebut didasari oleh pencarian sa-
tu pembagi positif tidak spesial yang unik untuk setiap satu pem-
bagi positif spesial. Dengan demikian, bisa didemonstrasikan bah-
wa banyaknya pembagi spesial tidak lebih dari setengah banyaknya
pembagi positif keseluruhan.
b) Beberapa kesalahan yang dilakukan para kontestan adalah:
i. Melakukan pemisalan yang tidak umum. Beberapa contoh pe-
misalan yang dilakukan tanpa dasar ialah memisalkan bahwa
n habis dibagi 2 atau 3.
ii. Menggunakan beberapa nilai n untuk membuat kesimpulan
umum. Kesalahan ini sangat klasik dan harus dihindari.

99
Buku KTO Matematika 1

4. Bukti.

Terdapat dua kasus yang mungkin. Kasus pertama adalah di mana A 0


dan B0 terletak pada sisi yang sama terhadap X Y . Kasus kedua adalah
di mana A 0 dan B0 terletak pada sisi yang berbeda terhadap X Y .

A0
E
X

B0

Y
C
B D

Untuk kasus pertama, perhatikan bahwa segitiga ADC dan BEC me-
rupakan segitiga siku-siku yang sebangun dengan besar sudut C yang
sama, menurut kriteria sudut-sudut. Sekarang, karena A 0 dan B0 meru-
pakan titik tengah sisi AD dan BE, berlaku bahwa segitiga A A 0 C seba-
ngun dengan segitiga BB0 C (bisa dibuktikan dengan kriteria sisi-sudut-
sisi). Diperoleh ∠ A A 0 C = ∠BB0 C. Dengan demikian,

∠ X A 0 Y = ∠C AD = 180° − ∠ A A 0 C = 180° − ∠BB0 C = ∠CB0 E = ∠Y B0 X .

100
Pembahasan Kontes September 2015

A0

X
Y
B0

C
B D

Pada kasus kedua, perhatikan bahwa dari kesebangunan A A 0 C dan


BB0 C diperoleh ∠BB0 C = ∠ A A 0 C, sehingga berlaku

∠ X A 0 Y = 180° − ∠ A A 0 C = 180° − ∠BB0 C.

Komentar:

a) Kunci dari penyelesaian soal ini adalah mendapatkan pasangan se-


gitiga yang sebangun kemudian menganalisis sudut.
b) Beberapa kesalahan yang dilakukan para kontestan adalah:
i. Kebanyakan peserta hanya mengerjakan satu kasus tanpa me-
merhatikan kasus yang lainnya.
ii. Menggunakan fakta bahwa A 0 BX Y merupakan segiempat tali
busur. Padahal, fakta inilah yang ingin dibuktikan.

101
5 Kontes Oktober 2015

5.1 Soal-Soal
Bagian A

Tuliskan jawaban akhir setiap soal di lembar jawaban. Setiap soal bernilai 1
angka. Jawaban Anda harus berupa bilangan bulat dari 0 sampai 999.

1. (1005) Untuk setiap bilangan real a, nyatakan bac sebagai bilangan bulat
terbesar yang tidak lebih dari a. Sebagai contoh, b3.14c = 3 dan b2c = 2.
Jika sebuah bilangan real x memenuhi x +b2xc = 3.14, tentukan nilai dari
100x.

2. (1012) Tentukan banyaknya pasangan bilangan bulat (m, n) yang meme-


nuhi persamaan 4(n2 + m) = mn.

3. (1019) Tentukan banyaknya cara mengacak huruf-huruf K, K, T, T, O,


dan O sedemikian rupa sehingga huruf-huruf K, T, dan O tidak terletak
bersebelahan dalam urutan tersebut.

4. (1026) Misalkan ABCDp adalah sebuah segiempat konveks yang meme-p


nuhi AB = 10, CD = 3 6, ∠ ABD = 60°, ∠BDC = 45°, dan BD = 13 + 3 3.
Tentukan panjang AC.

5. (1033) Diberikan sebuah polinomial P(x) = 1 − x + x2 − x3 + · · · + x14 − x15 .


Tentukanlah tiga digit (angka) terakhir dari koefisien x4 pada polinomial
P(x − 1).

6. (1040) Misalkan N adalah banyaknya tuple bilangan bulat terurut


(a, b, c, d) dengan a, b, c, d ∈ {0, 1, 2, 3, 4, 5, 6, 7, 8, 9, 10} yang memenuhi ab −
cd habis dibagi 11. Tentukan tiga digit (angka) pertama dari N.

103
Buku KTO Matematika 1

7. (1047) Misalkan E adalah sebuah titik yang terletak di luar sebuah bujur
sangkar ABCD. Jika jarak E ke AC adalah 6, jarak E ke BD adalah
17, dan jarak E ke titik sudut bujur sangkar yang terdekat adalah 10,
tentukanlah luas terkecil yang mungkin bagi bujur sangkar tersebut.
p p p
8. (1054) Nilai terbesar untuk x − 1 + 2x − 51 + 199 − 3x di interval 153 6 ≤
x ≤ 3986 adalah s; nilai tersebut terjadi pada saat x bernilai t. Hitunglah
s + t.

9. (1061) Tentukan banyaknya pasangan terurut (a 1 , a 2 , a 3 , a 4 , a 5 , a 6 ) de-


ngan a i ∈ {−1, 0, 1} untuk setiap i = 1, 2, 3, 4, 5, 6 demikian sehingga

a 1 · 5 + a 2 · 52 + a 3 · 53 + a 4 · 54 + a 5 · 55 + a 6 · 56

bernilai positif.

10. (1068) Misalkan C dan D adalah titik-titik pada lingkaran dengan titik
pusat O dan diameter AB, di mana C dan D berada pada sisi yang ber-
beda terhadap diameter AB. Misalkan H adalah kaki tegak lurus dari B
ke CD. Jika AO = 13, AC = 24, dan HD = 12, tentukanlah besar ∠DCB
dalam derajat.

11. (1075) Sebuah bilangan asli disebut spesial jika bilangan tersebut habis
dibagi oleh tiap-tiap digit (angka)-nya yang bukan 0. Ada berapa paling
banyak bilangan-bilangan berurutan yang semuanya spesial?

12. (1082) Misalkan E adalah sebuah titik di dalam sebuah belah ketupat
ABCD demikian sehingga AE = EB, ∠E AB = 12°, dan ∠D AE = 72°.
Tentukanlah besar ∠CDE dalam derajat.

13. (1089) Pada sebuah papan berukuran 3 × 3, dua kotak diwarnai biru dan
dua kotak lainnya diwarnai merah demikian sehingga dua kotak yang
sama warna selalu tidak sekolom maupun sebaris. Tentukan banyak pe-
warnaan yang demikian.

14. (1096) Untuk setiap dua bilangan real x dan y dengan x y = 1, berlaku
ketaksamaan
((x + y)2 + 4)((x + y)2 − 2) ≥ A(x − y)2 .
Tentukan nilai terbesar bagi A yang mungkin.

104
Soal-Soal Kontes Oktober 2015

Bagian B

Tuliskan jawaban beserta langkah pekerjaan Anda pada lembar jawaban.


Selain jawaban akhir, Anda perlu menuliskan argumentasi atau langkah-
langkah untuk memperoleh jawaban akhir tersebut. Gunakan halaman yang
berbeda untuk setiap soal yang berbeda. Setiap soal bernilai 7 angka.

1. a) (1117) Sebuah fungsi f : R → R memenuhi persamaan

f (x) f (y) − f (x y) = 2(x + y + 1)

untuk setiap dua bilangan real x dan y. R menyatakan himpunan


yang mengandung semua bilangan real.
i. Tunjukkan bahwa nilai-nilai yang mungkin bagi f (0) hanyalah
−1 dan 2.
ii. Jika f (0) = 2, tunjukkan bahwa haruslah f (x) = x + 2 untuk se-
mua x ∈ R. Periksa pula bahwa f (x) = x + 2 untuk semua x ∈ R
memang merupakan solusi.
iii. Tunjukkan bahwa tidak ada fungsi f yang memenuhi f (0) = −1.
b) Carilah semua fungsi g : R → R demikian sehingga

g(x)g(y) − g(x + y) = x y

untuk setiap dua bilangan real x dan y.

2. a) (1124) Misalkan a, b, c, d, e, f , g merupakan bilangan-bilangan asli


demikian sehingga masing-masing dari

ab, bc, cd, de, e f , f g, ga

merupakan bilangan kubik. Tunjukkan bahwa masing-masing dari


a, b, c, d, e, f , g juga merupakan bilangan kubik.
b) Jika a, b, c, d, e, f merupakan bilangan-bilangan asli demikian
sehingga masing-masing dari

ab, bc, cd, de, e f , f a

merupakan bilangan kubik, haruskah masing-masing dari a, b, c,


d, e, f merupakan bilangan kubik?

105
Buku KTO Matematika 1

3. (1131) Misalkan ABC merupakan sebuah segitiga lancip. Titik-titik H,


I, O, berturut-turut, adalah pertemuan titik tinggi, titik pusat lingkaran
dalam segitiga ABC, dan titik pusat lingkaran luar segitiga ABC. Ji-
ka ∠BHC = 120°, buktikan bahwa lingkaran luar segitiga H IO melalui
titik-titik B dan C.

4. (1136) Diberikan sebuah bilangan asli n ≥ 2. Tiap-tiap kotak pada sebuah


papan catur berukuran (2n + 1) × (2n + 1) diwarnai hitam atau putih (tidak
keduanya). Sebuah kotak dikatakan spesial jika terdapat setidaknya n
kotak lain di barisnya dan juga kolomnya yang sama warna dengan kotak
tersebut.

106
Soal-Soal Kontes Oktober 2015

a) Tunjukkan bahwa terdapat setidaknya 2n + 1 kotak spesial.


b) Berikan sebuah konfigurasi pewarnaan di mana terdapat tepat 4n
kotak spesial.

107
Buku KTO Matematika 1

5.2 Pembahasan
Bagian A

1. Jawab: 114.
Misalkan x = b xc+{ x}. Perhatikan bahwa pernyataan pada soal ekuivalen
dengan b xc + { x} + b2xc = 3.14. Karena 0 ≤ { x} < 1, diperoleh 2.14 < b xc +
b2xc ≤ 3.14. Selanjutnya, pandang bahwa b xc + b2xc ∈ Z, sehingga b xc +
b2xc = 3. Akibatnya, { x} = 0.14, dan x = 1.14. Nilai dari 100x adalah 114.
Komentar:

a) Berdasarkan data bahwa 86% kontestan mampu menjawab soal ini


dengan benar, bisa dikatakan bahwa soal ini adalah yang termudah
dari semua soal isian singkat pada kontes bulan ini.
b) Cukup dibutuhkan pengetahuan mendasar mengenai fungsi tangga
untuk dapat mengerjakan soal ini.
c) Beberapa kontestan tidak membaca dengan teliti apa yang dii-
nginkan soal, sehingga menjawab 1, 14 dan bukan 114.

2. Jawab: 14.
Perhatikan bahwa pernyataan pada soal bisa dinyatakan ulang sebagai
2
m = n4−n 4 = (4n + 16) + n64
−4 . Akibatnya, kita tinggal mencari banyaknya
nilai n yang mengakibatkan n − 4 habis membagi 64 = 26 (lihat bahwa
setiap nilai n akan menghasilkan pasangan (m, n) yang unik). Dapat di-
verifikasi bahwa terdapat 14 nilai n berbeda yang memenuhi, yang bera-
kibat kepada terdapatnya 14 pasangan (m, n) yang memenuhi.
Komentar:

a) Soal ini bisa diselesaikan oleh 36% peserta, sehingga bisa digolong-
kan sebagai soal bertingkat kesulitan sedang. Fakta ini cukup me-
ngejutkan mengingat tidak diperlukan pengetahuan spesifik untuk
mengerjakan soal ini dan natur soal yang cukup sering muncul di
olimpiade matematika.
b) Kunci dari soal ini ialah menyadari bahwa m bisa dinyatakan seba-
gai penjumlahan bilangan bulat dan n64
−4 , sehingga cukup kita tinjau
kasus di mana n − 4 habis membagi 64.

108
Pembahasan Kontes Oktober 2015

c) Beberapa kesalahan umum yang dilakukan oleh para kontestan


yakni lupa meninjau kasus untuk n − 4 < 0 dan tidak menyadari
bahwa 1 dan n adalah faktor dari n untuk setiap bilangan asli n.

3. Jawab: 67.
Untuk menyelesaikan soal ini, kita akan menggunakan prinsip inklusi-
eksklusi. Perhatikan bahwa semua permutasi yang mengandung blok
KTO bisa dikelompokkan menjadi empat kasus berdasarkan letak K (di
tempat pertama, kedua, ketiga, atau keempat). Untuk masing-masing
kasus, dapat diperiksa bahwa terdapat 3! = 6 permutasi. Berarti, terda-
pat 4 × 6 = 24 permutasi. Namun, perlu dilihat bahwa terdapat tepat 1
permutasi yang terhitung dua kali, yakni KTOKTO, yang masuk ke da-
lam kasus di mana K terletak di urutan pertama dan keempat. Oleh ka-
rena itu, banyaknya cara penyusunan yang mungkin adalah banyaknya
cara penyusunan tanpa syarat, yaitu 26!·2 = 90, dikurangi banyaknya cara
penyusunan yang mengandung blok KTO, yaitu 23. Akibatnya, jawaban
yang diinginkan adalah 67.
Komentar:

a) Hanya 14% peserta yang mampu menjawab soal ini dengan benar,
menandakan tingkat kesulitan soal yang tinggi.
b) Penggunaan prinsip inklusi-eksklusi menjadi kunci utama dalam
pengerjaan soal ini.
c) Banyak kontestan mendapatkan jawaban 66. Hal ini kemungkin-
an disebabkan karena mereka gagal menyadari bahwa permutasi
KTOKTO terhitung dua kali.

4. Jawab: 16.

C
K

B M N D

109
Buku KTO Matematika 1

Misalkan M dan N adalah proyeksi titik A dan C terhadap garis BD


secara berturut-turut. Perhatikan bahwa sin ∠ ABM = sin 60° = AM AB =
AM
p BM BM
10 ⇐⇒ AM = 10 sin 60° = 5 3 dan cos ∠ ABM = cos 60° = AB = 10 ⇐⇒

CN CN
p pandang pula bahwa sin ∠CDN
BM = 10 cos 60° = 5. Lalu,
DN
= sin 45° =
DN
CD = 3 6 ⇐⇒ CN = 3 3 dan cos ∠CDN = cos 45° = DC = 3 6 ⇐⇒
p p
p p p
DN = 3 6 cos 45°. Karena BM = 5, DN = 3 3 dan BD = 13 + 3 3, di-
peroleh MN = 8. Misalkan proyeksi C ke AM adalah K. Perhatikan
bahwa MNCKpadalah persegi panjang, sehingga CK = MN = 8 dan
MK = NC = 3 3. Lebih lanjut, ∠ AK C = 90° p mengisyaratkan bahwa
AC 2 = AK 2 + K C 2 = (AM + MK)2 + MN 2 = (8 3)2 + 82 = 256 menurut te-
orema Pythagoras, atau AC = 16.
Komentar:

a) Soal ini bisa dikerjakan oleh 38% dari seluruh peserta. Artinya, soal
ini memiliki tingkat kesulitan sedang berdasarkan statistik.
b) Observasi penting dalam pengerjaan soal ini adalah mencari hu-
bungan antarpanjang sisi dan antarbesar sudut, serta penggunaan
teorema Pythagoras. Teknik yang demikian sering dipakai dalam
menjawab soal-soal geometri yang melibatkan pencarian dan me-
ngandung informasi mengenai panjang segmen.

5. Jawab: 368.
15
(1 − x) i .
P
Perhatikan bahwa dari soal kita bisa memperoleh P(x − 1) =
i =1
15 ¡i¢
Menurut Binomial Newton, koefisien dari x4 di P(x − 1) adalah
P
4 .
i =4
k ¡ ¢
P i ¡k+1¢
Selanjutnya, akan dibuktikan dengan induksi di k − t bahwa, t = t+1
i= t
untuk k ≥ t. Perhatikan bahwa pernyataan ini benar untuk k − t = 0.
tP
+n ¡ ¢ ¡
i t+ n+1¢
Misalkan pernyataan ini benar untuk k − t = n; maka t = t+1 dan
i= t
t+P
n+1 ¡
i¢ ¡ t+n+1¢
+ t+t+n+1¢
¡ ¡ t+n+1¢ ¡ t+n+1¢
t = t+1 1 . Menurut identitas Pascal, t+1 + t+1 =
i= t
¡ t+n+2¢ t+P
n+1 ¡
i¢ ¡ t+n+2¢
t+1 , sehingga t = t+1 . Akibatnya, pernyataan benar untuk
i= t
k − t = n + 1, dan menurut induksi matematika, pernyataan benar untuk
semua k ≥ t.

110
Pembahasan Kontes Oktober 2015

15 ¡i¢ ¡16¢
P 4
Berarti, 4 = 5 , sehingga 3 digit terakhir dari koefisien x pada po-
i =4
linomial P(x − 1) adalah 368.
Komentar:

a) Sebanyak 38% siswa mampu mengerjakan soal ini dengan tepat,


mengisyaratkan bahwa soal ini tergolong dalam kategori sedang.
b) Kunci utama dalam penyelesaian soal ini ialah utilisasi Binomial
Newton dan identitas Pascal. Dua hal tersebut sering ditemui da-
lam soal-soal yang berjenis demikian. Pembaca yang belum familiar
dengan salah satu dari kedua hal tersebut direkomendasikan untuk
mencari referensi untuk memperkaya pengetahuan.
c) Ketidaktelitian yang ditemukan pada beberapa kontestan di anta-
ranya adalah tidak membaca bahwa hal yang diminta soal ialah
bukanlah nilai dari 16
¡ ¢
5 , melainkan tiga angka terakhirnya.

6. Jawab: 144.
Untuk menyelesaikan soal ini, kita akan membagi kasus berdasarkan
banyaknya angka 0 di antara a, b, c, dan d.
Perhatikan bahwa untuk semua pilihan a, b, c, dan d di mana tidak
ada variabel yang nilainya sama dengan 0, ab − cd ≡ 0 mod 11 ⇐⇒ d ≡
ab
c mod 11. Dari sini didapat bahwa untuk setiap pilihan a, b, c yang
semuanya taknol, ada tepat satu solusi d. Oleh karena itu, banyaknya
pasangan terurut a, b, c yang semuanya taknol 0 adalah 10.10.10 = 1000.
Jika di antara a, b, c, d ada tepat satu variabel yang bernilai 0, ab − cd ≡
0 mod 11 salah. Akibatnya, tidak ada solusi untuk kasus ini.
Jika di antara a, b, c, d ada tepat dua variabel yang bernilai 0, perha-
tikan bahwa tidak boleh a, b atau c, d yang keduanya bernilai 0 (silak-
an verifikasi sendiri). Akibatnya, di antara a, b ada tepat satu variabel
yang bernilai 0, dan di antara c, d ada tepat variabel satu yang bernilai
0. Dengan demikian, banyaknya pasangan terurut sehingga ada tepat
dua variabel yang bernilai 0 adalah 2 · 10 · 2 · 10 = 400.
Jika di antara a, b, c, d ada tepat tiga variabel yang bernilai 0, perhati-
kan bahwa satu bilangan yang ditentukan dapat mengambil nilai setiap
bilangan asli pada himpunan {1, 2, ..., 10}. Karena ada 4 kemungkinan
variabel yang tidak bernilai 0, banyaknya pasangan terurut yang meme-
nuhi kriteria soal dan memiliki tepat tiga buah 0 adalah 4.10 = 40

111
Buku KTO Matematika 1

Jika di antara a, b, c, d ada tepat empat yang bernilai 0, diperoleh hanya


ada satu pasangan terurut yang memenuhi.

Sebagai kesimpulan, banyaknya pasangan terurut yang memenuhi ada-


lah N = 1000 + 400 + 40 + 1 = 1441, sehingga tiga digit pertama N adalah
144.

Komentar:

a) Secara mengejutkan, hanya satu, atau 2% dari keseluruhan kontes-


tan, yang mampu menjawab soal ini dengan benar. Hal ini menun-
jukkan bahwa masih banyak peserta yang belum familiar dengan
soal bertipe demikian.

b) Pengamatan yang sangat penting dalam menjawab soal ini adalah


pembagian kasus berdasarkan banyaknya angka nol. Pembagian
kasus seperti ini adalah natural mengingat banyaknya cara pemi-
lihan angka sangat bergantung pada banyaknya nol.

7. Jawab: 162.

y
(0, 25)

E(17, 6)
A C (25, 0)
x

112
Pembahasan Kontes Oktober 2015

Kita letakkan struktur soal pada bidang kartesius. Misalkan O = (0, 0)


adalah perpotongan kedua diagonal persegi. Tanpa mengurangi keu-
muman, misalkan B = (0, m), D = (0, − m), C = (m, 0), dan A = (− m, 0)
dengan m > 0. Selanjutnya, mengingat jarak E ke BD dan AC adalah
17 dan 6 secara berturut-turut, tanpa mengurangi keumuman bisa kita
misalkan E = (17, 6). Perhatikan bahwa EC < E A dan EB < ED. Karena
EC 2 = (17− m)2 +62 = (172 +62 + m2 )−34m dan EB2 = 162 +(6− m)2 = (172 +
62 + m2 ) − 12m, didapat EC < EB. Pandang pula bahwa karena jarak E ke
titik sudut bujur sangkar terdekat adalah 10, EC 2 = 102 = (17 − m)2 + 62 ,
mengakibatkan 17 − m = ±8. Jika 17 − m = −8, didapat m = 25. Perhati-
kan bahwa saat (17, 8) ∈ BC dan (17, 0) ∈ OC, titik E ada di dalam OBC,
dan titik E ada di dalam bujur sangkar, sebuah kontradiksi. Berarti,
17 − m = 8, atau m = 9. Akibatnya, luas bujur sangkar tersebut adalah
4 · [BOC] = 4 × BO2·OC = 4 × 81
2 = 162.
Komentar:

a) Sebanyak 17% peserta mampu menjawab soal ini dengan benar. Ini
menandakan tingkat kesulitan soal ini yang cukup tinggi.
b) Solusi di atas memanfaatkan pendekatan analitik. Pemisalan yang
dibuat bertujuan untuk mempermudah penghitungan. Pembaca di-
harapkan untuk dapat melakukan pemilihan titik yang tepat da-
lam menjawab soal geometri menggunakan sistem koordinat untuk
menghemat waktu dan meminimalkan kesalahan penghitungan.
c) Soal ini dapat pula diselesaikan dengan meninjau panjang dari
segmen-segmen yang ada pada gambar. Pembaca dipersilakan un-
tuk mencobanya sendiri.

8. Jawab: 71.
Karena 153 398
6 ≤ x ≤ 6 , bisa disimpulkan bahwa nilai dari 2x − 51 dan 199 −
3x lebih besar atau sama dengan 0. Menurut pertidaksamaan AM-QM
(rataan aritmetik-rataan kuadratik),
p p p s s
x − 1 + 2x − 51 + 199 − 3x (x − 1) + (2x − 51) + (199 − 3x) 147
≤ = = 7,
3 3 3
p p p
sehingga x − 1 + 2x − 51 + 199 − 3x ≤ 21, mengakibatkan s ≤ 21. Per-
hatikan bahwa kesamaan
p mungkinp p x−1 = 2x−51 = 199−3x meng-
terjadi:
akibatkan x = 50 dan x − 1 + 2x − 51 + 199 − 3x = 21. Akibatnya, nilai

113
Buku KTO Matematika 1

minimum adalah s = 21 dan ini tercapai saat t = 50. Diperoleh bahwa


s + t = 71.
Komentar:

a) Sebanyak 48% dapat menjawab soal ini dengan tepat, menandakan


bahwa soal ini tergolong cukup mudah.
b) Penggunaan teorema AM-QM tidak menjamin bahwa nilai maksi-
mum ekspresi pada soal adalah 21; hal yang pasti adalah nilai mak-
simum ekspresi tersebut kurang dari 21. Perlu ditunjukkan bahwa
terdapat x yang menghasilkan nilai 21 sebelum berkesimpulan bah-
wa nilai tersebut adalah yang maksimum. Dalam hal ini, x = 50.

9. Jawab: 364.
n
5k+1 −5
5k = < 5k+1 .
P
Perhatikan kalau 4
k=1
Jika untuk i terbesar sehingga a i 6= 0µ diketahui a i = −1, dipunyai a 1 .5 +
iP
−1

a 2 .52 + a 3 .53 + a 4 .54 + a 5 .55 + a 6 .56 < 5k − 5 i < 5 i − 5 i = 0.
k=1
Jika untuk i terbesar sehingga a i 6= 0µdiketahui a i = 1, dipunyai a 1 .5 +
iP
−1

a 2 .52 + a 3 .53 + a 4 .54 + a 5 .55 + a 6 .56 > − 5k + 5 i > 5 i − 5 i = 0.
k=1

Oleh karena itu, nilai a 1 .5 + a 2 .5 + a 3 .53 + a 4 .54 + a 5 .55 + a 6 .56 adalah


2

positif jika dan hanya jika ada i sehingga a i 6= 0 dan untuk i terbesar
sehingga a i 6= 0, dipunyai a i = 1.
Pandang bahwa jika nilai i terbesar sehingga a i 6= 0 adalah j dan a 1 .5 +
a 2 .52 + a 3 .53 + a 4 .54 + a 5 .55 + a 6 .56 bernilai positif, kita dapat a j = 1, dan
untuk semua 1 ≤ k < j, nilai a k ada di antara {−1, 0, 1}. Untuk setiap j,
ada 3 j− i pasangan terurut (a 1 , a 2 , a 3 , a 4 , a 5 , a 6 ) yang memenuhi. Karena
nilai j yang mungkin adalah 1, 2, 3, 4, 5, 6, banyaknya cara penyusunan
adalah 30 + 31 + 32 + 33 + 34 + 35 = 364.
Komentar:

a) Berdasarkan data bahwa 26% dari keseluruhan kontestan mampu


menjawab soal ini dengan benar, bisa dikatakan bahwa soal ini ter-
golong sulit.
b) Observasi yang penting dalam mengerjakan soal ini adalah melihat
kondisi yang ekuivalen dengan nilai ekspresi pada soal yang positif.

114
Pembahasan Kontes Oktober 2015

Dalam hal ini, kondisi tersebut adalah terdapatnya nilai i sehingga


a i taknol dan untuk i terbesar sehingga a i taknol kita punya a i =
1. Kondisi tersebut bisa diperoleh dengan melakukan peninjauan
kasus yang lebih terjangkau.

10. Jawab: 30.

H
A B
O

Karena ABCD adalah segiempat siklis, dipunyai ∠BAC = ∠BDC =


∠BDH. Karena ∠BAC = ∠BDH dan ∠BC A = ∠BHD = 90°, segitiga
ACB sebangun dengan segitiga DHB. p Selanjutnya,
p perhatikan kalau
AB = 2AO = 26. Akibatnya, BC = AB2 − AC 2 = 262 − 242 = 10. Per-
hatikan bahwa 21 = DH BH
AC = BC = sin ∠BCH. Karena C dan D berada di
sisi yang berbeda terhadap diameter AB, diperoleh ∠BCH ≤ 90. Sebagai
konsekuensi, sin ∠BCH = 21 ⇐⇒ ∠BCH = 30°, dan ∠DCB = 30°.
Komentar:

a) Sebanyak 24% peserta mampu mengerjakan soal ini dengan benar.


Akibatnya, soal ini bisa dikatakan sulit.
b) Seperti soal sebelumnya, soal ini juga memanfaatkan teorema
Pythagoras serta hubungan antarpanjang sisi dan antarbesar su-
dut.

11. Jawab: 13.


Jika terdapat lebih dari 13 bilangan berurutan yang semuanya spesial
ada lebih dari 13, perhatikan bahwa terdapat sebuah bilangan t ∈
{3, 4, 6, 7, 8, 9} sedemikian sehingga ada dua di antara 13 bilangan ber-
urutan yang spesial yang angka satuannya t. Dengan kata lain, ada

115
Buku KTO Matematika 1

t ∈ {3, 4, 6, 7, 8, 9} sehingga t| k dan t| k + 10, yang tentunya salah. Akibat-


nya, paling banyak terdapat 13 bilangan spesial yang berurutan. Per-
hatikan bahwa menurut Prinsip Sangkar Merpati (Pigeonhole Princi-
ple), pada barisan 1, 11, . . . ada dua suku yang sisanya sama saat dibagi
h = k pk(3, 4, 6, 7, 8, 9), dengan k pk(a 1 , a 2 , . . . , a n ) menyatakan kelipatan
persekutuan ter-kecil dari a 1 , a 2 , . . . , a n . Selisih kedua bilangan tersebut
berbentuk 1111 . . . 110 . . . 00, yang habis dibagi h. Jika dihilangkan 0 . . . 00
(blok 0 berulang di akhir dari bilangan tersebut), bilangan hasil, sebut
m, tetap habis dibagi h (silakan verifikasi sendiri). Perhatikan kalau
10m, 10m + 1, . . . , 10m + 12 spesial. Sebagai konsekuensi, paling banyak
ada 13 bilangan berturutan yang semuanya spesial.
Komentar:

a) Soal ini pantas untuk disebut sangat sulit mengingat hanya satu
kontestan (2% dari keseluruhan) yang mampu menjawab dengan
benar.
b) Kunci dari penyelesaian soal ini adalah menemukan nilai 13. Nilai
tersebut bisa didapat dengan melakukan peninjauan angka satuan.
Setelah itu, barulah dicari konfigurasi yang memenuhi.

12. Jawab: 66.

D B

Pertama, perhatikan kalau ∠BAD = ∠BAE + ∠D AE = 12° + 72° = 84°.


Karena ABCD adalah belah ketupat, berlaku AB = AD. Ini berar-
ti ∠ ABD = ∠ ADB, dan bisa diperoleh ∠ ADB = 48°. Lihat pula bah-

116
Pembahasan Kontes Oktober 2015

wa AE = EB mengakibatkan 12° = ∠E AB = ∠EBA, sehingga ∠ AEB =


180° − ∠E AB − ∠EBA = 156°.
AB ∠ AEB
Menurut aturan sinus di segitiga AEB, AE = sin sin 156° sin 24°
sin ∠ ABE = sin 12° = sin 12° =
AD
2 cos 12°. Karena AB = AD, didapat AE = 2 cos 12°. Misalkan ∠ AED = a;
maka didapat ∠ ADE = 180° − ∠D AE − ∠DE A = 108° − a. Menurut atur-
an sinus di segitiga AED, 2 cos 12° = AD sin ∠ AED sin a
AE = sin ∠ ADE = sin(108°−a) ⇐⇒
sin a 1
2 sin 78° = sin(108° −a) ⇐⇒ sin(108° − a) · sin 78° = 2 sin a ⇐⇒ sin(108° −
a). sin 78° = sin 30° sin a ⇐⇒ cos(a − 30°) − cos(a + 30°) = cos(30° − a) −
cos(186° − a) ⇐⇒ cos(186° − a) = cos(a + 30°) (karena 0° < a < 180°,
cos(186° − a) = cos(a + 30°)) ⇐⇒ 186° − a = a + 30° ⇐⇒ a = 78°. Akibatnya,
∠ ADE = 108° − 78° = 30°, dan ∠BDE = ∠BD A − ∠ED A = 48° − 30° = 18°,
sehingga ∠CDE = ∠CDB + ∠BDE = ∠ ADB + 18° = 48° + 18° = 66°.
Komentar:

a) Soal yang bisa dikerjakan oleh 21% peserta ini bisa dikatakan sulit.
b) Pencarian hubungan antarbesar sudut dan antarpanjang sisi de-
ngan difasilitasi trigonometri menjadi kunci utama dari penyelesa-
ian soal ini.
c) Soal ini bisa diselesaikan menggunakan pendekatan Euclid. Pem-
baca sangat disarankan untuk menemukan solusi yang dimaksud
mengingat latihan seperti ini sangat dibutuhkan untuk meningkat-
kan intuisi dalam menjawab soal-soal geometri yang melibatkan
pembuktian.

13. Jawab: 198.


Katakan sebuah pewarnaan baik jika memenuhi kriteria soal, dan tidak
baik jika tidak memenuhi kriteria soal.
Perhatikan bahwa pada papan yang kosong, ada 9 cara untuk mewar-
nai sebuah kotak dengan warna biru. Lalu, ada 4 cara mewarnai kotak
lain dengan warna biru sehingga pewarnaan itu masih mungkin baik.
Namun, karena kedua kotak tersebut bisa ditukar, banyaknya cara me-
warnai 2 kotak dengan warna biru sehingga pewarnaan tersebut masih
mungkin baik adalah 92·4 = 18. Selanjutnya, perhatikan kalau ada 2 ko-
tak sedemikian sehingga ada satu buah kotak berwarna biru di kolom
dan barisnya (sebut keduanya kotak jenis a), ada 4 kotak sedemikian se-
hingga ada satu buah kotak berwarna biru di kolom atau barisnya, tetapi

117
Buku KTO Matematika 1

tidak keduanya (sebut ini kotak jenis b), dan ada 1 kotak sedemikian se-
hingga tidak ada kotak berwarna biru di kolom maupun barisnya (sebut
ini kotak jenis c). Bayangkan sebuah papan yang sudah diwarnai 2 ko-
taknya dengan warna biru dan pewarnaannya masih mungkin baik. Jika
kita awalnya mewarnai sebuah kotak jenis a dengan warna merah, ma-
ka ada 4 kotak lain yang bisa kita warnai dengan warna merah sehingga
pewarnaannya masih baik. Karena ada 2 kotak jenis a, ada 2 · 4 = 8 cara
untuk mewarnai 2 kotak dengan warna merah dengan cara ini. Jika kita
awalnyamewarnai kotak jenis b dengan warna merah, ada 3 kotak lain
yang kita bisa warnai merah. Karena ada 4 kotak jenis b, ada 3 · 4 = 12
cara untuk mewarnai 2 kotak dengan warna merah dengan cara ini. Jika
kita awalnya mewarnai kotak jenis c dengan warna merah, ada 2 kotak
lain yang bisa kita warnai dengan warna merah. Karena ada 1 kotak
jenis c, ada 1 · 2 = 2 cara untuk mewarai 2 kotak dengan warna merah
dengan cara ini. Perhatikan bahwa setiap pewarnaan yang baik bisa di-
representasikan dengan 2 cara berdasarkan kotak warna merah mana
yang diwarnai pada awalnya. Akibatnya, banyaknya pewarnaan 2 kotak
merah jika 2 kotak biru sudah diwarnai pada awalnya (dan pewarnaan-
nya mungkin baik) adalah 8+12 2
+2
= 11. Karena terdapat 18 cara untuk
mewarnai 2 kotak biru sedemikian sehingga pewarnaannya mungkin ba-
ik, banyaknya cara mewarnai papan berukuran 3 × 3 secara keseluruhan
adalah 18 · 11 = 198.
Komentar:

a) Mempertimbangkan bahwa hanya 7% kontestan yang mampu me-


ngerjakan soal ini dengan tepat, soal ini bisa dikatakan sangat sulit.
b) Observasi utama dalam menjawab soal ini adalah membagi 7 ko-
tak yang tersisa berdasarkan tiga karakteristik yang bisa memper-
mudah kita dalam mencari banyaknya pewarnaan yang memenuhi
syarat. Ide seperti ini bisa diperoleh dari latihan soal-soal kombina-
torika secara intensif.

14. Jawab: 18.


Misalkan (x − y)2 = t. Jelas bahwa t ≥ 0 dan (x + y)2 = (x − y)2 + 4x y = t + 4.
Akibatnya, pertidaksamaan pada soal ekuivalen dengan (t + 8)(t + 2) ≥
At ⇐⇒ t2 + (10 − A)t + 16 ≥ 0 ⇐⇒ (t − 4)2 + (18 − A)t > 0. Jika A > 18,
didapat t = 4 berarti (18 − A)t ≥ 0. Namun, 18 − A < 0 dan t ≥ 0. Sebagai

118
Pembahasan Kontes Oktober 2015

konsekuensi, tidaklah mungkin bahwa A > 18. Jika A = 18, pertidaksa-


maan selalu benar karena (t − 4)2 ≥ 0 selalu benar. Jadi, nilai A terbesar
yang mungkin adalah 18.
Komentar:

a) Terdapat 21% peserta yang menjawab soal ini dengan benar, me-
nandakan bahwa soal ini tergolong cukup sulit.
b) Pemisalan (x − y)2 = t adalah penting karena dapat mengurangi
kompleksitas soal. Setelah itu. nilai 18 bisa diperoleh dengan me-
mandang diskriminan persamaan t2 + (10 − A)t + 16 = 0, yang harus
sama dengan 0.

119
Buku KTO Matematika 1

Bagian B

1. Jawab: (b) g(x) = x + 1 ∀ x ∈ R dan g(x) = 1 − x ∀ x ∈ R.


Misalkan H(x, y) menyatakan persamaan f (x) f (y) − f (x y) = 2(x + y + 1).

a) i. H(0, 0) adalah f (0)2 − f (0) = 2 ⇐⇒ ( f (0) − 2)( f (0) + 1) = 0. Aki-


batnya, f (0) = 2 atau f (0) = −1.
ii. Jika f (0) = 2, H(x, 0) adalah f (x) f (0) − f (0) = 2(x + 1) ⇐⇒ 2 f (x) −
2 = 2x + 2 ⇐⇒ f (x) = x + 2. Perhatikan kalau H(x, y) adalah
(x + 2)(y + 2) − (x y + 2) = 2(x + y + 1) ⇐⇒ 2x + 2y + 2 = 2(x + y + 1).
Akibatnya, f (x) = x + 2 merupakan solusi.
iii. Jika f (0) = −1, H(x, 0) adalah f (x) f (0) − f (0) = 2(x + 1) ⇐⇒
− f (x) + 1 = 2(x + 1) ⇐⇒ f (x) = −(2x + 1). Akibatnya, f (1) = −3.
H(1, 1) adalah (−3)(−3) − (−3) = 2(1 + 1 + 1) ⇐⇒ 12 = 6, yang
merupakan pernyataan yang salah. Akibatnya, f (x) = −(2x + 1)
bukan solusi. Berarti, tidak ada fungsi yang memenuhi H(x, y)
dengan f (0) = −1.
b) Misalkan T(x, y) menyatakan persamaan g(x)g(y) − g(x + y) = x y.
T(0, 0) adalah g(0)(g(0) − 1) = 0, sehingga g(0) = 0 atau g(0) = 1.
Jika g(0) = 0, didapat T(x, 0) adalah g(x) = 0. Jika kita menyubsti-
tusi hasil tersebut ke T(x, y), diperoleh x y = 0, yang tidak benar.
Akibatnya, haruslah g(0) = 1.
Lihat lagi bahwa T(1, −1) adalah g(1)g(−1) − g(0) = −1, sehingga di-
punyai g(1)g(−1) = 0.
Jika g(1) = 0, T(1, y) adalah − g(1 + y) = y, sehingga g(y) = 1 − y. Jika
kita menyubstitusi hasil tersebut ke T(x, y) diperoleh (1 − x)(1 − y) −
(1 − x − y) = x y, sehingga fungsi ini memenuhi.
Jika g(−1) = 0, T(−1, y) adalah − g(y − 1) = − y, maka g(y) = y + 1.
Jika kita menyubstitusi hasil tersebut ke T(x, y), diperoleh (1+ x)(1+
y) − (x + y + 1) = x y, sehingga fungsi ini memenuhi.
Sebagai kesimpulan, semua solusi yang memenuhi adalah g(x) =
x + 1 ∀ x ∈ R dan g(x) = 1 − x ∀ x ∈ R.

Komentar:

a) Beberapa peserta mengasumsikan bahwa fungsi tersebut haruslah


berbentuk polinomial. Asumsi tersebut tidaklah valid karena tidak

120
Pembahasan Kontes Oktober 2015

semua fungsi berbentuk polinomial (tetapi semua polinomial adalah


fungsi).
b) Melakukan substitusi nilai identitas biasanya adalah ide yang baik
saat menyelesaikan soal persamaan fungsi. Sebagai contoh, substi-
tusi nilai y = 0 biasanya berguna saat terdapat suku f (x + y), dan
substitusi nilai y = 1 biasanya berguna saat terdapat suku f (x y).
Selain itu, substitusi nilai yang menghasilkan suku konstanta juga
adalah ide yang baik. Contohnya ialah melakukan substitusi y = 0
atau x = 0 saat terdapat suku f (x y). Namun, saat melakukan sub-
stitusi perlu diingat bahwa nilai yang dimasukkan ada di dalam
domain fungsi tersebut.

2. Jawab: (b) Tidak.


(a) Kita nyatakan Vp (n) sebagai fungsi p-adic, yaitu bilangan cacah
terbesar k sedemikian sehingga p k habis membagi n. Perhatikan ka-
lau ab · bc · cd · de · e f · f g · ga = (abcde f g)2 merupakan bilangan ku-
bik. Jika ada bilangan prima p sehingga 3 - Vp (abcde f g), kita punya
3 - 2.Vp (abcde f g) = Vp ((abcde f g)2 ). Padahal, (abcde f g)2 adalah bi-
langan kubik. Hal ini berarti untuk setiap bilangan prima p, berlaku
3|Vp (abcde f g), yang mengakibatkan abcde f g haruslah merupakan bi-
langan kubik. Perhatikan pula bahwa kalau untuk setiap bilangan pri-
ma p, 3|Vp (abcde f g), kita punya 3|Vp (ab), 3|V³p (cd), 3´|Vp (e f ), sehingga
abcde f g
3|Vp (abcde f g) − Vp (ab) − Vp (cd) − Vp (e f ) = Vp ab· cd · e f = Vp (g). Jadi, g
adalah bilangan kubik.
Dengan cara yang sama, a, b, c, d, e, dan f juga merupakan bilangan
kubik.
(b) Tidak; tinjau kasus penyangkal (a, b, c, d, e, f ) = (2, 4, 2, 4, 2, 4).
Komentar:

a) Banyak peserta yang berhasil berkonklusi bahwa g2 adalah bilang-


an kubik, tetapi gagal menunjukkan bahwa g haruslah merupakan
bilangan kubik.
b) Inspirasi di balik penemuan contoh penyangkal untuk soal bagian
(b) didasarkan pada pembuktian bagian (a), yakni melihat mengapa
ide pembuktian pada bagian (a) tidak berlaku untuk kasus pada
soal bagian (b).

121
Buku KTO Matematika 1

c) Beberapa peserta tidak meninjau keseluruhan kasus (dengan kata


lain hanya mempertimbangkan kasus-kasus tertentu). Hal terse-
but tidak dibenarkan karena kita perlu mengobservasi semua nilai
a, b, . . . , g yang memenuhi soal.

3. Bukti.

120°
H

I
O
A B

Karena segitiga ABC lancip, ∠BCH = 90° − ∠CBA, ∠CBH = 90° − ∠BC A.
Akibatnya, ∠BHC = 180° − ∠BCH − ∠CBH = ∠CBA + ∠BC A = 180° −
∠BAC = 120° maka ∠BAC = 60°. Perhatikan bahwa ∠BIC = ∠BAC +
∠ ACI + ∠ ABI = ∠BAC +∠ ABC
2
+∠BC A
+ ∠BAC
2 = 180° 60°
2 + 2 = 120°. Selan-
jutnya, perhatikan bahwa ∠BOC = 2∠BAC = 120°, sehingga ∠BHC =
∠BIC = ∠BOC, mengakibatkan fakta bahwa B, H, I, C, O berada di sa-
tu lingkaran. Dengan kata lain, lingkaran luar segitiga H IO melalui B
dan C.
Komentar:

a) Menyadari bahwa pernyataan yang ingin dibuktikan bisa ditulis


ulang menjadi “buktikan B, H, I, C, dan O terletak pada satu
lingkaran” adalah salah satu observasi kunci dari soal ini. Seca-
ra umum, mencari bentuk yang ekuivalen dengan soal cenderung

122
Pembahasan Kontes Oktober 2015

bersifat membantu dalam menjawab soal. Pembaca dapat menguji


kebenaran pernyataan tersebut dengan melihat penyelesaian dari
soal-soal yang lain pada buku ini, sebagai contoh soal uraian nomor
4 pada set soal ini.

4. Bukti.

a) Sebuah baris dilabeli hitam jika banyaknya kotak hitam lebih besar
dibandingkan banyaknya kotak putih di baris tersebut, dan dilabe-
li putih jika kondisi sebaliknya berlaku. Pewarnaan untuk kolom
didefinisikan dengan cara yang serupa. Perhatikan bahwa setiap
baris maupun kolom mempunyai warna karena banyaknya kotak
di setiap baris maupun kolom adalah ganjil, sehingga tidak mung-
kin banyaknya kotak hitam dan putih di suatu baris ataupun kolom
adalah sama.
Perhatikan bahwa suatu kotak adalah spesial jika dan hanya jika
kotak itu terletak pada baris dan kolom yang berwarna sama. Mi-
salkan a, b, c, d adalah banyak baris hitam, baris putih, kolom hi-
tam, kolom putih secara berturut-turut. Perhatikan bahwa banyak-
nya kotak spesial adalah ac + bd. Jika di antara empat variabel ter-
sebut tidak ada yang bernilai 0, diperoleh ac + bd ≥ a · 1 + b · 1 = 2n + 1,
yang berakibat kepada terjaminnya eksistensi 2n + 1 kotak spesial.
Jika tidak, tanpa mengurangi keumuman, misalkan c = 0. Ini berar-
ti d = 2n + 1 dan terdapat lebih banyak kotak putih dibanding kotak
hitam di papan. Akibatnya, b ≥ 1, sehingga banyak kotak spesial
adalah bd ≥ 2n + 1 (ada setidaknya 2n + 1 kotak spesial. Akibatnya,
terbukti bahwa pada papan tersebut terdapat setidaknya 2n + 1 ko-
tak spesial.
b) Perhatikan konfigurasi berikut: pada baris pertama, 2n kotak di ki-
ri diwarnai hitam, kotak terakhir diwarnai putih; pada baris ke 2
hingga baris ke n + 1, n kotak di kiri diwarnai hitam dan n + 1 kotak
sisanya diwarnai putih; pada baris ke n + 2 sampai baris ke 2n + 1, n
kotak pertama diwarnai putih, n kotak selanjutnya diwarnai hitam,
dan kotak terakhir diwarnai putih. Perhatikan kalau ada satu baris
hitam, 2n baris putih, 2n kolom hitam, dan 1 kolom putih. Akibat-
nya, ada 1 · 2n + 2n · 1 = 4n kotak spesial, sesuai yang diinginkan.

Komentar:

123
Buku KTO Matematika 1

a) Fakta bahwa tidak ada peserta yang menjawab soal ini dengan sem-
purna menandakan bahwa soal ini tergolong sulit.
b) Observasi besar dalam menjawab soal ini adalah mendefinisikan
ulang sebuah kotak spesial agar hal yang ingin dibuktikan menjadi
lebih intuitif. Pada solusi di atas, kotak spesial ditafsirkan menjadi
kotak yang terletak pada baris dan kolom yang berwarna sama.

124
6 Kontes November 2015

6.1 Soal-Soal
Bagian A

Tuliskan jawaban akhir setiap soal di lembar jawaban. Setiap soal bernilai 1
angka. Jawaban Anda harus berupa bilangan bulat dari 0 sampai 999.

1. (1006) Misalkan a, b, c, d, e adalah lima suku berurutan dari sebuah


barisan aritmetika dan a + b + c + d + e = 30. Ada tepat satu variabel dari
a, b, c, d, e yang nilainya bisa ditentukan hanya dengan informasi ini.
Tentukanlah nilai tersebut.

2. (1013) Suatu grup paduan suara terdiri dari beberapa anak laki-laki dan
perempuan mengamen untuk menggalang dana untuk mendanai kegiat-
an konser mereka. Pada mulanya, 40% grup ini adalah anak laki-laki.
Namun, setelah dua anak laki-laki keluar dari grup dan dua anak pe-
rempuan masuk ke grup, anak laki-laki menjadi 30% grup. Tentukan
banyak anak laki-laki pada mulanya.

3. (1020) Diberikan sebuah persegi ABCD dengan panjang sisi 10. Diberik-
an segitiga sama kaki ABK dengan alas AK = BK sehingga luas daerah
bersama segitiga ABK dan persegi ABCD ini adalah 80. Tentukan jarak
titik K ke sisi AB.

4. (1027) Tentukan banyaknya bilangan asli n ∈ {1, 2, 3, . . . , 1000} yang me-


menuhi 7 habis membagi n2 + 5.

5. (1034) Panjang jari-jari sebuah


p
segi-n beraturan adalah 1 dan rasio nilai
keliling dan luasnya adalah 4 3 3 . Tentukan n.

125
Buku KTO Matematika 1

6. (1041) Pada sebuah kotak terdapat 2 apel, 3 mangga, 3 jambu, dan 4


melon. Akan dipilih 4 buah secara acak dari kotak tersebut. Peluang
bahwa ada terpilih dua buah yang sama jenis dapat ditulis dalam bentuk
m
n dengan m dan n adalah dua bilangan asli yang relatif prima. Hitung
m + n.

7. (1048) Tentukan banyaknya pasangan terurut bilangan real (a, b, c) se-


hingga
a4 + b4 + c4 + 1 = 4abc.

8. (1055) Tentukan dua digit terakhir dari penulisan basis 4 untuk bilangan
2015
20152015 .

9. (1062) Tentukan banyaknya bilangan asli n sehingga polinomial 1 + x +


x2 + · · · + x n habis membagi polinomial x1000 − 1.

10. (1069) Misalkan ABCD adalah segiempat konveks sehingga ∠ ABD =


40°, ∠DBC = 70°, ∠BD A = 80°, dan ∠BDC = 50°. Tentukan besar ∠C AD
dalam derajat.

11. (1076) Tentukan banyaknya bilangan kuadrat sempurna yang habis


membagi
1! · 2! · 3! · . . . · 8! · 9!.

12. (1083) Jajargenjang yang dibatasi oleh garis y = ax + c, y = ax + d, y = bx +


c, dan y = bx + d memiliki luas 18. Jajargenjang yang dibatasi oleh garis
y = ax + c, y = ax − d, y = bx + c, y = bx − d memiliki luas 72. Diketahui
bahwa a, b, c, dan d merupakan bilangan asli. Tentukan nilai terkecil
yang mungkin untuk a + b + c + d.

13. (1090) Fungsi f : R+ → R memenuhi sifat bahwa


x y
f (x) f (y) − f (x y) = +
y x
untuk semua bilangan real positif x dan y. Hasil jumlah semua nilai
yang mungkin untuk f (10) dapat ditulis dalam bentuk m/n dengan m, n
merupakan bilangan asli yang relatif prima. Hitung m + n.

14. (1097) Pada segitiga ABC, diketahui ∠ ABC = 45°. Titik D pada sisi BC
sehingga CD = 2BD dan ∠D AB = 15°. Cari besar sudut ∠ ACB dalam
derajat.

126
Soal-Soal Kontes November 2015

Bagian B

Tuliskan jawaban beserta langkah pekerjaan Anda pada lembar jawaban.


Selain jawaban akhir, Anda perlu menuliskan argumentasi atau langkah-
langkah untuk memperoleh jawaban akhir tersebut. Gunakan halaman yang
berbeda untuk setiap soal yang berbeda. Setiap soal bernilai 7 angka.

1. (1118) Diberikan bilangan asli n. Empat orang bermain hompimpa untuk


menentukan pasangannya bermain badminton. Pada satu kali hompim-
pa, setiap orang secara acak menunjukkan telapak tangan atau pung-
gung tangan, masing-masing dengan peluang yang sama. Pilihan setiap
orang independen. Dua orang menjadi pasangan jika mereka menunjuk-
kan sisi tangan yang sama dan berbeda dengan dua orang lainnya. Jika
pasangan tidak terbentuk, dilakukan hompimpa lagi hingga akhirnya
pasangan terbentuk. Pilihan sisi tangan tiap-tiap orang pada hompimpa
berikutnya juga independen dari sebelumnya. Tentukan peluang dibu-
tuhkan tepat n buah hompimpa.

2. (1125) Titik K terletak pada diagonal BD dari jajargenjang ABCD. Ga-


ris AK memotong garis BC dan CD di titik L dan M, berturut-turut.
Buktikan bahwa
AK 2 = LK · K M.

3. (1132)

a) Tunjukkan bahwa terdapat bilangan asli n > 2015 sehingga

n2 + n + 1
20152 + 2015 + 1
merupakan bilangan asli.
b) Apakah ada tak hingga banyaknya bilangan asli n yang memenuhi
syarat pada soal (a)?

4. (1078) Tentukan semua fungsi f : R → R demikian sehingga

f (x + f (y)) = f (x) + b yc

untuk semua bilangan real x dan y.


Catatan: Notasi bac menyatakan bilangan bulat terbesar yang tidak lebih
dari a, misalnya bπc = 3 dan b2c = 2.

127
Buku KTO Matematika 1

6.2 Pembahasan
Bagian A

1. Jawab: 6.
Misalkan beda dari barisan aritmetika tersebut adalah x. Akibatnya,
kita punya b = a + x, c = a + 2x, d = a + 3x, dan e = a + 4x, sehingga 30 =
a + b + c + d + e = 5a + 10x, atau 6 = a + 2x = c. Dari sini kita peroleh bahwa
variabel yang dimaksud adalah c, yang nilainya adalah 6.
Komentar:

a) Soal ini dapat diselesaikan oleh 76% dari keseluruhan peserta. Hal
tersebut menunjukkan bahwa para kontestan pada umumnya tidak
merasa kesulitan dalam mengerjakan soal ini.
b) Sifat soal ini lebih condong kepada pengujian pemahaman konsep
mengenai barisan aritmetika. Pembaca hanya membutuhkan pe-
ngetahuan dasar tentang sifat-sifat barisan aritmetika untuk dapat
mengerjakan soal ini.

2. Jawab: 8.
Tanpa mengurangi keumuman, andaikan banyaknya anggota grup pada
mulanya adalah x. Oleh karena itu, banyaknya anggota laki-laki ada-
lah 0, 4x dan banyaknya anggota perempuan adalah 0, 6x. Setelah ada-
nya keluar-masuk anggota, banyaknya anggota laki-laki dan perempuan
adalah 0, 4x − 2 dan 0, 6x + 2, berturut-turut. Berdasarkan fakta bahwa
jumlah anggota laki-laki sekarang adalah 30% anggota grup, kita punya
0, 4x − 2 = 0, 3x, yang ekuivalen dengan x = 20. Akibatnya, banyaknya
anggota laki-laki pada mulanya adalah 0, 4 × 20 = 8. Kita selesai.
Komentar:

a) Soal yang bisa diselesaikan oleh 93% peserta ini jelas merupakan
soal yang sangat mudah. Pengerjaan soal ini hanya memerlukan
teknik substitusi dan eliminasi persamaan aljabar, yang sudah dia-
jarkan di tingkat sekolah menengah pertama (SMP).
b) Pembaca yang masih belum bisa mengerjakan soal ini dengan tepat
disarankan untuk mengulang kembali materi aljabar dasar yang
diajarkan di sekolah sebelum melanjutkan pembelajaran pada ting-
kat yang lebih tinggi.

128
Pembahasan Kontes November 2015

3. Jawab: 25.

A D

B C

Perhatikan bahwa titik K harus terletak pada sisi yang berlawanan de-
ngan titik A terhadap garis CD. Selanjutnya, misalkan L dan M ada-
lah titik-titik potong garis K A dan K B, berturut-turut dengan garis CD.
Karena luas segiempat ABCD dan ABML adalah 100 dan 80, berturut-
turut, diperoleh luas segitiga ADL dan BCM masing-masing adalah 10.
Akibatnya, DL = MC = 2, dan LM = 6. Misalkan x adalah jarak titik
K ke sisi AB. Sebagai akibat, jarak titik K ke sisi LM adalah x − 10.
Namun, segitiga K LM sebangun dengan segitiga K AB, sehingga berla-
x
ku AB = xLM
−10
, mengakibatkan x = 25 dengan manipulasi bentuk aljabar
sederhana. Kita selesai.
Komentar:

a) Soal ini dapat diselesaikan oleh 53% dari total peserta, sehingga
bisa dikategorikan sebagai soal yang cukup mudah. Metode penye-
lesaian yang digunakan dalam soal ini pun cukup standar, yakni
kesebangunan dan penghitungan luas bidang datar.
b) Pembaca yang belum mendapatkan jawaban yang benar direkomen-
dasikan untuk mempelajari (atau mempelajari ulang bagi yang su-
dah belajar) sifat-sifat kesebangunan, karena hal tersebut walau-
pun relatif sederhana tetapi sangat berguna dalam menyelesaikan
soal-soal geometri.

4. Jawab: 286.
Perhatikan bahwa soal ini sama dengan mencari banyaknya bilangan as-
li n sedemikian sehingga n2 bersisa 2 ketika dibagi 7. Kemudian, untuk
n yang bersisa 1 ketika dibagi 7, n2 juga bersisa 1 ketika dibagi 7 (jika

129
Buku KTO Matematika 1

pembaca belum mendapati, misalkan n = 7k + 1 dengan k merupakan bi-


langan bulat). Dengan cara yang serupa, n2 bersisa 4, 2, 2, 4, 1, dan 0
ketika dibagi 7 untuk n yang bersisa 2, 3, 4, 5, 6, dan 0 ketika dibagi
7, berturut-turut. Akibatnya, kita tinggal mencari banyaknya bilangan
asli n yang lebih kecil atau sama dengan 1000 dan memenuhi n bersisa 3
atau 4 ketika dibagi 7. Banyaknya bilangan asli n yang demikian adalah
286.
Komentar:

a) Soal ini adalah penerapan dari konsep dasar modulo, atau sisa pem-
bagian. Fakta bahwa 51% peserta mampu menjawab soal ini dengan
benar memberikan sugesti bahwa sebagian besar kontestan sudah
cukup menguasai soal-soal teori bliangan yang berkaitan dengan si-
sa pembagian.
b) Untuk mendapatkan pelajaran mengenai penggunaan teknik si-
sa pembagian dalam mengerjakan soal-soal olimpiade mate-
matika, pembaca dipersilakan untuk merujuk kepada laman
bit.ly/ktobuku1mod.

5. Jawab: 6.
Misalkan segi-n beraturan tersebut adalah A 1 A 2 . . . A n , dan α =
∠ A 1 O A 2 , dengan O adalah titik pusat lingkaran luar segi-n tersebut.
Kita dapatkan
p luas segi-n tersebut adalah 12 n sin α, dan kelilingnya
p ada-
lah
p
n 2 − 2 cos α . Dengan manipulasi aljabar, diperoleh 2 − 2 cos α=
2 3 4 2
3 sin α, yang mengakibatkan 2−2 cos α = 3 (1−cos α). Dengan demikian
cos α = 1 atau cos α = 12 . Namun, cos α 6= 1 (verifikasi sendiri). Akibatnya,
cos α = 12 , dan α = π6 , yang ekuivalen dengan n = 6.
Komentar:

a) Fakta bahwa 67% peserta mampu mengerjakan soal ini dengan be-
nar memberi saran bahwa soal ini cukup mudah.
b) Sebenarnya, nilai n = 6 bisa didapatkan dengan teknik mencoba
(atau trial and error). Namun, perlu diingat bahwa teknik terse-
but tidak valid dalam pengerjaan soal yang berbentuk uraian ka-
rena cara tersebut hanya memberikan sebuah kemungkinan solusi;
padahal, yang kita inginkan adalah semua nilai n yang memenuhi.

130
Pembahasan Kontes November 2015

6. Jawab: 102.
Perhatikan bahwa banyaknya ruang sampel pengambilan adalah 12
¡ ¢
4 =
495. Untuk mencari banyaknya pengambilan di mana terdapat dua bu-
ah dengan jenis sama yang terambil, kita dapat menghitung banyaknya
pengambilan di mana tidak ada dua buah berjenis sama yang terambil
(jika ini terjadi, keempat buah yang terambil harus memiliki jenis yang
berbeda), kemudian mengurangkan 495 dengan hasil tersebut. Padahal,
banyaknya pengambilan yang demikian adalah 2 × 3 × 3 × 4 = 72, sehing-
ga peluang yang diinginkan adalah 495 −72 423 47
495 = 495 = 55 . Dengan demikian,
m + n = 47 + 55 = 102. Kita selesai.
Komentar:

a) Mempertimbangkan bahwa soal ini adalah penerapan dari prinsip


dasar peluang serta prinsip komplemen, adalah cukup mengejutkan
bahwa hanya 24 % kontestan yang mendapatkan jawaban benar un-
tuk soal ini.
b) Hal yang membuat solusi soal ini menjadi jauh lebih mudah adalah
utilisasi dari prinsip komplemen, yakni menghitung banyaknya ke-
jadian di mana suatu hal tidak terjadi untuk mendapatkan banyak-
nya kejadian di mana suatu hal terjadi jika diketahui banyaknya
ruang sampel.
c) Pembaca dapat merujuk kepada buku kombinatorika yang ditulis
pada kolom komentar di soal nomor 1 KTO Juni untuk mengetahui
lebih jelas mengenai prinsip komplemen dan kegunaannya dalam
mengerjakan soal-soal kombinatorika di olimpiade matematika.

131
Buku KTO Matematika 1

7. Jawab: 4.

Jika (a, b, c) adalah solusi, dipunyai abc > 0 karena a4 + b4 + c4 ≥ 1 > 0.


Kemudian, pandang bahwa tidak boleh ada salah satu di antara a, b,
dan c yang sama dengan 0. Menurut ketaksamaan Rataan Aritmetik-
Rataan Geometrik (AM-GM), a4 + b4 + c4 + 1 ≥ 4abc, dan kesamaan terjadi
saat a4 = b4 = c4 = 1. Untuk setiap solusi yang memenuhi, ruas kiri
haruslah bernilai 4, sehingga abc = 1. Nilai tersebut benar saat (a, b, c) =
(1, 1, 1) atau (a, b, c) = (1, −1, −1) dan permutasinya. Akibatnya, terdapat
4 pasangan terurut bilangan real yang demikian.

Komentar:

a) Berdasarkan fakta bahwa 51% dari semua peserta mampu menger-


jakan soal ini dengan benar, bisa dikatakan bahwa soal ini cende-
rung cukup mudah.
b) Kenyataannya, soal ini bisa juga diselesaikan dengan teknik men-
coba (trial and error), tetapi rentan terhadap ketidaktelitian yang
diakibatkan karena kurangnya solusi.
c) Kesalahan yang ditemukan pada beberapa kasus adalah mengang-
gap persamaan a4 = b4 = c4 = 1 berakibat kepada a, b, c = ±1, se-
hingga terdapat 8 solusi. Namun, hal ini salah karena tidak se-
mua dari kedelapan solusi memenuhi persamaan pada soal. Untuk
menghindari kesalahan yang demikian, pembaca disarankan untuk
memverifikasi setiap solusi yang didapat pada pencarian solusi dari
persamaan atau sistem persamaan. Langkah ini, meskipun terlihat
tidak berguna, sangatlah esensial.

8. Jawab: 33.

Kalau diperhatikan secara saksama, soal ini meminta kita untuk menca-
2015
ri sisa pembagian 20152015 oleh 16 (silakan verifikasi sendiri). Karena
2015 2015
2015 ≡ −1 mod 16, kita peroleh 20152015 ≡ (−1)2015 ≡ −1 mod 16,
sehingga dua angka terakhir dari penulisan basis empatnya adalah 33.

Komentar:

a) 35% dari semua peserta mampu menjawab soal ini dengan baik,
sehingga soal ini bisa dikatakan cukup sulit.

132
Pembahasan Kontes November 2015

b) Pengamatan yang menjadi kunci, sekaligus menjadi titik kesulitan,


dari soal ini adalah bagaimana soal bisa diubah menjadi menghi-
tung sisa pembagian bilangan pada soal dengan 16. Setelah itu,
semuanya mengikuti.

9. Jawab: 15.
Misalkan n adalah bilangan asli yang memenuhi syarat soal. Aki-
n+1
batnya, x x−1−1 habis membagi x1000 − 1, atau x n+1 − 1 habis memba-
gi (x1000 − 1)(x − 1). Jika kita ambil x = 2, diperoleh 2n+1 − 1 memba-
gi 21000 − 1, atau gcd (2n+1 − 1, 21000 − 1) = 2n+1 − 1. Padahal, kita tahu
bahwa gcd (2n+1 − 1, 21000 − 1) = 2gcd (n+1,1000)−1 , sehingga didapat bahwa
n + 1|1000. Untuk setiap n yang memenuhi n + 1|1000, terlihat bahwa
syarat soal terpenuhi karena x n+1 − 1|(x1000 − 1)|(x1000 − 1)(x − 1). Jadi, se-
mua n yang memenuhi adalah faktor positif dari 1000 yang bukan 1, dan
banyaknya adalah 15.
Komentar:

a) 18% dari semua peserta mampu menjawab soal ini dengan baik,
sehingga soal ini termasuk dalam kategori soal sulit.
b) Pengetahuan bahwa gcd (a n − 1, a m − 1) = agcd (n,m)−1 memainkan
peran yang penting dalam alternatif solusi di atas. Pembaca dia-
jak untuk membuktikan lema kecil tersebut.

10. Jawab: 30.

B
40°
70°
80°
50°
D C

133
Buku KTO Matematika 1

Misalkan ∠BAC = α. Perhatikan bahwa


AB AB/BD sin 80° sin (70° − α)
= = = .
BC BC/BD sin 50° sin α

Ruas kiri bisa disederhanakan menjadi 2 sincos 40° cos 40°


40° = 2 sin 40° = sin 40°
sin 30° ,
sehingga kita peroleh 2 sin 40° sin α = 2 sin 30° sin (70° − α), yang ekuiva-
len dengan cos (40° + α) = cos (100° − α) (gunakan identitas trigonometri
cos − cos). Karena α < 60°, didapat α = 30°. Akibatnya, besar sudut C AD
dalam derajat adalah 30.

134
Pembahasan Kontes November 2015

Komentar:

a) Soal ini bisa dikerjakan oleh 22 % dari keseluruhan kontestan, me-


nandakan bahwa soal ini tergolong sulit.
b) Salah satu pendekatan yang bisa dimanfaatkan untuk menjawab
soal seperti ini adalah dengan trigonometri, seperti yang disajik-
an pada solusi. Solusi ini cenderung mudah dan jelas alurnya, te-
tapi memerlukan ketelitian dan kecermatan dalam memanipulasi
bentuk trigonometri. Alternatif yang bisa digunakan untuk me-
nyelesaikan soal ini adalah dengan pendekatan geometri Euclidean
(tanpa trigonometri maupun analitik; hanya memanfaatkan kese-
bangunan, properti-properti sudut dan garis, dan lain-lain), yang
relatif penuh trik dan terlihat elegan. Pembaca disarankan untuk
mencoba mencari solusi untuk masalah ini dengan metode Euclide-
an.

11. Jawab: 672.


Perhatikan bahwa 1! × 2! × · · · × 8! × 9! = 230 × 313 × 55 × 73 (verifikasi dise-
rahkan kepada pembaca). Akibatnya, banyaknya pembagi dari bilangan
tersebut yang merupakan bilangan kuadrat sempurna harus berbentuk
2a × 3b × 5 c × 7d , dengan a, b, c, dan d bilangan genap yang memenuhi
0 ≤ a ≤ 30, 0 ≤ b ≤ 13, 0 ≤ c ≤ 5, dan 0 ≤ d ≤ 3. Menurut prinsip perkalian,
terdapat 16 × 7 × 3 × 2 = 672 bilangan yang demikian.
Komentar:

a) Sebanyak 33% peserta dapat menyelesaikan soal ini dengan benar.


Soal ini bisa digolongkan sebagai soal yang cukup sulit, meskipun
cara yang digunakan relatif elementer.
b) Tidak ada cara khusus yang dipakai untuk menyelesaikan soal ini.
Hal yang utama dalam menjawab soal ini ialah kecermatan dan ke-
efektifan dalam menyatakan faktorisasi prima dari bentuk pada so-
al.

12. Jawab: 16.


Perhatikan bahwa kita bisa memisalkan a > b tanpa mengurangi keu-
muman soal (lihat kalau a = b tidak mungkin terjadi). Kita cari dahu-
lu koordinat titik-titik sudut setiap jajar genjang dengan menyelesaikan

135
Buku KTO Matematika 1

empat persamaan linear dua variabel.³Sebagai contoh, ´ jajar


³ genjang
´ per-
tama memiliki koordinat (0, c), (0, d), ad−−bc , ada−−bbc , dan ac−−db , aca−−bd
b . Se-
lanjutnya, kita hitung luas jajar genjang tersebut dengan teknik Shoelace
(mencari determinan matriks yang entri-entrinya adalah koordinat dari
jajar genjang tersebut). Setelah melakukan penyederhanaan, diperoleh
dua persamaan: (c − d)2 = 18(a − b) dan (c + d)2 = 72(a − b). Akibatnya,
c + d = 2(c − d) atau c + d = −2(c − d), yang ekuivalen dengan c = 3d atau
d = 3c. Jika c = 3d, didapat 2d 2 = 9(a − b). Lihat bahwa 9 haruslah
membagi d 2 , sehingga kita bisa menulis d = 3d 1 untuk suatu bilangan
asli d 1 . Jadi, (a, b, c, d) = (2d 12 + b, b, 9d 1 , 3d 1 ), dan nilai minimum dari
a + b + c + d = 16, yakni saat d 1 = b = 1. Untuk kasus kedua, dengan cara
yang sama diperoleh (a, b, c, d) = (2c21 + b, b, 3c 1 , 9c 1 ). Ternyata, nilai mi-
nimum dari a + b + c + d untuk kasus ini juga 16, yakni ketika c 1 = b = 1.
Akibatnya, nilai minimum dari a + b + c + d adalah 16. Kita selesai
Komentar:

a) Berdasarkan fakta bahwa hanya 16 % dari keseluruhan peserta


mampu menyelesaikan soal ini, soal ini tergolong sulit.
b) Pengetahuan akan teorema Shoelace adalah penting dalam menye-
lesaikan soal ini. Secara alternatif, soal ini dapat dikerjakan dengan
menggunakan pendekatan vektor. Pembaca dipersilakan mencoba-
nya sendiri.

13. Jawab: 111.


Masukkan x = y = 1, didapat f (1)2 − f (1)−2 = 0, atau ( f (1)−2)( f (1)+1) = 0,
sehingga f (1) = 2 atau f (1) = −1. Dapat diperiksa bahwa f (1) = −1 ti-
dak mungkin memenuhi persamaan fungsi tersebut. Akibatnya, harus-
lah f (1) = 2. Kemudian, substitusi y = 1 ke persamaan awal, didapat
f (x) = x + 1x . Ternyata, fungsi tersebut memenuhi syarat soal (silakan
verifikasi sendiri). Akibatnya, jumlahan nilai yang mungkin dari f (10)
adalah 101
10 , dan m + n = 111.
Komentar:

a) Soal persamaan fungsi ini sebenarnya tergolong mudah karena ha-


nya menyang-kut substitusi nilai ke dalam variabel yang terdapat
pada soal. Namun, soal ini hanya mampu dikerjakan dengan benar
oleh 11% dari keseluruhan peserta. Penjelasan yang paling masuk

136
Pembahasan Kontes November 2015

akal adalah kebanyakan kontestan belum familiar dengan materi


persamaan fungsi.
b) Eksistensi soal persamaan fungsi dalam olimpiade matematika pa-
da kenyataannya cukup sering. Sebagai contoh, soal International
Mathematical Olympiad tahun 2015 nomor 5 bertemakan persama-
an fungsi. Oleh karena itu, pemahaman akan materi persamaan
fungsi sangatlah esensial dalam pembelajaran olimpiade matemati-
ka.
c) Untuk memperkaya pengetahuan Anda tentang materi persamaan
fungsi, Anda dapat membaca buku Functional Equations: A Pro-
blem Solving Approach karya B. J. Venkatachala.

14. Jawab: 75.

D E

B
A

Misalkan titik E terletak pada perpanjangan garis AD ke arah D sede-


mikian sehingga ∠ ABE adalah sudut siku-siku. Perhatikan bahwa BD
adalah garis bagi ∠ ABE, sehingga berlaku BD 2 = AB × BE − AD × DE.
AB BE
Selanjutnya, berdasarkan aturan sinus, diperoleh AD = DE = sin 120°
sin 45° . Se-
µ³ ´2 ¶
bagai konsekuensi, BD 2 = AD × DE × sin 120°
sin 45° − 1 = 12 × AD × DE, yang
berakibat kepada BD × DC = 2 × BD 2 = AD × DE. Persamaan terakhir
mengandung arti bahwa ABEC adalah segiempat tali busur, sehingga
besar sudut ACB sama dengan besar sudut AEB, yakni 75 derajat. Kita
selesai.
Komentar:

137
Buku KTO Matematika 1

a) Berdasarkan kenyataan bahwa 40% dari keseluruhan peserta mam-


pu mengerjakan soal ini dengan benar, bisa dikatakan bahwa soal
ini tergolong cukup mudah.
b) Selain solusi di atas, ada banyak penyelesaian yang bisa digunakan
untuk mengerjakan soal ini. Salah satu di antaranya adalah meng-
gunakan persamaan trigonometri, seperti yang diutilisasi pada soal
nomor 10.

138
Pembahasan Kontes November 2015

Bagian B
¡ 5 ¢n−1
1. Jawab: 8 × 38 .
Katakan sebuah ronde hompimpa berhasil jika terbentuk dua pasangan
sebagai hasil ronde tersebut (sekaligus menghentikan hompimpa), dan
gagal jika ronde tersebut tidak menghasilkan dua pasangan (mengha-
ruskan diulangnya hompimpa). Hasil yang ingin kita cari adalah peluang
dibutuhkannya tepat n hompimpa, yang ekuivalen dengan peluang n − 1
hompimpa pertama gagal dan hompimpa setelahnya berhasil. Namun,
yang perlu kita ingat adalah hasil hompimpa pada tiap ronde adalah in-
dependen (yang satu tidak memengaruhi yang lain), sehingga cukup kita
hitung peluang sebuah ronde hompimpa berhasil atau gagal.
Sekarang, kita akan menentukan peluang sebuah ronde hompimpa ber-
hasil. Perhatikan bahwa setiap pemain bisa mengeluarkan telapak ta-
ngan atau punggung tangan. Akibatnya, terdapat 24 = 16 kemungkinan
hasil dari sebuah ronde hompimpa, yang merupakan ruang sampel dari
kejadian tersebut. Selanjutnya, pandang bahwa setiap ronde hompim-
pa adalah berhasil jika dan hanya jika tepat dua orang mengeluarkan
telapak tangan dan sisanya mengeluarkan punggung tangan. Banyak
kemungkinan terjadinya kondisi ini sama saja dengan banyak cara me-
milih 2 orang yang mengeluarkan telapak tangan, yang adalah 42 = 6.
¡ ¢

Sebagai kesimpulan, peluang sebuah ronde hompimpa berhasil adalah


6 3
16 = 8 .

Karena sebuah ronde hompimpa pasti berhasil atau gagal (dan tidak
mungkin keduanya), peluang sebuah ronde hompimpa gagal adalah
1 − 38 = 58 . Akibatnya, peluang dibutuhkannya tepat n hompimpa adalah
¡ 5 ¢n−1 3
8 × 8.
Komentar:

a) Observasi kunci dalam menyelesaikan soal ini adalah penggunaan


fakta bahwa hasil hompimpa adalah independen, atau tidak meme-
ngaruhi satu sama lain, sehingga kita tinggal menghitung peluang
hasil hompimpa berhasil atau gagal untuk satu ronde untuk meng-
hitung peluang yang diminta soal.
b) Beberapa kemungkinan penyelesaian yang menghasilkan nilai
yang tidak sempurna adalah:

139
Buku KTO Matematika 1

i. Tidak atau tidak dapat menjelaskan dengan terperinci dari ma-


na munculnya 42 ; hal ini patut disayangkan mengingat penulis
¡ ¢

solusi sebenarnya mengetahui, hanya saja lupa atau tidak da-


pat mengomunikasikan idenya dalam bentuk tulisan. Dalam
membahasakan solusi, Anda harus beranggapan bahwa pengo-
reksi tidak tahu apa-apa. Jika Anda tidak bisa menjelaskan
dengan jelas dan tepat bagaimana suatu hal bisa terjadi (mes-
kipun Anda mengetahuinya), poin Anda berpotensi untuk diku-
rangi.
ii. Menjawab 83 ; jawaban ini tidak dapat diterima karena per-
nyataan “dibutuhkan tepat n hompimpa” mensyaratkan bahwa
“n − 1 hompimpa pertama tidak berhasil”.

2. Bukti.

A D

M
L
B C

Perhatikan bahwa 4 ADK ∼ 4LBK karena ]DK A = ]BK L (kedua su-


dut saling bertolak belakang) dan ] ADK = ]LBK (kedua sudut saling
dalam berseberangan). Akibatnya, AK/LK = DK/K B.
Kemudian, pandang pula bahwa 4 AK B ∼ 4 MK D (verifikasi diserah-
kan kepada pembaca). Akibatnya, DK/K B = MK/AK. Menggabungkan
dua persamaan tersebut, diperoleh AK/LK = MK/AK, yang ekuivalen
dengan yang diminta soal.
Komentar:

a) Ide pokok dari penyelesaian soal ini adalah dengan mengobservasi


pasangan-pasangan segitiga yang sebangun. Untuk memfasilitasi
Anda dalam pencarian pasangan tersebut, Anda dapat melakukan
teknik angle chasing (pencarian besar sudut dan relasi-relasinya).

140
Pembahasan Kontes November 2015

b) Beberapa kemungkinan penyelesaian yang menghasilkan nilai


yang tidak sempurna adalah:
i. Tidak memberikan alasan yang cukup untuk membuat deduksi
bahwa dua segitiga adalah sebangun; hal ini perlu dihindari,
apalagi pada penyelesaian di mana kesebangunan adalah ba-
gian yang penting. Hal yang sama perlu Anda ingat: anggap
pengoreksi tidak tahu bahwa kedua segitiga itu sebangun, dan
tugas Anda adalah meyakinkan mereka.
ii. Salah menginterpretasi soal; satu-satunya hal yang perlu die-
laborasi adalah bahwa Anda harus membaca soal dengan hati-
hati sebelum mencoba untuk menyelesaikannya.

3. Jawab: (b) ada.


Tinjau bahwa n4 + n2 + 1 = (n2 + n + 1)(n2 − n + 1), sehingga 20154 + 20152 +
1 = (20152 + 2015 + 1)(20152 − 2015 + 1). Akibatnya, n = 20152 > 2015 me-
menuhi syarat, dan bagian (a) terbukti.
Untuk bagian (b), akan kita buktikan dengan induksi matematika bahwa
k
n = 20152 untuk setiap bilangan asli k memenuhi syarat. Untuk k = 1,
pernyataan benar. Misal pernyataan juga benar untuk k = w, dengan w
adalah suatu bilangan asli. Menggunakan identitas n4 + n2 + 1 = (n2 + n +
w w
1)(n2 − n + 1), bisa kita deduksi bahwa 20152 + 2015 + 1|(20152 )2 + 20152 +
w+1 w+1
1|(20152 )2 + 20152 + 1. Akibatnya, persamaan juga benar untuk k =
k
w + 1. Menurut induksi matematika, n = 20152 memenuhi syarat untuk
setiap bilangan asli k. Karena ada tak berhingga banyaknya bilangan
asli, bisa disimpulkan bahwa ada tak berhingga banyaknya bilangan asli
n yang memenuhi syarat pada soal (a).
Komentar:

a) Ide pokok dari penyelesaian soal ini adalah pemfaktoran n4 + n2 +1 =


(n2 + n + 1)(n2 − n + 1) dan pengamatan bahwa jika n = k memenuhi
syarat pada soal, n = k2 juga memenuhi syarat pada soal, sehingga
kita bisa mengeneralisasi nilai n yang memenuhi syarat menjadi
20152 , 20154 , 20158 , . . . .
b) Beberapa kemungkinan penyelesaian yang menghasilkan nilai
yang tidak sempurna adalah:
i. Penggunaan pola dalam membuat deduksi; kesalahan ini ba-
nyak dilakukan oleh para kontestan. Hal ini tidak diperbolehk-

141
Buku KTO Matematika 1

an dalam olimpiade matematika karena kesimpulan yang ber-


laku untuk sekumpulan nilai tidak tentu berlaku untuk semua
nilai. Sebagai contoh, jika sebuah fungsi f : N → N memenuhi
f (k) = k untuk setiap bilangan asli k yang kurang dari 2016,
tidak bisa disimpulkan bahwa f (n) = n untuk setiap bilangan
asli n; bisa saja fungsi f tersebut adalah f (k) = k untuk setiap
bilangan asli k < 2016 dan f (k) = 1 untuk k yang lain.
ii. Penggunaan fakta tanpa bukti; sebagai contoh, ada solusi yang
menyatakan bahwa x2 + x + 1 selalu tidak habis membagi x3k+1 −
1 untuk setiap bilangan asli k. Kemudian, ada juga solusi yang
mengandung klaim yang salah, contohnya bahwa n2 + n + 1 ada-
lah bilangan prima untuk setiap bilangan asli n.
iii. Mengasumsikan bahwa setiap n yang memenuhi haruslah me-
rupakan bilangan kuadrat sempurna; pembuatan asumsi ha-
ruslah didasarkan oleh fakta atau bukti. Meskipun asumsi ter-
sebut tidak Anda gunakan dalam solusi, penulisan asumsi yang
salah dapat membuat poin Anda dikurangi.

4. Jawab: f (x) = b xc untuk setiap x atau f (x) = −b xc untuk setiap x.

Substitusi x → f (x) ke persamaan awal, didapat f ( f (x) + f (y)) = f ( f (x)) +


b yc. Selanjutnya, tukar variabel x dan y untuk memperoleh f ( f (x) +
f (y)) = f ( f (x)) + b yc = f ( f (y)) + b xc. Akibatnya, f ( f (x)) − b xc = f ( f (y)) − b yc
untuk setiap dua bilangan real x dan y, sehingga kita dapatkan f ( f (x)) =
c + b xc untuk setiap bilangan real x dan suatu konstanta real c.

Selanjutnya, dengan mengaplikasikan fungsi f pada kedua ruas persa-


maan soal dan menggunakan persamaan pada soal, kita dapatkan c +
b x + f (y)c = f ( f (x + f (y))) = f (b yc + f (x)) = f (b yc) + b xc, atau c + b x + f (y)c =
f (b yc) + b xc, sebut sebagai persamaan 1. Jika x = 0, kita dapatkan
c + b f (y)c = f (b yc) untuk setiap bilangan real y, atau c = f (b yc) − b f (y)c.
Substitusi hasil tersebut ke persamaan 1, kita dapatkan b x + f (y)c =
b xc + b f (b yc)c, sebut sebagai persamaan 2.

Substitusi y → f (y) ke persamaan soal, menggunakan identitas yang su-


dah kita peroleh, kita dapatkan f (x + c + b yc) = f (x) + b f (y)c, sebut seba-
gai persamaan 3. Masukkan y → b yc ke persamaan tersebut, didapat
identitas b f (y)c = b f (b yc)c, sehingga persamaan 2 bisa kita tulis sebagai
b x + f (y)c = b xc + b f (y)c, sebut sebagai persamaan 4.

142
Pembahasan Kontes November 2015

Sekarang, akan kita tunjukkan bahwa f (n) haruslah bernilai bulat un-
tuk setiap bilangan real n. Misalkan ada y sehingga f (y) bukan meru-
pakan bilangan bulat. Tulis f (y) = p + q, di mana p adalah bagian bulat
dan q adalah bagian pecahan dari f (y). Substitusi x → 1 − q ke persama-
an 4, diperoleh 1 + p = p, sebuah kontradiksi. Akibatnya, f (n) haruslah
bernilai bulat.
Perhatikan bahwa c haruslah merupakan bilangan bulat (tinjau f ( f (x)) =
c + b xc) dan f (y) = f (b yc) (tinjau b f (y)c = b f (b yc)c) untuk setiap bilangan
real y. Kemudian, persamaan 3 sekarang bisa ditulis sebagai f (x + y+ c) =
f (x) + f (y) untuk setiap bilangan dua bilangan bulat x dan y. Lihat pu-
la bahwa persamaan pada soal bisa ditulis sebagai f (x + f (y)) = f (x) + y
untuk setiap dua bilangan bulat x dan y. Aplikasikan fungsi pada kedua
ruas, diperoleh c + x + f (y) = f (y) + x, atau c = 0 (pada langkah ini, kita
menggunakan fakta bahwa f ( f (x)) = x + c untuk setiap bilangan bulat x).
Akibatnya, kita punya f (x + y) = f (x) + f (y) untuk setiap bilangan bulat
x dan y, sehingga f (x) = px untuk suatu bilangan bulat p (silakan verifi-
kasi sendiri). Namun, karena f ( f (x)) = x, p yang memenuhi hanyalah 1
atau −1.
Jika p = 1, untuk setiap bilangan real x kita punya f (x) = f (b xc) = b xc.
Dengan cara yang sama, jika p = −1, kita punya f (x) = −b xc. Dapat dipe-
riksa bahwa kedua fungsi memenuhi syarat pada soal. Kita selesai.
Komentar:

a) Salah satu observasi yang penting pada soal ini adalah bahwa ki-
ta dapat mereduksi domain fungsi pada soal menjadi bilangan bu-
lat berdasarkan identitas f (x) = f (b xc). Identitas tersebut membuat
kita dapat mengeneralisasi nilai fungsi f (x) untuk setiap bilangan
real x jika kita mengetahui nilai f (x) untuk nilai x yang bulat.
b) Beberapa kemungkinan penyelesaian yang menghasilkan nilai
yang tidak sempurna adalah:
i. Memisalkan bahwa fungsi f memiliki sifat tertentu tanpa fak-
ta pendukung; sebagai contoh, ada solusi yang menggunakan
pemisalan fungsi f adalah linear, sehingga f (x) = px + q untuk
dua bilangan real p dan q.
ii. Mengasumsikan bahwa terdapat bilangan real a sehingga
f (a) = b, untuk suatu bilangan real b; hal ini tidak boleh di-
lakukan, sebab tidak semua fungsi adalah surjektif (ada titik

143
Buku KTO Matematika 1

pada domain yang dipetakan ke setiap titik pada kodomain).


Sebagai contoh, pada fungsi f (x) = x2 + 1, tidak ada titik pada
domain yang dipetakan ke 0.
iii. Menganggap bahwa fungsi periodik selalu memiliki nilai eks-
trem (minimum atau maksimum); pernyataan ini bisa dibantah
dengan mengambil contoh f (x) = tan x.
iv. Tidak memeriksa bahwa persamaan fungsi yang didapat me-
menuhi syarat soal; hal ini sangat fatal dan dapat berakibat
pada pengurangan poin. Walaupun terlihat jelas dan tidak ber-
guna, memasukkan fungsi yang didapat ke persamaan awal sa-
ngatlah esensial karena kasus di mana fungsi yang didapat ter-
nyata tidak memenuhi syarat soal mungkin terjadi.

144
7 Kontes Desember 2015

7.1 Soal-Soal
Bagian A

Tuliskan jawaban akhir setiap soal di lembar jawaban. Setiap soal bernilai 1
angka. Jawaban Anda harus berupa bilangan bulat dari 0 sampai 999. So-
al yang diberi (*) memiliki jawaban tidak bulat atau tidak di dalam
batas [0, 999].

1. (1007) Suatu bilangan asli dua digit bernilai 29 kali lebih besar jika dibaca
dari kanan ke kiri. Tentukan bilangan asli dua digit ini.

2. (1014) Diberikan persegi ABCD dengan panjang sisi 100. Titik E terle-
tak pada sisi BC sehingga luas segitiga ABE adalah adalah 41 dari luas
trapesium ADCE. Tentukan panjang BE.

3. (1021) Tentukan banyaknya pasangan terurut bilangan asli (a, b, c) se-


hingga
abc + ab + ac = 24.

4. (1028) Sebuah bilangan palindrom adalah sebuah bilangan yang sama ji-
ka dibaca dari kiri maupun dari kanan. Misalnya, 202 adalah palindrom.
Misalkan N adalah sebuah bilangan asli tiga digit palindrom sehingga
N + 32 merupakan bilangan empat digit palindrom. Tuliskan nilai N ini.

5. (1035) Pada sebuah papan catur berukuran 5 × 5 akan diletakkan 3 buah


benteng secara acak pada 3 kotak berbeda. Peluang bahwa setiap ben-
teng berada di kolom dan baris yang berbeda-beda dapat ditulis dalam

145
Buku KTO Matematika 1

bentuk m
n di mana m dan n adalah bilangan asli yang berbeda-beda. Hi-
tung m + n.

6. (1042) Diberikan sebuah segiempat konveks ABCD dengan AB = 3,


BC = 4, CD = 13, AD = 12, dan ∠ ABC = 90°. Berapakah luas segiem-
pat ABCD ini?

7. (1049) Tentukan banyaknya bilangan bulat n sehingga n3 + 8 memiliki


tidak lebih dari 3 buah pembagi positif.

8. [Soal ini tidak memiliki jawaban] Misalkan f adalah fungsi dengan


f (2x − 1) = x4 + x3 + x2 + x + 1 untuk setiap bilangan real x. Diketahui
bahwa berlaku f (x − 2) = ax2 + bx + c untuk suatu bilangan real a, b, c.
Tentukan nilai a + b + c.

9. (1063) Titik E terletak di luar persegi ABCD. Jika jarak dari E ke AC


adalah 6, ke BD adalah 17, dan ke titik sudut ABCD terdekat adalah 10,
berapakah luas persegi ABCD?

10. [Soal ini tidak memiliki jawaban] Misalkan P(x) merupakan polino-
mial kubik sehingga

P(0) = 10, P(1) = 20 dan P(2) = 30.

Hitung P(2) + P(−2).

11. (*) (1077) Misalkan S = {1, 2, 3, 4, 5}. Tentukanlah


X X
| X ∩ Y |,
X ⊆S Y ⊆S

yaitu hasil jumlah semua kardinalitas dari irisan setiap pasang subhim-
punan dari S.

12. (1084) Pada segitiga ABC, diketahui AB = 86 dan AC = 97. Sebuah ling-
karan dengan pusat A dan radius AB memotong segmen garis BC di
titik B dan X . Lebih jauh, segmen garis BX dan C X memiliki panjang
bilangan bulat. Tentukan panjang BC.

13. (1091) Cheryl akan menaiki 12 buah anak tangga. Setiap kali melang-
kah, Cheryl akan menaiki satu, dua, atau tiga anak tangga. Namun,
Cheryl tidak boleh menaiki satu anak tangga dua kali berturut-turut.
Tentukan banyak cara Cheryl dapat menaiki 12 buah anak tangga ini.

146
Soal-Soal Kontes Desember 2015

14. (1098) Diketahui bahwa


1
= 0.b 0 b 1 . . . b n−1
992
di mana b 0 , b 1 , b 2 , . . . , b n−1 merupakan digit-digit 0 hingga 9 dan n meru-
pakan panjang terkecil untuk ekspansi desimal yang berulang. Hitung
b 0 + b 1 + · · · + b n−1 .

147
Buku KTO Matematika 1

Bagian B

Tuliskan jawaban beserta langkah pekerjaan Anda pada lembar jawaban.


Selain jawaban akhir, Anda perlu menuliskan argumentasi atau langkah-
langkah untuk memperoleh jawaban akhir tersebut. Gunakan halaman yang
berbeda untuk setiap soal yang berbeda. Setiap soal bernilai 7 angka.

1. (1119) Tentukan banyaknya bilangan asli n ∈ {1, 2, 3, . . . , 2015} sehingga


n · 2n merupakan bilangan kuadrat sempurna.

2. (1126) Tentukan nilai terkecil yang mungkin untuk

(a − 2b + 1)2 + (b − 2c + 1)2 + (c − 2a + 1)2 + (a + b + c − 1)2

di mana a, b, c merupakan bilangan real.

3. (1133) Diberikan sebuah bilangan asli n. Pada suatu papan catur beru-
kuran 2 × n akan diletakkan k buah gajah di k kotak berbeda demikian
sehingga tidak ada dua gajah yang saling menyerang. Dua gajah dika-
takan saling menyerang jika mereka terletak pada dua kotak yang me-
miliki tepat satu titik sudut bersama.

4. (1087) Diberikan segitiga ABC dan titik P di dalam segitiga tersebut.


Titik A 0 , B0 , C 0 adalah hasil pencerminan P terhadap sisi-sisi BC, C A,
AB. Tunjukkan bahwa lingkaran-lingkaran luar segitiga AB0 C 0 , BC 0 A 0 ,
dan C A 0 B0 berpotongan di sebuah titik bersama.

148
Pembahasan Kontes Desember 2015

7.2 Pembahasan
Bagian A

1. Jawab: 18.
Misalkan bilangan tersebut adalah AB dengan A 6= 0. Kita peroleh

9AB = 2BA
⇐⇒ 9(10A + B) = 2(10B + A)
⇐⇒ 90A + 9B = 20B + 2A
⇐⇒ 88A = 11B
⇐⇒ 8A = B.

Karena B ∈ {0, 1, 2, . . . , 9} dan B kelipatan 8, kita peroleh B = 8 atau B =


0. Perhatikan bahwa B = 0 berakibat A = 0, kontradiksi dengan syarat
A 6= 0. Dengan demikian, B = 8, sehingga diperoleh A = 1. Jadi, bilangan
yang dimaksud adalah 18.
Komentar:

a) Soal ini dapat diselesaikan oleh 73% dari keseluruhan peserta. Hal
tersebut berarti bahwa para peserta pada umumnya tidak merasa
kesulitan dalam mengerjakan soal ini.
b) Cukup dibutuhkan pengetahuan dasar tentang penjabaran bilang-
an desimal untuk dapat mengerjakan soal ini.

2. Jawab: 40.

D C

A 100 B

149
Buku KTO Matematika 1

Perhatikan bahwa
1
[ABE] = BE × 100
2
dan
1
[ACDE] = (100 + (100 − BE)) × 100,
2
dengan [ABE] dan [ACDE] berturut-turut menyatakan luas segitiga
ABE dan luas trapesium ACDE. Berdasarkan fakta bahwa [ADCE] =
4 × [ABE], dapat diperoleh BE = 40.
Komentar:

a) Soal ini dapat diselesaikan oleh 85% peserta. Ini jelas merupakan
soal yang sangat mudah.
b) Pengerjaan soal ini hanya memerlukan pemahaman tentang luas
bidang datar.

3. Jawab: 3.
Kita coba selesaikan dengan membagi beberapa kasus berdasarkan ke-
mungkinan nilai b dan c. Pertama, kasus b, c ≥ 2. Dari persamaan awal,
kita peroleh
24 = abc + ab + ac ≥ 8a ⇐⇒ a ≤ 3.
Ada tiga kemungkinan untuk nilai a, yaitu 1, 2 atau 3. Jika a = 3, dengan
mensubstitusi ke persamaan awal diperoleh bc + b + c = 8 ⇐⇒ (b + 1)(c +
1) = 9. Artinya, b + 1 dan c + 1 faktor dari 9. Karena b, c ≥ 2, didapat
b + 1, c + 1 ≥ 3. Akibatnya, b + 1 = 3 dan c + 1 = 3, sehingga diperoleh solusi
pada kasus ini adalah (a, b, c) = (3, 2, 2). Dengan cara yang sama, dapat
dicek bahwa untuk a = 2 tidak ada solusi dan untuk a = 1 diperoleh solusi
(a, b, c) = (1, 4, 4).
Selanjutnya, kita tinjau kasus b = 1 atau c = 1. Apabila b = 1, persamaan
awal menjadi 24 = 2ac + a. Diperoleh a dan 2c + 1 faktor dari 24. Lebih
lanjut, kita peroleh a = 8 dan 2c + 1 = 3. Dengan demikian, solusi pada
kasus ini adalah (a, b, c) = (8, 1, 1). Solusi yang sama juga diperoleh untuk
kasus c = 1.
Jadi, banyaknya solusi ada 3, yaitu (3, 2, 2), (1, 4, 4), (8, 1, 1).
Komentar:

a) Soal ini dapat diselesaikan oleh 60% dari total peserta.

150
Pembahasan Kontes Desember 2015

b) Teknik penyelesaian yang digunakan dalam soal cukup standar,


yakni keterbagian dan teknik pembatasan solusi.
c) Perhatikan bahwa a merupakan faktor dari 24, sehingga penyelesa-
ian dari soal ini dapat diperoleh dengan mengecek setiap kemung-
kinan a, yaitu 1, 2, 3, 4, 6, 8, 12, 24. Namun, hal ini cukup mengha-
biskan banyak waktu, sehingga disarankan untuk melakukan pem-
batasan pada b dan c untuk meminimalkan jumlah kasus yang per-
lu dicek.

4. Jawab: 969.
Misalkan N = ABA dan N + 32 = CDDC. Karena 0 < 32 < 100, haruslah
900 < N < 1000. Dengan demikian, diperoleh A = 9 dan C = 1. Selanjut-
nya, perhatikan bahwa

9B9 + 32 = 1DD1
⇐⇒ 900 + 10B + 9 + 32 = 1000 + 100D + 10D + 1
⇐⇒ 10B = 60 + 110D
⇐⇒ B = 6 + 11D.

Oleh karena 0 ≤ B, D ≤ 9, diperoleh B = 6 dan D = 0. Jadi, N = 969.


Komentar:

a) Soal yang dapat diselesaikan oleh 98% peserta ini merupakan soal
yang sangat mudah.
b) Pengerjaan soal ini hanya memerlukan pemahaman tentang penja-
baran bilangan desimal seperti pada soal nomor 1.

5. Jawab: 29.
Perhatikan bahwa banyaknya cara meletakkan 3 buah benteng secara
acak pada 3 kotak berbeda dari 25 kotak yang tersedia adalah 25
¡ ¢
3 = 2300.

Cara meletakkan 3 benteng tersebut agar masing-masing berada pada


baris dan kolom yang berbeda dapat dilakukan dengan memilih 3 ko-
lom berbeda dari 5 kolom tersedia, kemudian dilanjutkan dengan mele-
takkan masing-masing benteng dalam kolom yang berbeda (dari 3 kolom
yang telah dipilih) dengan syarat masing-masing berada pada baris yang

151
Buku KTO Matematika 1

berbeda. Banyaknya cara ada 53 × (5 × 4 × 3) = 600. Dengan demikian,


¡ ¢

peluangnya adalah
m 600 6
= = .
n 2300 23
Jadi, m + n = 29.
Komentar:

a) Perlu ditambahkan keterangan pada soal bahwa FPB(m, n) = 1. Ji-


ka tidak, kemungkinan untuk nilai m + n ada tak berhingga banyak.
b) Tingkat kesulitan soal ini dapat dikategorikan sedang karena soal
ini hanya dapat diselesaikan oleh 43% peserta.
c) Pemahaman yang cukup mengenai prinsip kombinasi dan permuta-
si menjadi kunci dalam pengerjaan soal ini.

6. Jawab: 36.

A B

Perhatikan bahwa AC = 5. Oleh karena (5, 12, 13) merupakan tri-


pel Phytagoras, diperoleh ADC merupakan segitiga siku-siku dengan
∠D AC = 90°. Dengan demikian, luas segiempat ABCD sama dengan
luas segitiga ABC dijumlah dengan luas segitiga ACD, yaitu
1 1
(3 × 4) + (5 × 12) = 6 + 30 = 36.
2 2

152
Pembahasan Kontes Desember 2015

Komentar:

a) Soal ini dapat diselesaikan oleh 92% peserta. Akibatnya, bisa di-
simpulkan soal ini sangat mudah.
b) Pengetahuan mengenai tripel Phytagoras diperlukan dalam menye-
lesaikan soal ini. Setiap tripel phytagoras dapat dinyatakan dalam
bentuk (t(a2 + b2 ), 2tab, t(a2 − b2 )) atau permutasinya dengan t, a
dan b bilangan asli.

7. Jawab: 3.
Perhatikan bahwa n3 + 8 = (n + 2)(n2 − 2n + 4). Agar n3 + 8 memiliki paling
banyak 3 pembagi positif, haruslah salah satu kemungkinan berikut ini
terjadi: | n + 2| = 1 atau | n2 − 2n + 4| = 1 atau | n + 2| = | n2 − 2n + 4| = bi-
langan prima. Apabila | n + 2| = 1, diperoleh n yang memenuhi adalah −1
dan −3. Apabila | n2 − 2n + 4| = 1, diperoleh tidak ada n yang memenuhi.
Apabila | n + 2| = | n2 − 2n + 4| = bilangan prima, diperoleh nilai n yang me-
menuhi adalah 1. Dengan demikian, banyaknya bilangan bulat n yang
memenuhi adalah 3.
Komentar:

a) Soal ini dikategorikan soal sedang-sulit.


b) Dalam menyelesaikan soal ini, diperlukan pengetahuan tentang
teknik pemfaktoran dan observasi mengenai fakta bahwa bilangan
asli yang memiliki paling banyak 3 pembagi positif hanyalah bilang-
an prima atau kuadrat dari bilangan prima.

8. Jawab: Tidak ada fungsi f yang memenuhi.


Perhatikan bahwa tidak ada fungsi tak nol f yang merupakan suku ba-
nyak derajat empat sekaligus suku banyak derajat 2.
Komentar:

a) Secara umum, apabila fungsi yang memenuhi kondisi yang dibe-


rikan ada, penyelesaiannya dapat diperoleh dengan menggunakan
teknik substitusi. Dengan mengambil substitusi x = 1 didapat
a + b + c = f (−1). Dari persamaan pertama diperoleh f (−1) = 1 (sub-
stitusi x = 0).

153
Buku KTO Matematika 1

9. Jawab: 162.

A D
Q

O
E
B C

Tanpa mengurangi keumuman, misalkan diagonal dari persegi ABCD


yang terdekat dengan E adalah AC. Misalkan O pusat persegi ABCD
dan P dan Q berturut-turut adalah proyeksi titik E pada AC dan BD.
Kita peroleh PE = 6 dan QE = 17. Karena PE < QE, diperoleh titik su-
dut ABCD terdekat dengan E adalah C. Dengan demikian, CE = 10.
Sekarang, perhatikan bahwa apabila P berada di dalam persegi ABCD,
kita peroleh CP = 8, sehingga didapat AP = 17 + 8 = 25. Akibatnya berla-
ku PE + QE = 23 < AP, yang berarti E berada di dalam persegi ABCD,
kontradiksi dengan fakta E berada di luar persegi ABCD. Dengan de-
mikian, P berada di luar persegi ABCD, sehingga diperoleh CP = 8 dan
OC = 17 − 8 = 9. Jadi, luas persegi ABCD adalah 2OC 2 = 162.
Komentar:

a) Soal ini dapat diselesaikan oleh 38% peserta, sehingga dapat dika-
tegorikan soal sedang-sulit.
b) Pembaca disarankan untuk menggambar ilustrasi soal karena akan
memudahkan menemukan ide untuk penyelesaian soal ini.
c) Apabila telah ditemukan beberapa kemungkinan solusi, diharapk-
an untuk mengecek kembali apakah solusi yang diperoleh memenu-
hi kondisi yang diberikan pada soal. Dalam soal ini, apabila telah
diperoleh solusi, perlu dicek kembali apakah titik E berada di luar
persegi atau tidak.

154
Pembahasan Kontes Desember 2015

10. Jawab: Ada tak berhingga banyak nilai yang mungkin dari P(2) +
P(−2).
Perhatikan bahwa P0 (x) = 10x + 10 memenuhi P(0) = 10, P(1) = 20 dan
P(2) = 30. Dibentuk suku banyak Q(x) dengan Q(x) = P(x) − P0 (x).
Diperoleh Q(x) merupakan suku banyak kubik (berderajat 3) dengan
Q(0) = Q(1) = Q(2) = 0. Artinya, 0, 1 dan 2 merupakan akar-akar dari
Q(x), sehingga diperoleh

Q(x) = Ax(x − 1)(x − 2)

untuk suatu bilangan real A. Dengan demikian, kita peroleh

P(x) = Ax(x − 1)(x − 2) + 10x + 10

dan didapat
P(2) + P(−2) = 20 − 24A.
Nilai P(2) + P(−2) bergantung A.
Komentar:

a) Soal ini dianulir sebab solusi masih bergantung pada koefisien de-
rajat tertinggi polinomial P(x). Perlu diberikan keterangan/syarat
tambahan agar solusi tersebut berupa konstanta yang berupa bi-
langan antara 1-999. Sebagai contoh, apabila diberi keterangan
bahwa koefisien derajat tertinggi P(x) adalah -1, diperoleh P(2) +
P(−2) = 44.
b) Ada beberapa teknik yang dapat digunakan untuk menyelesaikan
permasalahan seperti ini, yaitu:
i. Memisalkan P(x) = a n x n + a n−1 x n−1 + · · · + a 1 x + a 0 , kemudian
mensubstitusi kondisi yang diberikan sehingga diperoleh suatu
sistem persamaan linear dan dicari penyelesaiannya.
ii. Memanfaatkan bentuk polinomial Lagrange dengan berdasar
pada fakta bahwa “apabila diberikan n buah pasangan bilangan
real (x i , yi ), dapat ditemukan dengan tunggal polinomial P(x)
dengan derajat kurang dari n yang memenuhi P(x i ) = yi untuk
setiap i = 1, 2, . . . , n”. Referensi mengenai polinomial Lagrange
dapat diperoleh dari artikel Wikipedia bahasa Inggris yang ber-
judul Lagrange polynomial.

155
Buku KTO Matematika 1

11. Jawab: 1280.

Diambil sebarang X ⊆ S. Perhatikan bahwa

| X ∩ Y | + | X ∩ (S \Y )| = | X |

untuk setiap Y ⊆ S. Selanjutnya, perhatikan bahwa

| X | = 2|S | | X | = 32| X |.
X X X X
2 |X ∩ Y | = |X ∩ Y | + | X ∩ (S \Y )| =
Y ⊆S Y ⊆S Y ⊆S Y ⊆S

Dengan demikian, kita peroleh

X X X
| X ∩ Y | = 16 |X |
X ⊆S Y ⊆S X ⊆S
ÃÃ ! Ã ! Ã ! Ã ! Ã ! !
5 5 5 5 5
= 16 0 + ×1+ ×2+ ×3+ ×4+ ×5
1 2 3 4 5
= 16 × 80 = 1280.

Komentar:

a) Soal ini dianulir karena solusi merupakan bilangan empat digit, ti-
dak memenuhi persyaratan solusi pada lembar soal.

b) Soal ini dapat dikerjakan dengan menuliskan semua kemungkinan


pasangan (X , Y ), kemudian menghitung banyak anggota masing-
masing irisannya dan menjumlahkan semuanya. Akan tetapi hal
ini akan menghabiskan cukup banyak waktu. Salah satu cara yang
disarankan adalah dengan memanfaatkan fakta dalam teori him-
punan bahwa
| X ∩ Y | + | X ∩ (S \Y )| = | X |.

Untuk memudahkan pembaca dalam memahami fakta ini, pembaca


dapat mengilustrasikannya dalam diagram Venn.

12. Jawab: 61.

156
Pembahasan Kontes Desember 2015

x X

A B

Perhatikan bahwa A X = 86. Misalkan C X = a, BX = b dan ∠ A X C = x.


Diperoleh ∠BX A = 180° − x. Dengan menggunakan aturan cosinus pada
segitiga A X C dan A X B, diperoleh
972 = 862 + a2 − 172a cos x
dan
862 = 862 + b2 + 172b cos x.
Dengan menjumlahkan b kali persamaan pertama dengan a kali persa-
maan kedua, diperoleh a2 +ab = 2013 ⇐⇒ a(a+ b) = 3×11×61. Mengingat
a dan b bulat, dan a < a + b < 97 + 86 = 183, nilai a dan b yang memenuhi
hanya a = 33 dan b = 28. Jadi, BC = a + b = 61.
Komentar:

a) Sebanyak 28% peserta mampu menjawab soal ini, sehingga soal ini
dikategorikan soal cukup sulit.
b) Pemahaman mengenai aturan cosinus dan fakta bahwa a dan b me-
rupakan bilangan bulat sangat berperan dalam menyelesaikan soal
ini.

13. Jawab: 189.


Misalkan A(n) menyatakan banyaknya cara Cheryl menaiki n anak tang-
ga dengan setiap langkah, Cheryl menaiki satu, dua, atau tiga anak
tangga dengan syarat tidak boleh menaiki satu anak tangga dua kali
berturut-turut. Akan dihitung A(n) dengan membagi kasus pada ba-
nyaknya anak tangga yang ia naiki pada langkah pertama.

157
Buku KTO Matematika 1

a) Apabila pada langkah pertama Cheryl menaiki 3 anak tangga, ba-


nyaknya cara melangkah selanjutnya yang dapat ia lakukan agar
tetap memenuhi kondisi yang diberikan ada A(n − 3) cara.
b) Apabila pada langkah pertama Cheryl menaiki 2 anak tangga, ba-
nyaknya cara melangkah selanjutnya yang dapat ia lakukan agar
tetap memenuhi kondisi yang diberikan ada A(n − 2) cara.
c) Apabila pada langkah pertama Cheryl menaiki 1 anak tangga,
pada langkah kedua Cheryl harus menaiki 2 atau 3 anak tang-
ga. Diperoleh banyaknya cara melangkah selanjutnya yang da-
pat ia lakukan agar tetap memenuhi kondisi yang diberikan ada
A(n − 1 − 2) + A(n − 1 − 3) = A(n − 3) + A(n − 4) cara.

Dengan demikian, diperoleh hubungan

A(n) = A(n − 2) + 2A(n − 3) + A(n − 4).

Perhatikan bahwa A(1) = 1, A(2) = 1, A(3) = 3 dan A(4) = 4. Dengan


memanfaatkan persamaan di atas, diperoleh nilai dari A(1), . . . , A(12)
berturut-turut adalah 1, 1, 3, 4, 6, 11, 17, 27, 45, 72, 116, 189. Dengan
demikian diperoleh banyaknya cara Cheryl dapat menaiki 12 buah anak
tangga adalah 189 cara.
Komentar:

a) Sebanyak 18% peserta dapat menyelesaikan soal ini dengan benar.


Soal ini bisa digolongkan sebagai soal yang sulit.
b) Diperlukan pemahaman yang cukup baik tentang pembentukan re-
lasi rekurensi untuk menyelesaikan soal ini.
c) Bentuk eksplisit dari A(n) dapat dicari dengan memanfaatkan tek-
nik rekurensi atau teknik fungsi pembangkit. Namun, dalam kasus
ini, A(n) sedikit rumit untuk dituliskan ke dalam bentuk eksplisit-
nya. Di sisi lain, karena nilai n yang ditanyakan adalah 12, akan
lebih mudah dengan langsung mencari nilai A(n), n ∈ {5, . . . , 12} se-
cara rekursif.

14. Jawab: 75.


Pertama-tama, akan dicari nilai n. Dengan mengalikan persamaan pa-
da soal dengan 10n , kemudian dikurangi dengan persamaan awal, kita

158
Pembahasan Kontes Desember 2015

peroleh
10n − 1
= b 0 b 1 · · · b n−1 .
992
Oleh karena b 0 b 1 · · · b n−1 bulat, didapat 992 |10n − 1. Lebih lanjut, n me-
rupakan bilangan asli terkecil sehingga 992 |10n − 1.
Perhatikan bahwa dari fakta 11|99 dan 992 |10n − 1, didapat 11|10n − 1
atau 10n ≡ 1 (mod 1)1. Karena 10 ≡ −1 (mod 11), haruslah n merupakan
bilangan genap. Misalkan n = 2k untuk suatu bilangan asli k. Perhati-
kan bahwa

10n − 1 = 100k − 1 = (99 + 1)k − 1


à !
k k
99 i + 99k + 1 − 1
X
=
i =2 i
à !
k k
2
99 i−2 + 99k.
X
= 99
i =2 i

Karena 992 |10n − 1, diperoleh 99| k. Dengan demikian, k = 99 dan n = 198.


Selanjutnya, akan dicari nilai b 0 + b 1 + · · · + b 198 . Perhatikan bahwa
10198 − 1 1 ¡
= 2 10099 − 1
¢
99 2 99
1
= 2 (100 − 1) 10098 + 10097 + · · · + 100 + 1
¡ ¢
99
1 ¡
10098 + 10097 + · · · + 100 + 1
¢
=
99
1 ¡
(10098 − 1) + (10097 − 1) + · · · + (100 − 1) + (1 − 1) + 99
¢
=
99
= 10097 + 10096 + · · · + 1
¡ ¢

+ 10096 + 10095 + · · · + 1
¡ ¢

+ · · · + (100 + 1) + 1 + 1
= 10203040506070809101112 . . . 969799

Perhatikan bahwa pada bilangan 10203040506070809101112 . . . 969799,


setiap digit i ∈ {1, 2, . . . , 7} muncul tepat sebanyak 20 kali dan digit 8 dan
9 masing-masing muncul tepat sebanyak 19 kali. Dengan demikian, kita
peroleh

b 0 + b 1 + · · · + b 198 = 20 × (1 + 2 + · · · + 7) + 19 × (8 + 9) = 883.

159
Buku KTO Matematika 1

Komentar:

a) Berdasarkan fakta bahwa hanya 16 % dari keseluruhan peserta


mampu menyelesaikan soal ini, soal ini dikategorikan sulit.
b) Ide awal dalam menyelesaikan soal ini serupa dengan ide yang digu-
nakan dalam mengubah bentuk desimal bilangan rasional ke ben-
tuk pecahan.
c) Pengetahuan akan teknik pemfaktoran, keterbagian dan Binomial
Newton adalah penting dalam menyelesaikan soal ini. Selain itu
juga diperlukan kecermatan dan ketelitian dalam menghitung nilai
b 0 + b 1 + · · · + b 198 .

160
Pembahasan Kontes Desember 2015

Bagian B

1. Jawab: 22.
Misalkan n = 2k (2t + 1) di mana k, t ≥ 0 merupakan bilangan bulat. De-
ngan demikian, haruslah 2k+n (2t + 1) merupakan bilangan kuadrat sem-
purna. Jadi, haruslah k + n bilangan genap. Jika k = 0, berarti n ganjil,
sehingga k + n ganjil. Jadi, haruslah k > 0. Karena n genap, berarti k
juga merupakan bilangan genap. Sekarang, haruslah 2t + 1 merupakan
bilangan kuadrat sempurna. Jadi, kita peroleh n harus merupakan bi-
langan kuadrat genap. Semua bilangan ini adalah 22 , 42 , 62 , . . . , 442 , jadi
ada sebanyak 22 bilangan.
Komentar:

a) Ide utama dalam menyelesaikan soal ini adalah menemukan fakta


bahwa n harus genap dan merupakan bilangan kuadrat, sehingga
kita cukup menghitung banyaknya bilangan kuadrat genap antara
1 sampai 2015.
b) Banyak peserta tidak membuktikan mengapa n harus genap atau
n dan 2n bilangan kuadrat. Hal ini tentu akan menghasilkan nilai
yang tidak sempurna. Bukti dari fakta tersebut dapat diperoleh
dengan mengobservasi faktor 2 dari n kali 2n untuk n ganjil dan
membandingkannnya dengan sifat bilangan kuadrat.

2. Jawab: 1.
Solusi 1.
Nilai minimumnya tercapai saat a = b = c = 21 .
Perhatikan pula bahwa

(a − 2b + 1)2 + (a + b + c − 1)2 = 6 a2 − 2
X X X X
ab − 4 a+4
c yc s ym c yc s ym

(a − b)2 + (2a − 1)2 + 1


X X
=
c yc c yc

≥ 1.

Solusi 2.
Kita buktikan ketaksamaan bernilai lebih besar atau sama dengan 1 de-
ngan cara lain. Perhatikan bahwa p2 + q2 + r 2 + s2 ≥ 41 (p + q + r + s)2 untuk

161
Buku KTO Matematika 1

setiap bilangan real p, q, r, s. Dengan demikian, berlaku ketaksamaan


pada soal ≥ 1.
Komentar:

a) Ide pokok dari penyelesaian soal ini adalah menggunakan teknik


melengkapkan kuadrat. Penyelesaian juga dapat diperoleh dengan
menggunakan ketaksamaan QM-AM.
b) Beberapa kemungkinan penyelesaian yang menghasilkan nilai
yang tidak sempurna adalah:
i. Peserta tidak memberikan konfirmasi apakah nilai minimum
yang diperoleh tercapai.
ii. Peserta langsung mengasumsikan/mengklaim nilai minimum
tercapai saat a = b = c.

3. Bukti.
Pandang papan catur sebagai sebuah graf G dengan kotak-kotak papan
catur menjadi verteks dan dua verteks dihubungkan dengan sebuah si-
si jika dan hanya jika kedua kotak yang diwakilkan oleh kedua verteks
tersebut memiliki tepat satu titik bersama di papan catur. Perhatikan
bahwa diperoleh dua buah subgraf terpisah, masing-masing berupa graf
path P n dengan n verteks. Pandang meletakkan gajah pada sebuah ko-
tak di papan catur sebagai pewarnaan sebuah verteks. Dengan demikian,
kita ingin mencari banyak maksimum verteks yang bisa diwarnai agar
tidak ada dua verteks berwarna yang bertetangga. Perhatikan bahwa
banyak verteks maksimal yang bisa diwarnai dari P n adalah d n2 e. Jadi,
karena ada dua buah subgraf P n , banyak maksimum verteks yang bisa
diwarnai adalah 2d n2 e.
Konfigurasi untuk n genap adalah memilih semua n kotak di baris per-
tama. Konfigurasi untuk n ganjil adalah memilih kedua kotak di setiap
kolom ganjil.
Komentar:

a) Ide pokok dari penyelesaian soal ini adalah fakta bahwa gajah yang
berada pada kotak berbeda warna tidak akan saling serang. De-
ngan demikian, cukup dicari maksimal banyaknya gajah yang bisa
diletakkan pada kotak berwarna sama sehingga tidak saling serang.

162
Pembahasan Kontes Desember 2015

b) Beberapa kemungkinan penyelesaian yang menghasilkan nilai


yang tidak sempurna adalah:
i. Peserta tidak menunjukkan mengapa gajah-gajahnya harus di-
letakkan sede-mikian rupa sehingga banyak gajah yang bisa
ditaruh maksimal. Hal ini sebenarnya dapat ditunjukkan de-
ngan mencoba partisi papan menjadi 2 × 2 atau membagi kasus
berdasarkan peletakkan gajah-gajahnya, dimulai dari kiri.
ii. Peserta tidak memberikan contoh konfigurasi secara umum un-
tuk setiap n.

4. Bukti.

A
C0
B0
P

B
C

A0

Misalkan C 0 dan D adalah titik perpotongan lingkaran luar segitiga


AB0 C 0 dan BC 0 A 0 . Cukup ditunjukkan bahwa titik D terletak pada ling-
karan luar segitiga C A 0 B0 . Perhatikan bahwa A 0 adalah refleksi titik P

163
Buku KTO Matematika 1

terhadap sisi BC. Kita peroleh ∠PCB = ∠BC A 0 dan ∠PBC = ∠CBA 0 .
Dengan cara yang sama, diperoleh ∠PC A = ∠ ACB0 , ∠P AC = ∠C AB0 ,
∠P AB = ∠BAC 0 dan ∠PBA = ∠ ABC 0 . Perhatikan bahwa

∠PCB + ∠PBC + ∠PC A + ∠P AC + ∠P AB + ∠PBA = 180°.

Karena D terletak pada lingkaran luar segitiga AB0 C 0 , diperoleh AB0 DC 0


segiempat talibusur, sehingga berlaku

∠B0 DC 0 = 180° − ∠B0 AC 0 = 180° − 2∠P AB − 2∠P AC.

Dengan cara yang sama, diperoleh ∠ A 0 DC 0 = 180° − 2∠PBA − 2∠PBC.


Dengan demikian,

∠B0 D A 0 = 360° − 2(∠P AB + ∠P AC + ∠PBA + ∠PBC)


= 360° − 2(180° − ∠PC A − ∠PCB)
= ∠B 0 C A 0 .

Artinya, A 0 B0 DC merupakan segiempat talibusur atau dengan kata lain


titik D terletak pada lingkaran luar segitiga C A 0 B0 . Terbukti.
Komentar:

a) Salah satu teknik yang dapat digunakan dalam menyelesaikan soal


tipe ini adalah dengan mengobservasi titik perpotongan dua ling-
karan, kemudian menunjukkan titik perpotongan tersebut berada
pada lingkaran ketiga. Fakta mengenai sudut-sudut dengan besar
sama dan sifat segiempat talibusur sangat membantu dalam me-
nyelesaikan soal ini.
b) Beberapa kemungkinan penyelesaian yang menghasilkan nilai
yang tidak sempurna adalah:
i. Memisalkan ketiga lingkaran tersebut berpotongan. Hal ini ti-
dak boleh dilakukan, sebab hal itulah yang harus dibuktikan.
ii. Kesalahan menuliskan notasi titik-titik dalam soal dan kesala-
han dalam merumuskan yang diketahui pada soal. Walaupun
terlihat tidak penting, namun hal ini memiliki pengaruh besar
dalam penyelesaian soal, terutama soal geometri.
iii. Membuktikan hanya dengan gambar. Hal ini tidak boleh dila-
kukan, sebab gambar bukan merupakan bukti, melainkan alat

164
Pembahasan Kontes Desember 2015

untuk mendukung pembuktian. Tentunya dalam membuktik-


an suatu pernyataan, Anda harus memberikan alasan-alasan
yang runtut dan sesuai dengan teori atau postulat yang sudah
ada.

165
8 Petunjuk

8.1 Petunjuk Level 1000


1001. Gunakan prinsip perkalian.

1002. Gunakan teorema Pythagoras pada segitiga ABC.

1003. Pandang paritas ketiga bilangan tersebut.

1004. Telitilah beberapa kasus pemotongan horizontal dan vertikal, dan berapa
potongan yang dihasilkan.

1005. Tinjau bagian pecahan dari x.

1006. Misalkan a dan x adalah suku awal dan beda dari barisan tersebut, lalu
nyatakan jumlah kelima anggota barisan dalam a dan x.

1007. Misalkan bilangan tersebut AB, lalu cari hubungan A dan B dengan
menjabarkan kondisi yang diberikan.

1008. Jika ekspresi pada soal dikurangi dengan sebuah bilangan bulat, perha-
tikan bahwa hasilnya juga akan berupa bilangan bulat.

1009. Perhatikan bahwa bilangan yang jumlah digit-digitnya habis dibagi 3


adalah bilangan yang habis dibagi 3.

1010. Temukan cara untuk ’mempercantik’ persamaan tersebut.

1011. Faktorkanlah pembilang dari pecahan tersebut dalam n.

1012. Nyatakan m dalam n.

167
Buku KTO Matematika 1

1013. Misalkan banyaknya anggota grup pada awalnya adalah x.

1014. Periksa luas segitiga ABE dan segiempat ACDE.

1015. Secara intuitif, bagaimana konfigurasi koin-koin yang meminimalkan


jumlah koin pada tumpukan paling besar?

1016. Gunakan fakta bahwa I adalah pertemuan garis-garis bagi segitiga ABC.

1017. (2001) Dengan substitusi, coba sederhanakan persamaan menjadi dua


variabel dengan memperhatikan syarat masing-masing variabel.

1018. Tujuan turnamen adalah menyisakan 1 pemain. Hal ini sama saja de-
ngan mengeluarkan 73 pemain lainnya.

1019. Gunakan prinsip inklusi-eksklusi.

1020. Misalkan L dan M adalah titik-titik potong garis K A dan K B dengan


garis CD.

1021. (2002) Bagi kasus berdasarkan kemungkinan nilai b dan c.

1022. Misalkan a adalah nilai tengah dari ke-2015 bilangan tersebut.

1023. Lakukan penjabaran pada operasi soal, lalu gunakan teorema Vieta.

1024. Selesaikan persamaan atau gunakan AM-GM.

1025. Dapatkan koordinat titik B dan D dengan merotasi titik A dan C dengan
pusat P dan sudut 90◦ .

1026. (2003) Gunakan teorema Pythagoras untuk mencari panjang AC.

1027. Tinjau sisa pembagian n2 dengan 7 berdasarkan sisa pembagian n de-


ngan 7.

1028. (2004) Selidiki kemungkinan digit pertama bilangan N + 32 dan digit per-
tama bilangan N.

1029. Gambar dengan rapi.

1030. Gunakan aturan perkalian.

168
Petunjuk Level 1000 Petunjuk

1031. Perpanjang FD hingga memotong BC, kemudian lakukan observasi pada


segitiga-segitiga yang sebangun.

1032. (2005) Perhatikan fakta bahwa dua garis berpotongan jika dan hanya
jika mereka tidak sejajar.

1033. (2006) Gunakan Binomial Newton untuk mencari ekspresi yang ekuiva-
len dengan koefisien x4 .

1034. Misalkan α adalah besar sudut A 1 O A 2 , lalu gunakan trigonometri untuk


menyatakan luas dan keliilng bangun.

1035. Selidiki banyaknya cara meletakkan 3 benteng pada baris berbeda.

1036. Lihat bahwa segitiga CDE sebangun dengan segitiga C AB.

1037. (2007) Perhatikan bahwa ABOP adalah segiempat tali busur.

1038. Substitusikan x dengan − x.

1039. Tinjau faktor-faktor prima dari 1515 , serta faktor-faktor prima yang
mungkin dari a dan b.

1040. (2008) Bagi kasus berdasarkan banyaknya 0 di (a, b, c, d).

1041. Gunakan prinsip komplemen.

1042. Perhatikan bahwa (5, 12, 13) merupakan tripel Pythagoras.

1043. Gunakan prinsip penjumlahan dan prinsip perkalian.

1044. Gunakan ketaksamaan segitiga untuk membatasi nilai s.

1045. Selidiki hubungan kesebangunan antarsegitiga.

1046. (2009) Pandang keadaan tersebut sebagai sebuah graf.

1047. (2010) Gunakan pendekatan analitik.

1048. Gunakan ketaksamaan rataan aritmetik-rataan geometrik (AM-GM).

1049. (2011) Faktorkan bentuk n3 + 8. Selidiki kemungkinan nilai dari faktor-


faktor tersebut.

1050. Lihat bahwa bentuk ab + bc + cd + da bisa difaktorkan.

169
Buku KTO Matematika 1

1051. Perhatikan 17 bilangan cacah terkecil.

1052. (2012) Coba dari kasus yang lebih kecil (substitusi satu variabel dan ten-
tukan kedua variabel lainnya) untuk mendapatkan pola.
GA GC
1053. (2013) Nyatakan FG dan FG dalam panjang jari-jari lingkaran (D) dan
(E).

1054. Gunakan ketaksamaan rataan aritmetik-rataan kuadratik (AM-QM).

1055. (2014) Definisikan ulang soal.

1056. Tidak ada fungsi f yang memenuhi.

1057. (2015) Misalkan banyaknya medali emas, perak, dan perunggu yang di-
raih adalah x, y, dan z, berturut-turut.

1058. Gunakan sifat kuasa titik E pada lingkaran luar persegi ABCD.

1059. (2016) Apa yang terjadi pada f (y) jika f (x) = y?

1060. (2017) Gunakan teknik teleskoping.

1061. (2018) Buktikan bahwa jika i adalah bilangan bulat terbesar sehingga a i
bukan 0, kita punya a i = 1.

1062. (2019) Perhatikan bahwa faktor persekutuan terbesar dari a n − 1 dan


a m − 1 adalah a k − 1, dengan k adalah faktor persekutuan terbesar da-
ri n dam m.

1063. (2020) Gambar ilustrasi soal. Misalkan sisi AB terdekat dengan titik E,
P dan Q berturut-turut adalah proyeksi titik E pada AC dan BD.

1064. Gunakan sifat kesebangunan dan hubungannya dengan perbandingan


luas antarsegitiga.

1065. Kalikan kedua ruas dengan 2 x − 1.


p
1066. (2021) Coba cari cara untuk menghilangkan bentuk 69.

1067. (2022) Misalkan lingkaran-lingkaran (B, BP) dan (C, CP) berpotongan di
P, Q. Lihat bahwa PQ tegak lurus dengan BC.

1068. (2023) Perhatikan bahwa segitiga DBH sebangun dengan segitiga ABC.

170
Petunjuk Level 1000 Petunjuk

1069. Gunakan aturan sinus secara berulang.

1070. (2024) Perhatikan Q(x) = P(x) − (10x + 10) merupakan suku banyak ber-
derajat 3.

1071. (2025) Nyatakan (xk )n menjadi ekspresi dalam f (xk ).

1072. (2026) Ubah persamaan soal menjadi (1 + a)(1 + b) = 1 + k.

1073. Perhatikan bahwa ABCD adalah trapesium sama kaki; gunakan teore-
ma Pythagoras.

1074. (2027) Tinjau fakta bahwa t(n) = n untuk n ganjil.

1075. (2028) Perhatikan digit satuan dari bilangan-bilangan spesial berurutan.

1076. (2029) Tentukan faktorisasi prima dari ekspresi pada soal.

1077. Perhatikan bahwa | X ∩ Y | + | X ∩ (S \Y )| = | X |.

1078. (2074) Substitusi x dengan f (x) pada persamaan awal, lalu tunjukkan
bahwa dipunyai f ( f (x)) = c + b xc untuk suatu konstanta c.

1079. Bagi kasus berdasarkan kemungkinan banyaknya nanas dan apel yang
dibeli.

1080. (2030) Ingat bahwa | x| ≥ 0.

1081. (2031) Ekspresikan pecahan tersebut dalam beberapa nilai f .

1082. Aplikasikan aturan sinus secara berulang.

1083. (2032) Gunakan pendekatan analitik.

1084. (2033) Gunakan aturan cosinus pada segitiga A X C dan A X B.

1085. Tinjau sisa pembagian kedua ruas dengan 24.

1086. (2034) Cerminkan titik P terhadap sumbu-x positif, misalkan hasilnya


adalah titik P 0 .

1087. (2075) Misalkan C 0 dan D adalah titik perpotongan lingkaran luar segiti-
ga AB0 C 0 dan BC 0 A 0 . Tunjukkan bahwa titik D terletak pada lingkaran
luar segitiga C A 0 B0 .

171
Buku KTO Matematika 1

1088. Jika H adalah proyeksi B pada GE dan I adalah proyeksi F pada BH,
lihat bahwa segitiga FBI kongruen dengan segitiga GFC.

1089. (2035) Diberikan sebuah papan 3 × 3 yang dua kotaknya sudah diwarnai
biru, dan kedua kotak tersebut berada di baris dan kolom yang berbeda.
Pandang banyaknya cara mewarnai dua kotak yang belum diwarnai de-
ngan warna merah sehingga kedua kotak yang diwarnai dengan warna
merah berada di baris dan kolom yang berbeda.

1090. (2036) Masukkan x = y = 1, dan dapatkan semua nilai yang mungkin


untuk f (1).

1091. (2037) Misalkan A(n) menyatakan banyaknya cara Cheryl menaiki n


anak tangga dengan setiap langkah, Cheryl menaiki satu, dua, atau tiga
anak tangga dengan syarat tidak boleh menaiki satu anak tangga dua
kali berturut-turut. Cari A(1), A(2), A(3), A(4) dan relasi rekurensi dari
A(n).

1092. Gambarkan grafik untuk mempermudah visualisasi.

1093. (2038) Misalkan garis tinggi dari sudut A memotong BC di F.

1094. (2039) Perhatikan bahwa B, I, dan O kolinear.

1095. (2040) Carilah bentuk lain (yang tidak melibatkan fungsi tangga) dari N.

1096. (2041) Misalkan (x − y)2 = t.

1097. (2042) Misalkan titik E terletak pada perpanjangan garis AD ke arah D


sehingga sudut ABE siku-siku.

1098. (2043) Manfaatkan teknik mengubah bentuk desimal bilangan rasional


ke bentuk pecahan.

1
1099. Pindahkan b−2015 ke kiri dan a ke kanan.

1100. (2044) Perhatikan bahwa hanya ada paling banyak satu pemain yang
tidak terkalahkan.

1101. (2045) Lihat bahwa B, I, dan I B kolinear.

172
Petunjuk Level 1000 Petunjuk

1102. (2046) Perhatikan bahwa sebuah petak dapat dijangkau hanya dari petak
sebelumnya atau 2 petak sebelumnya, terkecuali untuk petak ke-15 dan
petak ke-16.

1103. (2047) Mulai dengan angka yang kecil. Ubah bentuk soal agar menjadi
lebih terjangkau.

1104. (2048) Perhatikan perubahan luas kertas, dan dapatkan perbandingan


luas antardaerah hasil lipatan.

1105. (2049) Definisikan fungsi baru g(x) sebagai perantara.

1106. (2050) Lihat bahwa bilangan kelipatan 2 atau 5 tidak mungkin kuat.

1107. (2051) Gunakan ketaksamaan Cauchy-Schwarz untuk membatasi nilai


dari a + b + c.

1108. (2052) Pindahkan penyebut ke ruas kanan.

1109. Carilah bentuk umum koefisien a i dengan teorema Binomial Newton.

1110. Misalkan AD = 3x. Gunakan teorema titik kuasa terhadap lingkaran


dari titik A dan dari titik D. Kemudian, gunakan teorema Stewart.

1111. (2053) Perhatikan bahwa kita bisa misalkan x ≥ y ≥ z.

1112. (2054) Buatlah definisi baru untuk n − φ(n).

1113. Gunakan teorema Pythagoras.

1114. Pertama, selidiki faktorisasi prima dari 2015. Kemudian, dengan mem-
perhatikan definisi fungsi d, konstruksi bilangan yang faktorisasi prima-
nya adalah 2015 bilangan prima berbeda dengan eksponen 2014. Terak-
hir, lakukan pemasangan semua faktor dari n dan tinjau saat di mana
pemasangan tidak bisa dilakukan.

1115. (2055) Cari kondisi yang selalu tetap (invariant).

1116. (2056) Katakan dua kotak sediagonal jika garis yang menghubungkan
kedua kotak sejajar dengan diagonal utama papan catur. Perhatikan
bahwa kita bisa mengelompokkan semua kotak selain ujung kiri bawah
dan ujung kanan atas menjadi beberapa kelas sehingga setiap kotak pa-
da masing-masing kotak semuanya sediagonal.

173
Buku KTO Matematika 1

1117. (2057) Substitusi y = 0 pada persamaan di soal, lalu carilah semua yang
mungkin untuk g(0).

1118. (2058) Perhatikan bahwa hasil hompimpa adalah independen, sehingga


kita dapat menggunakan nilai peluang hasil hompimpa berhasil atau ga-
gal untuk satu ronde dalam mendapatkan solusi.

1119. (2059) Tunjukkan n genap.

1120. (2060) Lakukan observasi untuk kasus-kasus yang lebih kecil.

1121. (2061) Lihat bahwa setiap pertandingan minimal menghasilkan 2 poin.

1122. (2062) Ingat bahwa besar sudut keliling dan sudut garis singgung adalah
setengah sudut busur yang dipotong.

1123. Tinjau nilai dari (a + b)(a + c).

1124. (2063) Buktikan abcde f g merupakan bilangan kubik.

1125. (2064) Pandang pasangan-pasangan segitiga yang sebangun.

1126. Gunakan ketaksamaan rataan kuadratik-rataan aritmetik (QM-AM).

1127. (2065) Bagi kasus berdasarkan banyaknya kotak satuan yang terpilih
pada kotak 2 × 2 di bagian tengah.

1128. (2066) Misalkan x y = a dan x2 + y2 = b.

1129. (2067) Apakah konkatenasi dari 2 bilangan yang habis dibagi 2015 juga
habis dibagi 2015?

1130. Lihat bahwa jika d adalah pembagi spesial, kita punya d(d + 1) = n atau
n
d +1 bukan pembagi spesial.

1131. (2068) Buktikan bahwa ∠BHC = ∠BOC = ∠BIC.

1132. (2069) Faktorkan n4 + n2 + 1.

1133. (2070) Lihat bahwa gajah yang berada pada kotak berbeda warna tidak
akan saling menyerang.

1134. (2071) Jika ruas kiri adalah S, pandang nilai dari S − 2. Ganti setiap
suku f1k dengan f k−1 +1 f k−2 .

174
Petunjuk Level 1000 Petunjuk

1135. (2072) LIhat bahwa A 0 dan B0 dapat terletak pada sisi yang sama mau-
pun berbeda terhadap X Y .

1136. (2073) Perhatikan komposisi dari baris dan kolom (apa warna yang do-
minan) pada sebuah kotak spesial.

175
Buku KTO Matematika 1

8.2 Petunjuk Level 2000


2001. Pandang kelipatan persekutuan terkecil dari koefisien-koefisien persa-
maan untuk mendapatkan solusi lainnya.

2002. Bagi menjadi 2 kasus, yaitu b, c ≥ 2 dan b = 1 atau c = 1, lalu selidiki nilai
a yang mungkin.

2003. Cari jarak A dan C ke BD.

2004. Cari kemungkinan digit kedua bilangan N dengan menjabarkan kondisi


yang diberikan.

2005. Bagi garis-garis menjadi beberapa kelompok berdasarkan hubungan ke-


sejajaran, kemudian cari banyaknya perpotongan.
k ¡ ¢
P i ¡k+1¢
2006. Buktikan kalau t = t+1 .
i= t

2007. Gunakan teorema Ptolemy.

2008. (3001) Cari bilangan-bilangan a, b, c sehingga terdapat d yang mengaki-


batkan ab − cd habis dibagi 11.

2009. Buktikan bahwa n ≤ 500 dengan memandang banyaknya hubungan pe-


ngenalan dari himpunan n orang pertama ke himpunan sisa orang lain
yang menghadiri pesta tersebut.

2010. Misalkan AC dan BD adalah sumbu-x dan sumbu-y, berturut-turut.

2011. Pandang bahwa bilangan asli yang memiliki paling banyak 3 pembagi
positif hanyalah bilangan prima atau kuadrat dari bilangan prima.

2012. (3002) Apa yang terjadi jika c ≡ k mod 13 untuk k = 1, 2, . . . ?

2013. Nyatakan panjang FG dalam panjang jari-jari lingkaran (D) dan (E).

2014. Perhatikan bahwa soal ini sebenarnya meminta kita untuk mencari sisa
2015
pembagian 20152015 oleh 16.

2015. Lihat bahwa z haruslah merupakan bilangan genap.

2016. (3003) Bagi kasus berdasarkan banyaknya titik tetap (titik tetap adalah
x sedemikian sehingga f (x) = x.

176
Petunjuk Level 2000 Petunjuk

2017. Lihat bahwa a k = (k + 1)2 k! − k2 (k − 1)!.

2018. Buktikan bahwa jika i bilangan bulat terbesar sehingga a i bukan nol,
untuk setiap j yang lebih kecil dari i, kita selalu punya a 1 .5 + a 2 .52 +
a 3 .53 + a 4 .54 + a 5 .55 + a 6 .56 > 0.

2019. Ambil x = 2, dan buktikan bahwa n + 1|1000.

2020. (3004) Simpulkan titik A terdekat dengan E, lalu hitung jarak A ke pusat
persegi.
p
2021. (3005) Perhatikan suku-suku dari (9 − 69)n .

2022. Lihat bahwa besar sudut ACB adalah dua kali besar sudut QP A.
BH
2023. Perhatikan bahwa sin ∠DCB = BC .

2024. Ada tak berhingga banyak nilai yang mungkin dari P(2) + P(−2).

2025. Gunakan teorema Vieta.

2026. Perhatikan bahwa suatu bilangan adalah prima jika dan hanya jika fak-
tornya hanyalah 1 dan bilangan itu sendiri.

2027. Nyatakan jumlahan t(k) untuk k = n + 1, n + 2, . . . , 2n dalam n.

2028. (3006) Buktikan bahwa tidak mungkin angka-angka 3, 4, 6, 7, 8, 9 beru-


lang sebagai angka satuan bilangan-bilangan spesial berurutan.

2029. Perhatikan syarat cukup dan perlu bagi sebuah bilangan agar bernilai
kuadrat berdasarkan nilai pangkat pada faktorisasi primanya.

2030. (3007) Bagaimana cara menyatakan 4 sebagai jumlahan bilangan-


bilangan asli?

2031. Tunjukkan bahwa nilai terkecilnya adalah 11.

2032. Tentukan luas jajar genjang dengan teknik Shoelace.

2033. Eliminasi persamaan-persamaan yang dimiliki hingga diperoleh suatu


persamaan dalam C X dan BC, lalu cek kemungkinan nilai C X dan BC.

2034. Perhatikan bahwa nilai minimum tercapai saat P 0 , Q, dan R kolinear.

177
Buku KTO Matematika 1

2035. Saat diberikan papan yang dua kotaknya sudah diwarnai biru, klasifika-
sikan kotak yang biru berdasarkan banyaknya kotak biru yang berada di
baris atau kolom yang sama dengan kotak tersebut.

2036. (3008) Tunjukkan bahwa fungsi dengan f (1) = −1 tidak mungkin meme-
nuhi persamaan pada soal.

2037. (3009) Tunjukkan bahwa A(n) = A(n − 2) + 2A(n − 3) + A(n − 4) untuk setiap
bilangan asli n ≥ 5.

2038. Perhatikan bahwa BD − CE = BF − CF.

2039. (3010) Perhatikan bahwa segitiga ABO sebangun dengan segitiga IBC.

2040. Lihat bahwa b a5 c = a− b


5 , dengan b adalah sisa pembagian a dengan 5.

2041. Ekspresikan pertidaksamaan pada soal dalam bentuk t dan A.

2042. (3011) Pandang bahwa BD 2 = AB × BE − AD × DE.

2043. (3012) Tunjukkan n genap dan manfaatkan penjabaran bentuk Binomial


Newton dari 10n = (99 + 1)n/2 untuk mendapatkan n = 198.

2044. Gunakan prinsip komplemen.

2045. (3013) Jika X adalah titik singgung dari lingkaran dalam segitiga ABC
pada sisi AB, dan Y adalah titik singgung dari lingkaran singgung luar
segitiga ABC pada sisi AB, lihat bahwa segitiga BI X dan BI B Y seba-
ngun.

2046. Terdapat 1 cara untuk sampai pada petak pertama, dan 2 cara untuk
sampai pada petak kedua. Gunakan relasi rekurensi.

2047. (3014) Nomori ulang kartu sebagai berikut: kartu ke-i dari atas diberi
nomor i.

2048. Gunakan teorema Pythagoras untuk mencari perbandingan panjang dan


lebar persegi panjang.
f ( x)
2049. Misalkan g(x) = x+1 .

2050. (3015) Gunakan teorema Euler untuk menunjukkan bahwa setiap bilang-
an yang bukan kelipatan 2 atau 5 adalah kuat.

178
Petunjuk Level 2000 Petunjuk

2051. (3016) Dengan nilai a + b + c yang sudah dibatasi, mulailah menebak nilai
k yang dimaksud.

2052. (3017) Temukan nilai a dan d dalam b dan c.

2053. (3018) Gunakan ketaksamaan AM-GM dan AM-HM untuk dua suku.

2054. (3019) Perhatikan bahwa n − φ(n) adalah banyaknya bilangan dari 1 sam-
pai n yang tidak relatif prima dengan n, lalu tunjukkan bahwa faktor
prima terkecil dari n haruslah ≤ 15.

2055. Pandang selisih banyaknya tiap penyakit dalam modulo 3.

2056. Perhatikan bahwa di setiap kelompok kotak sediagonal hanya bisa dile-
takkan satu buah gajah.

2057. Tunjukkan bahwa g(1)g(−1) = 0.

2058. Gunakan prinsip perkalian.

2059. Tunjukkan n adalah bilangan kuadrat sempurna.

2060. (3020) Tentukan nilai konstanta k sedemikian sehingga ka n < b n+1 un-
tuk setiap bilangan asli n, dan verifikasi bahwa nilai tersebut memenuhi
ketaksamaan yang diberikan.

2061. (3021) Jika n adalah banyaknya tim, tinjau bahwa 4 ≤ n ≤ 6.

2062. (3022) Gunakan teorema kuasa titik, lalu perhatikan apakah segitiga
M X P dan MY P sebangun.

2063. Perhatikan mengapa bukti untuk bagian (a) tidak bisa digeneralisasi un-
tuk bagian (b).

2064. (3023) Lihat bahwa segitiga ADK sebangun dengan segitiga LBK. Cari
sepasang segitiga lagi yang saling sebangun.

2065. (3024) Lakukan analisis berdasarkan cycle-cycle langkah kuda.

2066. (3025) Selidiki apa yang terjadi bila a ≤ 0.

2067. Pandang jumlah digit dari konkatensi bilangan-bilangan tersebut.

2068. Buktikan bahwa ∠BAC = 60◦

179
Buku KTO Matematika 1

2069. (3026) Dengan pola, tebak nilai-nilai n yang bisa diverifikasi memenuhi
syarat dengan mudah.

2070. Cari banyak maksimal gajah yang bisa diletakkan pada kotak berwarna
sama sehingga tidak saling menyerang.

2071. (3027) Coba menggunakan teorema rataan aritmetik-rataan harmonik


(AM-HM), apakah cukup kuat?

2072. Buktikan bahwa besar sudut A A 0 C sama dengan besar sudut BB0 C.

2073. Definisikan ulang sebuah kotak spesial.

2074. (3028) Aplikasikan fungsi f pada kedua ruas persamaan soal, lalu tun-
jukkan bahwa b x + f (y)c = b xc + b f (b yc)c.

2075. (3029) Tunjukkan bahwa ∠PCB = ∠BC A 0 , ∠PBC = ∠CBA 0 , ∠PC A =


∠ ACB0 , ∠P AC = ∠C AB0 , ∠P AB = ∠BAC 0 dan ∠PBA = ∠ ABC 0 .

180
Petunjuk Level 3000 Petunjuk

8.3 Petunjuk Level 3000


3001. Tunjukkan bahwa untuk setiap a, b, c yang semuanya tidak sama de-
ngan 0, terdapat tepat satu solusi 0 ≤ d ≤ 10 sehingga ab − cd habis dibagi
11.

3002. Pandang sistem sebagai SPLDV dengan menganggap c sebagai konstan-


ta.

3003. Gunakan kombinasi dan permutasi untuk penghitungan akhir.

3004. Cek posisi titik P, apakah di dalam persegi atau di luar persegi.

3005. Lakukan penjabaran suku-suku, dan ingat bahwa 692 habis dibagi 9.

3006. Konstruksi 13 bilangan spesial berurutan dengan prinsip sangkar bu-


rung.

3007. Tentukan suku mana dari |a i − i | yang kita inginkan untuk bernilai lebih
besar dari 0, dan lakukan penyesuaian dengan bilangan-bilangan asli
yang jika dijumlahkan bernilai 4.

3008. Untuk f (1) = 2, substitusi y = 1 ke persamaan awal.

3009. Bagi kasus berdasarkan banyaknya anak tangga yang ia naiki pada lang-
kah pertama (1, 2, atau 3 anak tangga).

3010. Selidiki perbandingan panjang antarsisi.

3011. Gunakan aturan sinus untuk membuktikan bahwa ABEC adalah segi-
empat tali busur.

3012. Perhatikan bahwa

10198 − 1 1 ¡ 98 97
¢
= (100 − 1) + (100 − 1) + · · · + (100 − 1) + (1 − 1) + 99 .
992 99

3013. Gunakan rumus panjang jari-jari lingkaran dalam dan jari-jari lingkaran
singgung luar.

3014. Tinjau pengambilan kartu sesaat sebelum nomor kartu yang terambil
menjadi kecil.

181
Buku KTO Matematika 1

3015. Gunakan prinsip inklusi-eksklusi.


31
3016. Pandang bahwa k = 7 adalah kandidat yang memenuhi soal.

3017. Temukan cara untuk ‘menghilangkan’ suku c − d.


p
3018. Tunjukkan bahwa 2 3 ≥ x − z.

3019. Bagi kasus menurut besar faktor prima terkecil dari n.

3020. Tentukan nilai konstanta k sedemikian sehingga ka k+1 < b k+2 untuk se-
tiap bilangan asli k, dan verifikasi bahwa nilai tersebut memenuhi ke-
taksamaan yang diberikan.

3021. Untuk n = 4 dan n = 6, buktikan bahwa tidak ada konfigurasi yang me-
mungkinkan.

3022. Tunjukkan bahwa MQY haruslah merupakan segitiga siku-siku sama


kaki.

3023. Lihat bahwa segitiga AK B sebangun dengan segitiga MK D.

3024. Gunakan prinsip perkalian dan prinsip penjumlahan.

3025. Gunakan ketaksamaan AM-GM.


k
3026. Lihat bahwa n = 20152 memenuhi untuk setiap bilangan asli k.

3027. Gunakan teorema AM-HM berbobot (weighted AM-HM).

3028. Tunjukkan bahwa f (n) harus bernilai bulat untuk setiap bilangan real
n.

3029. Tunjukkan bahwa AB0 DC 0 segiempat talibusur (∠B0 D A 0 = ∠B0 C A 0 ).

182

Anda mungkin juga menyukai